PICU Prep Board Review Questions 2010

Ace your homework & exams now with Quizwiz!

April 2010 - Question 2 A 6-month-old male infant is brought to the emergency department following the acute onset of an irregular heart beat and poor feeding. He is afebrile, and his parents report that he was well until today. On physical examination, the alert and fussy child has a heart rate of 225 beats/min, a respiratory rate of 60 breaths/minute, and a blood pressure of 87/60 mmHg. Other than tachycardia, results of his cardiovascular examination are normal, and other findings on his physical examination are unremarkable. Chest radiography reveals a normal-size cardiac silhouette. Electrocardiography documents a narrow-complex tachycardia with inverted P waves, a regular R-R interval, no variation with respiration, and a ventricular rate of 225 (Figure). Three consecutive escalating doses of adenosine fail to resolve the tachycardia. Echocardiography reveals a structurally normal heart with only mildly reduced ventricular function (shortening fraction 27%). The infant continues to exhibit good peripheral perfusion and stable blood pressure and is admitted to the PICU for further management of the dysrhythmia. Of the following, the BEST choice for first-line treatment of the dysrhythmia is A. amiodarone B. esmolol C. flecainide D. lidocaine E. verapamil

B The infant described in the vignette most likely has atrial ectopic tachycardia (AET), a type of supraventricular tachycardia caused by an automatic focus within the myocardium. The automatic focus in cases of AET often is in an abnormal location in the atria, away from the sinus node. Sometimes it is associated with the atrial appendage. AET frequently can be resistant to medical therapy, and patients often present with secondary cardiomyopathy induced by prolonged tachycardia. Therefore, early rate control should be one of the priorities of treatment. Of the drugs listed, an infusion of esmolol offers the best chance of early and rapid control of the patient's tachycardia with the fewest adverse effects. Esmolol is an ultrashort half-life beta blocker that is delivered by continuous intravenous infusion, sometimes after an intravenous loading dose. Beta blockade can occur in as few as 2 to 10 minutes, and the drug's elimination half-life in children can be as short as 3 to 5 minutes. The rapidity of onset and short half-life make esmolol an effective and safe choice for initial control of AET and some other tachycardias. In the setting of AET, beta blockade does not treat the underlying automatic focus causing the dysrhythmia, but it offers control of ventricular rate, thereby decreasing the likelihood of developing symptoms of tachycardia-induced cardiomyopathy. For most children, the primary adverse effect of esmolol is hypotension. Because the patient in the vignette is normotensive and exhibits only a very mild reduction in ventricular function by echocardiography, the risk of inducing hypotension with esmolol is offset by the benefit of gaining control of the tachycardia. The very short half-life of the drug is attractive if significant hypotension develops. As with other drugs causing beta blockade, esmolol is contraindicated for a patient who has a history of severe bronchospasm, which the infant in the vignette does not have. Amiodarone is a potent class III antiarrhythmic drug that acts primarily on the myocardial cell potassium channels to prolong the action potential duration. Amiodarone has been incorporated into Pediatric Advanced Life Support algorithms for ventricular tachycardia and ventricular fibrillation, and it is one of the primary antiarrhythmic drugs used for postoperative junctional ectopic tachycardia. Although amiodarone can be used to control AET, it has a number of significant adverse effects and usually is not considered a first-line therapy. Amiodarone can cause significantly diminished ventricular function and severe bradycardia. In addition, it can be proarrhythmic, with a particular risk of precipitating torsades de pointes, especially in the setting of uncorrected hypokalemia or hypomagnesemia. Rapid administration of intravenous amiodarone may cause fatal For a patient who has a perfusing rhythm, a recommended loading dose should be administered no more rapidly than over 20 to 60 minutes. Flecainide is a class IC antiarrhythmic drug that inhibits sodium channels in fast-response cells and has a marked depressant effect on conduction velocity. Clinically, flecainide can be effective at depressing abnormal automaticity, as seen in AET, but it can have serious proarrhythmic effects, including induction of ventricular tachycardia. Flecainide can be administered only by an enteral route, and its time to peak concentration can be as much as 6 hours. Its elimination half-life in infants and children is typically 8 to 12 hours. Although it has been used successfully to treat AET, it generally is not used as first-line therapy. Lidocaine is a venerable class IB antiarrhythmic drug that is used primarily for the treatment of ventricular dysrhythmias. It is recommended in the Pediatric Advanced Life Support algorithms as an alternative to amiodarone for treatment of ventricular tachycardia or ventricular fibrillation. Lidocaine does not have a role in the treatment of supraventricular tachycardias such as AET. Verapamil is a calcium channel-blocking drug (class IV antiarrhythmic) that blocks the slow, inward calcium current in the sinus and atrioventricular nodes, which decreases sinus node automaticity, slows AV node conduction, and prolongs refractoriness. Although verapamil can be useful in treating re-entrant type supraventricular tachycardia, its use is contraindicated in children younger than 1 year of age due to reports of cardiovascular collapse in this age group after the administration of intravenous doses. The infant's myocardium is particularly dependent on free intracellular calcium for adequate function, making calcium channel-blocking agents a dangerous choice in this age group. A recent review of cases of AET over a 9-year period at a large pediatric cardiology program revealed interesting differences in the course of AET based on age at presentation. Children younger than 3 years of age at presentation were more likely to have AET controlled with medication and to have spontaneous resolution than children who were older than 3 years of age at onset. Among the older children, only 37% responded to antiarrhythmic medication and only 16% experienced spontaneous resolution. The investigators recommended early pursuit of radiofrequency ablation of the abnormal automatic focus in older children who had AET. In the younger children in the series, control of AET was achieved most often with a beta-blocking drug, digoxin, or both. In both older and younger children, radiofrequency ablation frequently was successful at extinguishing the ectopic automatic focus of the tachycardia. References: Salerno JC, Kertesz NJ, Friedman RA, Fenrich AL Jr. Clinical course of atrial ectopic tachycardia is age-dependent: results and treatment in children < 3 or ³ 3 years of age. J Am Coll Cardiol. 2004;43:438-444. Abstract accessed October 2009 at: http://www.ncbi.nlm.nih.gov/pubmed/15013128 Moak JP. Dysrhythmias. In: Slonim AD, Pollack MM, eds. Pediatric Critical Care Medicine. Philadelphia, Pa: Lippincott Williams & Wilkins; 2006:644-658 American Board of Pediatrics Content Specification(s): Differentiate the effects of various classes of drugs on impulse conduction Understand the pathogenesis of disorders of impulse conduction Know the treatment of disorders of impulse conduction

Dec 2010 - Question 2 An obese, 7-year-old boy is admitted to the pediatric intensive care unit after several days of vomiting and distress with hypotension, poor perfusion, and right lower quadrant pain. The Figure shows his monitor strip. He is agitated and uncooperative, although weak and moderately obtunded. His only intravenous access is a tenuous catheter secured to the back of his hand. You elect to intubate him and then obtain central venous access. Figure: The patient's monitor strip Which of the following agents would be the BEST choice for rapid sequence intubation? A. etomidate B. fentanyl C. ketamine D. midazolam E. propofol

B The patient described in the vignette has hypotension and may be septic. Sepsis itself impairs myocardial function, so, in this context, anesthetic agents pose an increased risk of cardiovascular collapse. The primary useful effects of anesthetics are sedation, analgesia, anxiolysis, amnesia, and euphoria, although they also have many other effects, such as respiratory depression, that must be considered in context when they are used. Virtually all anesthetic agents impair sympathetic activity and have significant, dose-related hemodynamic effects. The most important of these are myocardial depression, alteration of vascular resistance (usually vasodilation), and reduced heart rate. Choice of the best anesthetic for rapid sequence intubation of this patient is dominated by hemodynamic considerations. In general, opiates cause relatively little myocardial depression and cause less vasodilation than other agents in common use. Fentanyl produces rapid onset of sedation and excellent analgesia. It causes moderate slowing of the heart and little vasodilation and hence little hypotension and little impairment of myocardial perfusion. Fentanyl is also suitable for maintenance of sedation once intubated. Of the available responses, fentanyl is preferred. An alternative choice for a child such as the one described in the vignette is ketamine. Although a myocardial depressant, ketamine releases endogenous norepinephrine, which supports cardiac contractility and maintains or elevates systemic vascular resistance, blood pressure, and cardiac output. It is, therefore, generally associated with hypertension rather than with hypotension. Because of this child's premature ventricular contractions, abrupt norepinephrine release could prove catastrophic. In patients who are catecholamine depleted (from persistent high sympathetic output), ketamine may cause uncompensated myocardial depression. This adds to the risk of ketamine induction in a setting of long-standing stress. Ketamine is believed to have specific benefits in the face of endotoxemia and sepsis through blunting of the proinflammatory cytokine response. It may reduce the interleukin 6 and tumor necrosis factor a responses to endotoxin by inhibition of NF-κB. Although experimental evidence suggests benefits in animal models, there are few human data to support ketamine's specific benefits in sepsis. Etomidate has little effect on myocardial contractility and is a popular induction agent for rapid sequence intubation but causes adrenal suppression. A single dose is associated with cortisol deficiency and decreases the efficacy of catecholamines, so it is an undesirable option for rapid sequence intubation in this patient. Propofol has a significant adverse effect on blood pressure in patients dependent on intense adrenergic activity to maintain vascular tone, such as certain patients with sepsis. This increases its risk in the presence of sepsis. Midazolam, which generally has mild hemodynamic effects, can cause exaggerated myocardial depression in sepsis. It produces amnesia, which is a desirable characteristic but is not an analgesic, so it is generally given with another anesthetic during rapid sequence intubation. References: Hohl CM, Kelly-Smith CH, Yeung TC, Sweet DD, Doyle-Waters MM, Schulzer M. The effect of a bolus dose of etomidate on cortisol levels, mortality, and health services utilization: a systematic review. Ann Emerg Med. 2010;56:105-119. DOI: 10.1016/j.annemergmed.2010.01.030. Abstract accessed September 2010 at: http://www.ncbi.nlm.nih.gov/pubmed/20346542. Morris C, Perris A, Klein J, Mahoney P. Anaesthesia in haemodynamically compromised emergency patients: does ketamine represent the best choice of induction agent? Anaesthesia. 2009;64:532-539. DOI: 10.1111/j.1365-2044.2008.05835.x. Accessed September 2010 at: http://onlinelibrary.wiley.com/doi/10.1111/j.1365- 2044.2008.05835.x/full. Ray DC, McKeown DW. Effect of induction agent on vasopressor and steroid use, and outcome in patients with septic shock. Crit Care. 2007;11:R56. DOI: 10.1186/cc5916. Accessed September 2010 at: http://www.ncbi.nlm.nih.gov/pmc/articles/PMC2206408/? tool=pubmed. Sun J, Wang XD, Liu H, Xu JG. Ketamine suppresses endotoxin-induced NF-kB activation and cytokines production in the intestine. Acta Anaesthesiol Scand. 2004;48:317-321. DOI: 10.1111/j.0001-5172.2004.0312.x. Abstract accessed September 2010 at: http://www.ncbi.nlm.nih.gov/pubmed/14982564. American Board of Pediatrics Content Specification(s): Recognize the effects of anesthetic drugs on myocardial function, heart rate, and systemic vascular resistance Know that narcotics cause the least depression of myocardial function of all anesthetics Recognize that a patient with sepsis may have impaired myocardial function and hence an exaggerated risk of cardiac depression during anesthesia

August 2010 - Question 1 A 4-year-old boy is hit by a car and brought to the emergency department by his parents. The trauma team finds the boy breathing spontaneously with gurgling noises associated with bloody fluid in the mouth and nose. He is moaning incoherently. Measurement of his vital signs reveals the following: temperature, 36.5°C; heart rate, 143/min; respiratory rate, 14/min; blood pressure, 91/70 mm Hg; and oxygen saturation on mask oxygen, 96%. He has a palpable step off on the right parietal skull with local ecchymosis and edema. The right pupil is 6 mm and not reactive. The left pupil is 3 mm and reactive. Breath sounds are slightly louder on the left than the right side. Heart tones are rapid but regular without murmur, rub, or gallop. Pulses are thready, with capillary refill at 4 seconds. The abdomen is distended and somewhat firm, but there are no palpable masses and no organomegaly. Bowel sounds are absent. His pelvis and genitourinary examination results are normal. The right femur is shortened, outwardly rotated, and has ecchymoses and edema. There is no response to voice, but he opens his eyes and withdraws extremities to painful stimuli. This movement is stronger on the right than the left side. After rapid sequence orotracheal intubation, orogastric tube placement, placement of a cervical collar, placement of 2 large-bore peripheral intravenous catheters, and administration of 20 mL/kg of normal saline bolus, the patient undergoes computed tomography (CT) of the head, chest, and abdomen. The CT demonstrates an epidural hematoma on the right side with 5 mm of midline shift, a small right hemopneumothorax with opacities suggesting right lung contusion or aspiration in the lower lobe, and a grade 3 liver laceration with free fluid in the peritoneal space. Of the following, the BEST next step in the treatment of this patient is to: A. insert a thoracostomy tube B. perform a craniectomy C. place arterial and central catheters D. reduce femur fracture E. transfuse packed red blood cells

B Trauma is the main cause of childhood mortality. A standardized team approach to the evaluation and stabilization of a child with severe trauma significantly reduces mortality and morbidity. The primary survey of a trauma patient includes evaluation of the following: A - airway, with attention to possible cervical injury to avoid harming the spinal cord B - breathing C - circulation D - disability, neurologic evaluation and Glasgow Coma Scale (GCS) score E - exposure, to ensure that hidden injuries are exposed while avoiding hypothermia During this primary survey, the patient should receive cardiorespiratory monitoring, undergo pulse oximetry measurement, have blood pressure checked regularly, and have end-tidal carbon dioxide measured when intubated. Strong consideration should be given to placement of a urinary catheter (unless there is suspicion of urethral injury, such as blood at the meatus) and gastric tube (avoiding the nose should there be evidence of basilar skull fracture, epistaxis, coagulopathy, or cerebrospinal fluid rhinorrhea). Intubation of the patient described in the vignette is appropriate because he has a compromised airway from bloody secretions and a GCS score of only 7. Intubation would also be expected to improve breathing, although positive pressure ventilation could increase the size of a pneumothorax and lead to tension pneumothorax. His rapid heart rate, narrow pulse pressure, and poor pulses and perfusion suggest that he needs volume resuscitation, although tension pneumothorax and pericardial tamponade also need to be considered. After the primary survey led to a secure airway, vascular access, and attention to improving circulating blood volume, the child undergoes CT. Although the CT scan confirms many injuries needing attention, the most immediately life-threatening injury, which could also lead to permanent disability, is his epidural hematoma. Therefore, it is essential that the patient undergoes emergent craniectomy to remove the epidural hematoma and stop ongoing intracranial hemorrhage. Intensive care physicians are used to placing pressure monitoring catheters in central veins and arteries. These catheters help us with the minute to minute monitoring that guides our therapy. However, the most emergent need for the patient described in the vignette is to proceed to the operating room as fast as possible to undergo removal of the epidural hemorrhage and have his multiple other problems addressed. The placement of central venous and arterial catheters should be deferred until the life-threatening surgical conditions have been addressed. A femur fracture is a serious injury that can bleed enough to cause hemorrhagic shock. Femur fractures can lead to nerve and vascular compromise that could lead to permanent damage to the leg. Therefore, a trauma patient should be carefully screened for this injury, and the distal pulses, perfusion, and sensory and motor nerve function should be assessed. The blood loss from a femur fracture can be addressed with isotonic volume resuscitation and, if necessary, packed red blood cell transfusion. Immobilization alone usually reduces bleeding satisfactorily to allow some delay in formal reduction, with or without surgical intervention. Although potentially limb-threatening, the more lifethreatening injuries must take precedence. It would be reasonable to have an orthopedic surgeon evaluate and treat the femur fracture in the operating room after a neurosurgeon has controlled the epidural hemorrhage. There is little question that the patient described in the vignette has lost a fair amount of blood from his many injuries. Transfusion may well become necessary, and the physician should send blood samples for typing and cross-matching as soon as possible to get packed red blood cells, and potentially other blood components, ready to transfuse. Type O negative cells must also be considered if the blood loss is so great that blood pressure is not supported with 60-mL/kg crystalloid fluid boluses before type-specific packed red blood cells are available. In nonemergent surgical procedures, physicians should perform transfusion if necessary to achieve satisfactory hemoglobin and hematocrit levels. In this emergent situation, transfusion should not delay getting the patient to the operating room to treat the potentially lethal intracranial injury. Rather, transfusion should be ordered as needed to support the patient as he is being transported to the operating room and throughout the operation as needed. The patient described in the vignette has only received 20 mL/kg of crystalloid and should receive more isotonic crystalloid before assuming transfusion is necessary. Hemopneumothorax is a common injury from major blunt trauma. If during the primary trauma survey (see above) a patient has absent breath sounds on 1 side, clinical evidence of tension pneumothorax with poor circulation, or deviation of the trachea or cardiac impulse, it is necessary to perform a needle thoracotomy even before imaging studies. This is followed by formal insertion of a thoracostomy tube. If a hemopneumothorax is not found clinically, but only radiographically, then there is no absolute indication to place a chest tube. The physician must closely monitor a patient for the possibility that further air leak could lead to impaired gas exchange or even tension pneumothorax or that ongoing hemorrhage could lead to inadequate gas exchange. If either were to occur, thoracostomy would be indicated. The patient described in the vignette may or may not require chest tube placement because the small pneumothorax seen on the CT may resolve without therapy. If gas exchange is poor because of evolving lung injury from lung contusions, higher pressures will likely be required to achieve adequate tidal volumes, oxygenation, and ventilation. If peak inspiratory pressures reach approximately 30 cm H2O, the physician may consider evacuating the pleural gas and any related fluid to optimize lung expansion and compliance. References: Lee LK, Fleisher ER. Trauma management: approach to the unstable child. UpToDate. Accessed March 2010 at: http://www.uptodate.com/online/content/topic.do? topicKey=ped_trau/8997&selectedTitle=3%7E150&source=search_result. Mattox KL, Feliciano DV, Moore EE, eds. Trauma. 4th ed. Columbus, OH: McGraw-Hill; 2000:155-170, 236-242, 377-399, 473-482, 636-648, 1075-1096. Mejia R, ed. Traumatic injuries in children. In: Pediatric Fundamental Critical Care Support. Mount Prospect, IL: Society of Critical Care Medicine; 2008:10-1 to 10-16. Moore FA, Moore EE. Trauma resuscitation. In: Wilmore DW, Cheung LY, Harkin AH, et al, eds. ACS Surgery, Principles and Practice. New York, NY: WebMD; 2002:31-47. American Board of Pediatrics Content Specification(s): Identify the situations in which immediate surgical intervention is required Know the general principles and sequence of the concurrent evaluation, stabilization, and prioritization of injuries Know the physical findings of traumatic hemothorax Plan the treatment of a patient with epidural hematoma

Feb 2010 - Question 6 You are designing a phase I clinical trial to assess the effect of a new medication on hypoxic brain injury. Although preliminary data are scant, they suggest a dramatic beneficial effect on neurologic outcomes with little reported toxicity. You have decided to limit the trial to Englishspeaking individuals because no one involved in the research is fluent in any other language and there are no available funds for translation fees (verbal or written). When your proposal is discussed at the Institutional Review Board (IRB) meeting, a colleague objects to the proposal because of this limitation. He cites a growing Hispanic population in the community and contends that this limitation denies the Spanish-speaking people of the community access to a potential therapy. Of the following, his contention is MOST supported by the basic bioethical principle of A. autonomy B. beneficence C. justice D. nonmaleficence E. respect for persons

C In 1978, the National Commission for the Protection of Human Subjects of Biomedical and Behavioral Research published the report "Ethical Principles and Guidelines for the Protection of Human Subjects of Research." It was named the Belmont Report for the Belmont Conference Center where the National Commission met when first drafting the report. The report defined three fundamental ethical principles for all human subject research: 1) respect for persons, 2) beneficence, and 3) justice. Corollary to respect for persons is autonomy, and corollary to beneficence is absence of maleficence. These principles are the basis for the Department of Health and Human Services human subject protection regulations. For the scenario described in the vignette, the principle of justice supports the IRB member's contention. Justice refers to the principle of fairness in distribution and incorporates the concept that research benefits and burdens should be shared equally. Throughout recent history, examples of violation of the principle of justice have been reported, including the use of unwilling prisoners as research subjects by Nazi physicians. This serves as an example of injustice as these unwilling individuals were disproportionately exposed to the risks of research. In addition, research that does not provide benefit to all participants equally or inappropriately denies benefit to specific individuals also violates the principle of justice. Neither respect for persons nor beneficence applies to the exclusion of Spanish-speaking individuals in the proposed trial. The principle of "respect for persons" encompasses the concept of autonomy, asserting that people should be treated as free-thinking individuals. Moreover, the principle of autonomy assures the protection of individuals who have diminished autonomy. Among those who have diminished autonomy are pediatric subjects. The principle of respect for persons underlies the need for informed consent and the protection of vulnerable subjects. This principle assures that research subjects are provided adequate information to make an educated decision to participate in research and that they participate voluntarily. The principle of beneficence asserts the need to maximize possible benefits and minimize possible harms. It also incorporates the concept of nonmaleficence or the Hippocratic maxim of "do no harm." Claude Bernard extended this principle to the realm of research, advocating that "one person should not be injured regardless of the benefits that might come to others." No data in the proposed trial suggest a violation of the principle of beneficence or the concept of nonmaleficence. References: Office for Human Research Protections. Belmont Report. Washington, DC: United States Department of Health & Human Services. 2008. Accessed September 2009 at: http://www.hhs.gov/ohrp/belmontArchive.html Office of Human Subjects Research. The Belmont Report: Ethical Principles and Guidelines for the Protection of Human Subjects of Research. Washington, DC: National Institutes of Health; 1979. Accessed September 2009 at: http://ohsr.od.nih.gov/guidelines/belmont.html American Board of Pediatrics Content Specification(s): Recognize the ethical issues that complicate obtaining informed consent from minors in research Understand the importance of examining minority populations in clinical research

Jan 2010 - Question 1 A 5-year-old child, who has had increasing tiredness with physical activity over the past several months, is admitted for progressive respiratory distress and fever. On physical examination, the child has pronounced facial weakness with bilateral ptosis, diplopia, and weakness of the proximal muscles; diminished but present reflexes; and a temperature of 38.9°C. His chest radiograph shows a right lower lobe infiltrate. His mother reports that he has had a "reaction" to cephalosporin in the past. You initiate therapy with ampicillin and gentamicin for presumed pneumonia and order a neurology consultation. One hour later, you are called emergently to the child's bedside for respiratory arrest. The child exhibits apnea, and his airway is being maintained easily with bag-valve-mask ventilation. Of the following, the MOST likely explanation for the exacerbation in this child's condition is A. acute exacerbation of Werdnig-Hoffman disease B. elevation in intracranial pressure from an infiltrating brainstem C. exacerbation of myasthenia gravis due to aminoglycoside administration D. probable allergic reaction to ampicillin E. progression of Guillain-Barré syndrome to involve the respiratory muscles

C Myasthenia gravis is an autoimmune disorder of peripheral nerves that affects transmission at the neuromuscular junction and is characterized by decremental repetitive nerve conductions and a positive response to neostigmine. The central defect is the formation of antibodies against acetylcholine (ACh) nicotinic postsynaptic receptors at the myoneural junction. This directly results in reduced receptor availability and a characteristic pattern of weakness and decreasing muscle strength with repeated use. Rest allows more receptors to become available, and recovery of muscle strength after rest is a hallmark of the disease. The disease most commonly affects the ocular and bulbar muscles, with ptosis or diplopia being the presenting signs in most cases, as described for the child in the vignette. Generalized weakness frequently is present as well and may include the respiratory muscles, creating a true emergency. If the respiratory muscles are affected, the gag reflex may be absent, creating conditions for aspiration pneumonia. Numerous medications could exacerbate the weakness and progressive respiratory distress described for the child in the vignette. These include antibiotics (macrolides, fluoroquinolones, aminoglycosides, tetracyclines, and chloroquine), antidysrhythmic agents, antiseizure medications, hormones, and steroids. In this case, treating the childs pneumonia with gentamicin exacerbated the condition, progressing to true respiratory failure. The acuity of the childs presentation and presence of reflexes rules out Guillain-Barré syndrome. Although infiltrating brainstem lesions and other mass lesions can cause multiple cranial neuropathies, they rarely are symmetric and do not cause proximal weakness. The ease with which the childs airway is secured suggests that an allergic or anaphylactic reaction is unlikely. Werdnig-Hoffman disease is a hereditary form of spinal muscular atrophy that presents in infancy and rarely effects eye movements.

Dec 2010 - Question 5 A 15-year-old girl is brought to the emergency department with a suspected heroin overdose. She is obtunded and presents with shallow breathing. Track signs are visible on both upper extremities. Her pupils are constricted. Of the following manifestations of overdose syndrome, which is MOST likely to require admission of this girl to the pediatric intensive care unit? A. bradycardia B. diarrhea C. hypertension D. pulmonary edema E. seizures

D Acute lung injury or noncardiogenic pulmonary edema has been described as a complication of heroin overdose since autopsy reports performed by William Osler in 1880. Acute lung injury may occur as early as 2 hours after parenteral use or 4 hours after intranasal use of heroin. Typical presentation includes hypoxemia, pulmonary rales, and classic pink, frothy sputum. Presumed causes of this noncardiogenic pulmonary edema in heroin users include hypersensitivity reactions, immune complex deposition in the alveolar capillary membrane, capillary leakage due to histamine release, sympathetic discharge, hypoxemia, and irregularities in the pulmonary lymphatic drainage. Respiratory depression may follow opiate administration and is due to a direct effect on the brainstem respiratory centers and a depression of the centers located in the pons and medulla responsible for the regulation of respiratory rhythm. Medullary chemoreceptors become insensitive to increases in the acidity of cerebrospinal fluid that accompany increasing carbon dioxide tension. Hypoxic drive is also diminished, resulting in apnea. The degree of respiratory depression correlates with the dose administered. Respiratory depression, part of the overdose syndrome (see below), is the main mechanism of death due to opiate overdose. Peak respiratory depression usually occurs 5 to 10 minutes after an intravenous administration of morphine or 30 minutes after an intramuscular dose. Acute muscular rigidity that involves the chest wall may occur if certain opiates, such as fentanyl, are used. Neuromuscular blocking agents and mechanical ventilation may be necessary to permit chest excursion in such cases. Although miosis is typically seen with opiate use, it may be absent in some cases. Meperidine and propoxyphene typically do not affect pupillary size. Mydriasis may be present in patients who have become hypoxic or anoxic because of an opiate overdose. Drug combinations may also alter pupillary size (heroin and cocaine, heroin and scopolamine mixes), depending on the relative concentration of each drug. Seizures are rare with an acute opiate overdose and are most likely due to hypoxia. Some opiates, including meperidine and propoxyphene, are associated with seizures. Cardiovascular effects of opiates include vasodilation with ensuing hypotension. This latter effect has been attributed to histamine release by opiates. Fentanyl is the opiate associated with the least degree of histamine release, whereas meperidine is associated with the most. Bradycardia is not common with opiate use. Propoxyphene is the only opiate associated with significant cardiovascular toxic effects. Other cardiac manifestations of heroin toxicity or of its adulterants (quinine, scopolamine) include dysrhythmias, ventricular fibrillation, and conduction abnormalities. Gastrointestinal effects of opiates include an increase in the muscular tone of the gastrointestinal tract, a decrease in peristalsis, and a decrease in intestinal, biliary, and pancreatic secretions. The anal sphincter tone is increased. Constipation is the result of these effects and explains the use of opiates as antidiarrheal agents. Urinary retention results from an increase in bladder sphincter tone. Opiates act mainly through μ (OP3) opiate receptors. Spinal and supraspinal receptors mediate the analgesic effects of opiates. In addition, changes in mood, such as euphoria, may occur, probably mediated through dopaminergic receptors. Other effects include miosis, depression of the cough reflex, nausea, and vomiting. These are due, respectively, to direct opiate effect on the parasympathetic innervations of the pupils and direct action on the cough center and chemoreceptor trigger zone for emesis in the medulla. Opiate toxicity depends on the dosage, duration, and route of administration. Opiates may be administered through various routes, including oral, parenteral, transdermal, intranasal, intrapulmonary (by smoking), epidural, and intrathecal. Protein-binding properties of opiates vary widely, from as low as 7% for codeine to over 90% for methadone. Decreased levels of albumin may result in possible toxic effects due to higher free levels of opiates. Hepatic metabolism usually involves conjugation, hydrolysis, oxidation, or dealkylation. Toxic adverse effects of some opiates may be due to the resultant metabolites. Codeine is metabolized to morphine and morphine is metabolized to morphine-6-glucuronide, which is a more active form. Meperidine metabolizes to normeperidine, which is more neurotoxic than the parent drug. Propoxyphene's metabolite norpropoxyphene is more cardiotoxic than the parent drug.

August 2010 - Question 6 A 9-year-old boy with progeria is admitted to the pediatric intensive care unit with signs and symptoms consistent with myocardial ischemia. He has left-sided chest pain during exercise radiating to his left upper extremity and chin. Electrocardiographic findings include ST-segment elevation, and you suspect unstable angina. You wish to treat your patient with a drug that will provide coronary vasodilation but are concerned about causing hypotension. Which of the following drugs is the BEST choice in this case? A. amlodipine B. diltiazem C. nicardipine D. nimodipine E. verapamil

E Calcium channel blockers (CCBs) bind to L-type calcium channels, blocking the inward movement of calcium in the cardiac myocyte, cardiac nodal tissue, and arterial vascular smooth muscle. Therapeutic uses of CCBs include treatment of hypertension, angina, and arrhythmias. The L-type calcium channels regulate the influx of calcium across the cell membrane, stimulating smooth muscle contraction and cardiac myocyte contraction. L-type calcium channels in cardiac nodal tissue play an important role in phase 0 of action potentials and in pacemaker activity. Therefore, CCBs cause vascular smooth muscle relaxation with resultant vasodilation, as well as negative chronotropy, negative inotropy, and negative dromotropy. The atrioventricular (AV) node is particularly affected. Administration of a CCB may lead to coronary artery dilation, improving myocardial oxygen delivery. However, this desired effect must be counterbalanced against the risk of hypotension and reflex tachycardia caused by peripheral vascular smooth muscle Three distinct classes of CCBs exist based on their chemical structure and unique binding sites to the L-type calcium channels. Nondihydropyridines include phenylalkylamines and benzothiazepines. These drugs are more cardioselective than the other major class of CCBs, the dihydropyridines. Verapamil is the only drug in the phenylalkylamine group. It binds to the V-binding site of the L-type calcium channel and is relatively selective for the myocardium. In addition, although it does not cause effective systemic vasodilation, it does cause vasodilation of the coronary arteries. The combination of coronary vasodilation with limited systemic vasodilation, along with its cardiodepressant actions, which reduce myocardial oxygen demand, makes verapamil an excellent choice for the treatment of angina. Dihydropyridines are the other major class of CCBs and include nifedipine, nicardipine, felodipine, amlodipine, and nimodipine. These drugs bind to the N-binding site of the calcium channel. They possess high selectivity for vascular smooth muscle, resulting in reduced systemic vascular resistance and a decrease in arterial pressure. Thus, they are primarily used to treat hypertension. Their use in the treatment of angina is limited because their potent vasodilator properties lead to reflex cardiac stimulation with resultant tachycardia, which in turn causes an undesired increase in myocardial oxygen consumption. Diltiazem is a benzothiazepine and binds to the D-binding site of the L-type calcium channel. Its effects are intermediate between verapamil and the dihydropyridines. The degree of cardiac stimulation with diltiazem is lower than with the dihydropyridines. The CCBs (specifically, verapamil, diltiazem, and nifedipine) are class IV antiarrhythmic drugs. They decrease sinus node automaticity, slow AV conduction, and increase the refractory period. The PR interval is therefore prolonged and sinus bradycardia ensues. Verapamil is useful in the treatment of reentrant tachycardia involving the AV node. It should not be used in children younger than 1 year because of reports of cardiovascular collapse with the use of intravenous verapamil in this age group. Another contraindication to the use of verapamil is Wolff-Parkinson-White syndrome. Although verapamil will slow AV conduction, accessory pathway conduction may be enhanced. Patients being treated with β-adrenergic receptor antagonists should not receive CCBs because of increased depression of cardiac electrical and contractile activities. References: Arroyo AM, Kao LW. Calcium channel blocker toxicity. Pediatr Emerg Care. 2009;25:532- 541. Eisenberg MJ, Brox A, Bestawros AN. Calcium channel blockers: an update. Am J Med. 2004;116:35-43. DOI: 10.1016/j.amjmed.2003.08.027. Abstract accessed April 2010 at: http://www.ncbi.nlm.nih.gov/sites/entrez/14706664. American Board of Pediatrics Content Specification(s): Distinguish among the calcium channel blockers with respect to their relative antidysrhythmic, negative inotropic, and vasodilating activities

Dec 2010 - Question 4 A previously healthy, 6-month-old infant is referred to the emergency department with a chief concern of "funny eye movements that would come and go" for the past 5 weeks according to the patient's mother. The mother states that she has also noted some unusual motor movements on occasion. The infant has been feeding well but became a fussy eater during the past 2 weeks. Vital signs include the following: heart rate, 120/min; blood pressure, 80/44 mm Hg; respiratory rate, 30/min; and temperature, 37oC. The pupils are slightly dilated and unequal. Figure 1 shows a photograph of the infant's head as a light is held above the infant's eyes. The infant is noted to have brisk deep tendon reflexes and an extensor plantar response. Figure 1 Which of the following BEST explains these findings? A. aqueductal obstruction B. compression of cranial nerve II C. epidural hematoma D. neuroblastoma E. spinal cord tumor

A The patient described in the vignette is demonstrating what has been called the "setting sun" eye phenomenon, which is an ophthalmologic sign in young children that results from an upward gaze paresis. As in this infant, both eyes appear driven downward, the sclera may be seen between the upper eyelid and the iris, and part of the pupil may be covered by the lower eyelid. Although the exact pathogenesis of this sign is not well understood, it appears related to aqueductal distension with compression of periaqueductal structures secondary to increased intracranial pressure. It has been called Parinaud syndrome, sylvian aqueduct syndrome, Koerber-Salus-Elschnig syndrome, and dorsal midbrain or pretectal syndrome. This entity consists of upward gaze paresis, pupillary abnormalities, lid retraction, and a loss of convergence and accommodation. The clinical abnormalities result from pressure on the dorsal midbrain and its nuclei, primarily the interstitial nucleus of the medial longitudinal fasciculus, nucleus of the posterior commissure, nucleus of Darkschewitz, and the interstitial nucleus of Cajal. Cranial nerve III (motor movement) and the Edinger-Westphal nuclei (pupillary responses) are located near the superior colliculus and the mesencephalic tectum, and thus it is cranial nerve III that is affected primarily in this entity and causes the motor and pupillary dysfunction of the eyes (Table). Table: Ocular Motor Cranial Nerves Ocular Muscle Innervation Function Lateral rectus Abducens (VI) Abduction Medial rectus Oculomotor (III) Adduction Superior rectus Oculomotor (III) Elevation, intorsion, adduction Inferior rectus Oculomotor (III) Depression, extorsion, adduction Inferior oblique Oculomotor (III) Extorsion, elevation, abduction Superior oblique Trochlear (IV) Intorsion, depression, abduction Pathophysiologic mechanisms include compression and distortion of the tectal plate, axial enlargement of the third ventricle with stretching of the third ventricle and the posterior commissure, and dilation of the rostral aqueduct with stretching of the periaqueductal gray matter. This sign is found in approximately 40% of infants and children with hydrocephalus and in 13% of patients with ventriculoperitoneal shunt malfunction. It is an earlier sign of hydrocephalus than enlarged head circumference, full fontanelle, separation of sutures, irritability, or vomiting. Patients who present with eye findings such as these need to have a neuroimaging study performed to elucidate the origin. This patient underwent magnetic resonance imaging, which revealed a choroid plexus tumor causing an obstruction of the aqueduct of Sylvius (Figure 2). Figure 2: Lateral view (A) and coronal view (B) of the patient's magnetic resonance image demonstrating a choroid plexus tumor originating in the third ventricle and causing obstructive hydrocephalus and pressure on the tectal plate. The cranial nerves responsible for ocular movement are summarized in the Table. Cranial nerve II (optic nerve) is NOT responsible for motor movements of the eye but is responsible for controlling pupillary size. The iris controls the size of the pupil and contains 2 groups of smooth muscle fibers. The sphincter pupilla is a circular constrictor and is innervated by the parasympathetic nervous system producing pupillary constriction (miosis). The dilator pupilla is a radial dilator and is innervated by the sympathetic nervous system and produces pupillary dilatation (mydriasis). A lesion on cranial nerve III (oculomotor) produces impairment of eye and lid movement and disturbance of pupillary response. An epidural hematoma, if unrecognized and untreated, may cause unilateral compression of the brain on the side of the lesion and lead to uncal herniation, which would produce a unilateral dilated pupil (which is not seen in this patient). Patients with neuroblastoma have been reported to have opsoclonus, which is defined as irregular, hyperkinetic, multidirectional spontaneous eye movements that may be accompanied by myoclonic jerks of the face and body and cerebellar ataxia. The eye movements are not always in unison in opsoclonus and can be present in sleep with a lower amplitude and rate. Patients with spinal cord tumors below the brainstem do not usually present with any eye findings. References: Abend NS, Kessler SK, Helfaer MA, Licht DJ. Evaluation of the comatose child. In: Nichols DG. Roger's Textbook of Pediatric Intensive Care. 4th ed. Philadelphia, PA: Wolters Kluwer/Lippincott Williams & Wilkins; 2008:846-861. Alberstone CD, Benzel EC, Najm IM, Steinmetz MP. Anatomic Basis of Neurologic Diagnosis. New York, NY: Thieme Medical Publishers; 2009:421-469. Jacobs DA, Galetta SL. Neuro-ophthalmology for neuroradiologists. Am J Neuroradiol. 2007;28:3-8. Accessed September 2010 at: http://www.ajnr.org/cgi/content/full/28/1/3. Maramattom BV, Wijdicks EFM. Dorsal mesencephalic syndrome and acute hydrocephalus after subarachnoid hemorrhage. Neurocrit Care. 2005;3:57-58. DOI: 10.1385/NCC:3:1:057. Abstract accessed September 2010 at: http://www.ncbi.nlm.nih.gov/pubmed/16159097. American Board of Pediatrics Content Specification(s): Know the ocular findings associated with third cranial nerve palsy Know the cranial nerves that control the eyes Differentiate between the causes of various gaze abnormalities Know the oculomotor findings associated with neuroblastoma

August 2010 - Question 7 A 1-month-old boy develops gaseous abdominal distension while receiving narcotics for postoperative pain relief. He is referred to the PICU for treatment of respiratory distress characterized by tachypnea, nasal flaring, and grunting respirations. On physical examination, the boy's breath sounds are normal and symmetric, and his abdomen is enlarged but not firm. Radiography of the chest and upper abdomen shows gaseous distension of the stomach and large and small bowel. His lung fields are clear. The ICU fellow states that an older child would have less respiratory embarrassment from comparable distension, attributing the baby's exaggerated respiratory distress to developmental differences between infants and older patients. Of the following, the MOST likely development difference responsible for the infant's respiratory distress is A. different angle of origin of the diaphragm B. higher pulmonary vascular resistance C. less cartilaginous airways D. proportionately smaller alveolar surface area E. smaller airway diameter

A Although infants and older children have many developmental differences in respiratory anatomy and physiology, the angle of origin of the diaphragm makes infants especially vulnerable to respiratory embarrassment from abdominal distension. The diaphragm consists of a central aponeurosis and a peripheral muscular rim and essentially is a piston pulled down by a muscle (Figure). Figure: Attachment of the diaphragm The vector of force transmitted to displace the diaphragm downward is proportional to the cosine of the angle β in the Figure. Therefore, the force is greatest when β approaches 0° and least when β approaches 90°, as it does when an infant's abdomen becomes grossly distended. The same analysis suggests that a diaphragm flattened by alveolar air trapping can make little or no contribution to spontaneous inspiration. Infants also have smaller airway diameters at the vocal cords, subglottic region, trachea, and bronchi compared with older children. The dimensions are appropriate for the rate of airflow, which is proportionate to patient size. However, a small degree of mucosal swelling or a small accumulation of mucus in the airway may compromise the infant's airway severely. There is no suggestion of such compromise in the boy described in the vignette. The cords also are more distally displaced from the oropharynx in older children than in infants, which affects the approach to laryngoscopy and intubation. However, this is not an issue for this infant. The infant's major airways are less cartilaginous than those of the older child, which makes them susceptible to extrinsic compression. Tracheomalacia and bronchomalacia are states in which an extreme deficiency of cartilage makes the airway prone to collapse during forced expiration, compromising breathing during crying or respiratory distress. Such conditions are not an issue for this infant. Pulmonary vascular resistance is elevated at birth, but approaches normal adult levels by several weeks of age in healthy infants. Pulmonary vasoconstriction has no direct effect on respiratory effort. The pulmonary vasoconstriction of "black lung" persistent pulmonary hypertension of the newborn is associated with tachypnea and hypoxemia but is not generally associated with much mechanical respiratory embarrassment. Alveolar surface area generally is proportional to body surface area. It is not greatly altered during development. References: Takahashi E, Atsumi H. Age differences in thoracic form as indicated by thoracic index. Hum Biol. 1955;27:65-74 Fontan JJP. Mechanical dysfunction of the respiratory system. In: Fuhrman BP, Zimmerman JJ, eds. Pediatric Critical Care. 3rd ed. Philadelphia, Pa: Mosby Elsevier; 2006:543-556 American Board of Pediatrics Content Specification(s): Know the anatomic differences between the airways of infants and adolescents

Dec 2010 - Question 8 A 5-year-old boy has been admitted to the pediatric intensive care unit with severe hypoxemic lung injury secondary to H1N1 influenza and secondary methicillin-resistant Staphylococcus aureus pneumonia. He currently is being treated with vancomycin and oseltamivir. Despite this therapy, his course has deteriorated during the past 3 days, and he is now being transitioned to extracorporeal membrane oxygenation (ECMO). With this form of support, which of the following changes in vancomycin pharmacokinetics is MOST likely to occur? A. decreased clearance B. decreased protein binding C. decreased volume of distribution D. increased concentration due to membrane binding E. no pharmacokinetic changes related to ECMO

A Although the field remains ripe for further study, much data suggest that the pharmacokinetics of many medications are altered with the use of extracorporeal membrane oxygenation (ECMO). Vancomycin is one of the medications for which changes in pharmacokinetics have best been documented in children receiving ECMO support. Nearly all such studies have found that the implementation of ECMO increases the volume of distribution, decreases the clearance, and increases the half-life of vancomycin. Similar observations have been made for the effect of ECMO on the pharmacokinetics of other commonly used medications in the pediatric intensive care unit, including gentamicin, tobramycin, ranitidine, midazolam, theophylline, bumetanide, and morphine. Recent publications of small series have reviewed the pharmacokinetics of a number of medications during ECMO support, including oseltamivir, cefotaxime, caspofungin, voriconazole, furosemide, and amiodarone. The volume of distribution is the conceptual volume of fluid through which a drug disseminates after administration. It relates the total drug in the body to the concentration of the drug in the bloodstream: VD = Total Drug/Concentration in Blood Most studies of drug pharmacokinetics during ECMO published thus far have evaluated pharmacokinetic changes in term neonates undergoing venoarterial (VA) ECMO support. Again, these studies have generally documented an increase in the VD for a given drug during ECMO. The extra blood volume contained within the ECMO circuitry would appear to account for the increased volume of distribution with ECMO. In addition, the severity of the patient's underlying illness may contribute to increases in VD. For instance, a drug that is widely distributed in body water, such as vancomycin, will have an increase in VD due to the "third spacing" of fluids that accompanies a systemic inflammatory response. Potential loss of drug by circuit binding may be an additional mechanism by which VD is increased. The decreased clearance of many drugs noted in ECMO patients is believed to be related to alterations in the function of the 2 main organs responsible for drug clearance from the body, namely, the kidney and liver. Clearance is defined as the total volume of blood from which all drug is removed per unit of time. It is the sum of all the mechanisms of elimination throughout the body. The 2 main processes by which clearance occurs in the body are biotransformation and excretion. Excretion occurs primarily via the kidneys but may also occur via the lungs, gastrointestinal tract, or sweat and saliva. Renal mechanisms of clearance consist primarily of glomerular filtration and active secretion. Glomerular filtration is the primary form of renal elimination, and it is dependent on glomerular integrity, as well as the size, charge, degree of protein binding, and water solubility of the medication. Active secretion occurs mainly at the proximal convoluted tubule, where active transport systems secrete predominantly organic acids and organic bases. Cephalosporins, penicillins, both loop and thiazide diuretics, and nonsteroidal antiinflammatory agents are organic acids secreted at the proximal tubule; morphine and ranitidine are examples of organic bases. A significant percentage of patients supported with ECMO develop acute renal insufficiency, related both to the underlying disease process and potentially to ECMO support itself. Often, patients with renal dysfunction are supported with hemofiltration with or without dialysis while receiving ECMO support. As previously noted, most published studies have evaluated changes in drug pharmacokinetics associated with VA ECMO support. During VA ECMO, the kidneys are perfused with decreased pulsatility of blood flow. Some studies have suggested that glomerular filtration rate, and hence drug clearance, declines due to decreased pulsatility. However, decreased renal function has also been documented in patients supported with venovenous (VV) ECMO, during which end organs are perfused with pulsatile flow from the patient's native cardiac output. Thus, the decreased pulsatility seen during VA ECMO support cannot account for all the decrease in drug clearance so commonly demonstrated. Biotransformation, or the metabolism of medications, occurs primarily in the liver. In addition, redistribution into other body compartments may also contribute to the clearance of a medication from the blood. When the maintenance of a steady-state drug concentration is essential to ensure drug efficacy, the drug clearance is a particularly important clinical parameter. At steady state, it is the drug clearance that determines the amount of medication needed to maintain that concentration. Although overall clearance is decreased during ECMO for many medications, it is believed that many medications also adhere to the ECMO circuit and its membrane oxygenator, resulting in decreased serum levels. In fact, data suggest that the age of the circuit may influence this adherence. In one report, 36% of the vancomycin injected into a new, isolated ECMO circuit was lost within 4 hours compared with only 11% of that injected into a 5-day-old circuit previously used by a patient. Similar results were found for gentamicin, morphine, phenobarbital, and phenytoin using that model. Moreover, in a clinical study of neonates, significantly more midazolam was required for adequate sedation than anticipated within the first 24 hours of ECMO, presumably because of the expanded circulating volume and sequestration by the circuit. However, because of subsequent circuit saturation, the required maintenance doses were reduced. Fentanyl is notorious for being sequestered by the ECMO circuit, with the majority being irreversibly bound to the membrane oxygenator. Potential changes in protein binding have not been thoroughly evaluated in patients supported with ECMO. It should be again noted that most studies regarding pharmacokinetic issues in ECMO patients have examined term neonates supported with VA ECMO. Potential pharmacokinetic changes in older patients supported with VV ECMO are less well documented. In addition, studies to date have almost exclusively documented pharmacokinetic changes in patients supported with a silicone membrane oxygenator and a roller-head pump in the ECMO circuit. The potential effect of newer circuits with smaller priming volumes, newer-generation centrifugal pumps, and polymethylpentene hollow fiber oxygenators on drug pharmacokinetics is currently a fruitful field for investigation. References: Ahsman MJ, Wildschut ED, Tibboel D, Mathot RA. Pharmacokinetics of cefotaxime and desacetylcefotaxime in infants during extracorporeal membrane oxygenation. Antimicrob Agents Chemother. 2010;54:1734-1741. Abstract accessed August 2010 at: http://www.ncbi.nlm.nih.gov/pubmed/20176908. Buck ML. Pharmacokinetic changes during extracorporeal membrane oxygenation: implications for drug therapy of neonates. Clin Pharmacokinet. 2003;42:403-417. Abstract accessed August 2010 at: http://www.ncbi.nlm.nih.gov/pubmed/12739981. Dagan O, Klein J, Gruenwald C, Bohn D, Barker G, Koren G. Preliminary studies of the effects of extracorporeal membrane oxygenator on the disposition of common pediatric drugs. Ther Drug Monit. 1993;15:263-266. Abstract accessed August 2010 at: http://www.ncbi.nlm.nih.gov/pubmed/8236359. Mulla H, Lawson G, Peek GJ, Firmin RK, Upton DR. Plasma concentrations of midazolam in neonates receiving extracorporeal membrane oxygenation. ASAIO J. 2003;49:41-47. Abstract accessed August 2010 at: http://www.ncbi.nlm.nih.gov/pubmed/12558306. Mulla H, McCormack P, Lawson G, Firmin RK, Upton DR. Pharmacokinetics of midazolam in neonates undergoing extracorporeal membrane oxygenation. Anesthesiology. 2003;99:275-282. DOI: 10.1097/00000542-200308000-00008. Abstract accessed August 2010 at: http://www.ncbi.nlm.nih.gov/pubmed/12883399. Mulla H, Pooboni S. Population pharmacokinetics of vancomycin in patients receiving extracorporeal membrane oxygenation. Br J Clin Pharmacol. 2005;60:265-275. DOI: 10.1111/j.1365-2125.2005.02432.x. Primak L, Blumer JL. Principles of drug distribution in the critically ill child. In: Fuhrman BP, Zimmerman JJ, eds. Pediatric Critical Care. 3rd ed. Philadelphia, PA: Mosby-Elsevier; 2006:1639-1658. Spriet I, Annaert P, Meersseman P, et al. Pharmacokinetics of caspofungin and voriconazole in critically ill patients during extracorporeal membrane oxygenation. J Antimicrob Chemother. 2009;63:767-770. DOI: 10.1093/jac/dkp026. Accessed August 2010 at: http://jac.oxfordjournals.org/cgi/content/full/63/4/767? view=long&pmid=19218271. van der Vorst MM, den Hartigh J, Wildschut E, Tibboel D, Burggraaf J. An exploratory study with an adaptive continuous intravenous furosemide regimen in neonates treated with extracorporeal membrane oxygenation. Crit Care. 2007;11:R111. DOI: 10.1186/cc6146. Accessed August 2010 at: http://www.ncbi.nlm.nih.gov/pmc/articles/PMC2556764/? tool=pubmed. Wildschut ED, de Hoog M, Ahsman MJ, Tibboel D, Osterhaus AD, Fraaij PL. Plasma concentrations of oseltamivir and oseltamivir carboxylate in critically ill children on extracorporeal membrane oxygenation support. PLoS One. 2010;5:e10938. DOI: 10.1371/journal.pone.0010938. Accessed August 2010 at: http://www.ncbi.nlm.nih.gov/pmc/articles/PMC2880602/?tool=pubmed. Zuppa AF, Barrett JS. Pharmacology. In: Nichols DG, ed. Roger's Textbook of Pediatric Intensive Care. 4th ed. Philadelphia, PA: Lippincott Williams & Wilkins; 2008:266-268. American Board of Pediatrics Content Specification(s): Know the common, clinically relevant pharmacokinetic alterations with extracorporeal support Understand the components and the concepts of drug clearance

April 2010 - Question 1 A 3-year-old boy who has a history of prematurity and subglottic stenosis is admitted to the PICU after placement of a tracheostomy tube in the operating room. He has had multiple previous intubations for upper respiratory tract obstruction. During this hospital admission, he again required tracheal intubation for upper airway obstruction, and a tracheostomy was performed on the fourth day after intubation. Three days after the procedure, he develops increased tracheal secretions and a fever. A chest radiograph reveals a focal consolidation of his left lower lobe. Of the following, the action that would have been MOST likely to reduce the occurrence of ventilator-associated pneumonia in this patient, based on recommendations from the Centers for Disease Control and Prevention, is A. elevation of the head of the bed to 30 to 45 degrees B. use of intermittent versus continuous feedings C. use of nasotracheal intubation instead of orotracheal intubation D. use of sucralfate instead of histamine-2 antagonist E. ventilator circuit change every 24 hours

A The child described in the vignette has ventilator-associated pneumonia (VAP), defined by the Centers for Disease Control and Prevention (CDC) as pneumonia in a patient who has a device to assist or control ventilation through a tracheostomy or endotracheal tube for at least 48 hours before the onset of infection. VAP is associated with increased length of stay in the ICU, increased hospital length of stay, and increased ventilator-days. Whether VAP also is associated with increased mortality rates is debatable. The CDC has developed guidelines for prevention of VAP. As of August 2009, they recommend: 1. Educate and involve staff in infection prevention 2. Conduct surveillance for bacterial pneumonia in high-risk patients to determine trends and identify outbreaks 3. Avoid routine changing of breathing circuits unless visibly soiled 4. Drain and discard condensate in tubing away from patient 5. Use sterile water for humidification 6. Use sterile water for nebulization 7. When not medically contraindicated, use noninvasive positive-pressure ventilation (NIPPV) by face or nasal mask instead of endotracheal intubation in patients who have respiratory failure and do not need immediate intubation 8. Use NIPPV as part of the weaning process to shorten the duration of endotracheal intubation 9. Perform orotracheal instead of nasotracheal intubation unless contraindicated by the patient's condition 10. Clear secretions above the tube cuff prior to deflating the cuff of an endotracheal tube when preparing for tube removal or before changing the position of the tube 11. Elevate the angle of the head of the bed to 30 to 45 degrees in patients receiving mechanical ventilation and enteral feedings The CDC does not recommend the use of sucralfate rather than histamine-2 receptor antagonists for prevention of stress bleeding in the mechanically ventilated patient, even though it has been shown that antacids and histamine-2 receptor antagonists are associated with gastric bacterial overgrowth and increased risk of pneumonia. Sucralfate has little effect on gastric pH and has been considered a substitute, but recent studies have not shown a decreased risk of VAP in patients receiving sucralfate, and another study has suggested a possible increased risk of VAP with sucralfate use. The CDC has no recommendations regarding the superiority of intermittent enteral feedings in preventing VAP compared with a continuous feeding regimen. Even though gastric pH has been shown to be decreased with intermittent feedings, the incidence of VAP and the rates of oropharyngeal and tracheal colonization are similar with both feeding strategies. The American Thoracic Society (ATS) has issued additional recommendations for preventing VAP, such as maintaining the endotracheal cuff pressure, when applicable, at greater than 20 cm H2O to prevent leakage of bacterial organisms around the cuff into the lower respiratory tract. This recommendation should be considered with caution in children because of the risk of causing tracheal injury and subglottic stenosis when cuff pressures are maintained above tissue perfusion pressures. Other interventions recommended by the ATS include attention to oral care and hygiene in an attempt to modulate oropharyngeal colonization and daily interruption or lightening of sedation plus avoidance of neuromuscular blockade, when possible, to avoid depression of the cough protective mechanism. The CDC have classified nosocomial pneumonia as early- and late-onset. The distinction is based on the organisms responsible for early-onset pneumonia (occurring within 4 days of hospitalization) typically differing from those causing late-onset pneumonia. Moraxella catarrhalis, Haemophilus influenzae, and Streptococcus pneumoniae are frequent pathogens in early-onset pneumonia. Late-onset pneumonia more often is caused by Staphylococcus aureus, including methicillin-resistant S aureus, or gram-negative organisms (Acinetobacter, Enterobacter, Pseudomonas). The importance of these organisms lies in their propensity to be multidrug-resistant and, thus, more difficult to treat, translating into worse outcomes compared with pneumonia caused by susceptible pathogens. Other causative agents of late-onset pneumonia include yeasts, fungi, Pneumocystis jirovecii, and Legionella. The definition of clinical pneumonia published by the CDC includes both radiographic and clinical/laboratory criteria (Tables 1 through 4). Table 1: Radiographic Criteria for Pneumonia In patients with no history of heart or lung pathology, only one radiograph is necessary Two or more serial chest radiographs in all other patients with the appearance of one of the following: New or progressive and persistent infiltrate Consolidation Cavitation Pneumatocele in infants younger than 1 year of age Table 2: Clinical Criteria for Pneumonia (Infants <1 Year of Age) Worsening gas exchange AND at least three of the following: 1. Temperature instability with no other recognized cause 2. Change in white blood cell count a. Leukopenia (<4.0x103/mcL (4.0x109/L) b. Leukocytosis (>15.0x103/mcL [15.0x109/L]) and left shift (>10% bands) 3. Changes in sputum a. New onset of purulent sputum b. Change in character of sputum c. Increased respiratory secretions d. Increased suctioning requirements 4. Change in respiratory pattern a. Apnea b. Tachypnea c. Nasal flaring with retraction of chest wall d. Grunting 5. Change in respiratory auscultation a. Wheezing b. Rales c. Rhonchi 6. Cough 7. Changes in heart rate a. Bradycardia (<100 beats/min) b. Tachycardia (>170 beats/min) Table 3: Clinical Criteria for Pneumonia (Children >1 or <12 Years Old) > At least three of the following: 1. Temperature changes with no other recognized cause a. Fever (>38.4°C) b. Hypothermia (<37°C) 2. Change in white blood cell count a. Leukopenia (<4.0x103/mcL [4.0x109/L]) b. Leukocytosis (>15.0x103/mcL [15.0x109/L]) 3. Changes in sputum a. New onset of purulent sputum b. Change in character of sputum c. Increased respiratory secretions d. Increased suctioning requirements 4. Change in respiratory pattern a. New onset or worsening cough b. Dyspnea c. Apnea d. Tachypnea 5. Change in respiratory auscultation a. Rales b. Bronchial breath sounds 6. Worsening gas exchange a. Oxygen desaturations b. Increased oxygen requirements c. Increased ventilation demand Table 4: Clinical Criteria for Pneumonia (Any Patient >13 Years Old) At least one of the following: 1. Fever (>38.0°C) with no other recognized cause 2. Change in white blood cell count a. Leukopenia (<4.0x103/mcL [4.0x109/L]) b. Leukocytosis (>12.0x103/mcL [12.0x109/L]) AND at least two of the following: 1. Change in sputum a. New onset of purulent sputum b. Change in character of sputum c. Increased respiratory secretions d. Increased suctioning requirements 2. Change in respiratory pattern a. New onset or worsening cough b. Dyspnea c. Tachypnea 3. Change in respiratory auscultation a. Rales b. Bronchial breath sounds 4. Worsening gas exchange a. Oxygen desaturations b. Increased oxygen requirements c. Increased ventilation demand The distal airways become colonized a few hours after intubation. Pneumonia develops when pathogens invade and remain in the lower respiratory tract and lung tissue. Potential sources of the invading pathogen are: 1. Endotracheal tube carriage of microorganisms in the oropharynx into the lower airway during the intubation process 2. Presence of biofilm on the endotracheal tube as bacteria form a glycocalyx that acts as a protective environment for the microorganism 3. Dislodgment of bacterial aggregates with suctioning, manipulation of the tube, or ventilation flow into the lower respiratory tract 4. Leakage of secretions containing bacteria around the endotracheal tube 5. Reservoirs for nosocomial pathogens in the stomach and sinuses 6. Health-care devices, possible inoculation of ventilator circuit condensate or inhalation of contaminated aerosols, environment (water, equipment, air), cross-contamination via health-care personnel or other patients Risk factors for nosocomial pneumonia have been identified and are grouped by the CDC into four general categories (Table 5). Table 5: Risk Factors for Nosocomial Pneumonia 1) Factors that enhance colonization of the oropharynx and/or stomach by microorganisms: - Administration of antimicrobial agents - Admission to the ICU - Presence of underlying chronic lung disease 2) Conditions favoring aspiration into the respiratory tract or reflux from the gastrointestinal tract: - Endotracheal intubation (initial or repeat) - Supine position - Presence of nasogastric tube - Coma - Surgery to the head, neck, thorax, or upper abdomen - Immobilization 3) Conditions requiring prolonged use of mechanical ventilation, with potential exposure to contaminated devices or hands, especially of health-care personnel 4) Other host factors: - Extreme age - Malnutrition - Immunosuppression References: Centers for Disease Control and Prevention. Criteria for Defining Nosocomial Pneumonia. 2005. Accessed August 2009 at: http://www.cdc.gov/ncidod/hip/NNIS/members/pneumonia/Final/PneumoCriteriaV1.pdf Curley MA, Schwalenstocker E, Deshpande JK, et al. Tailoring the Institute for Health Care Improvement 100,000 Lives Campaign to pediatric settings: the example of ventilatorassociated pneumonia. Pediatr Clin North Am. 2006;531231-1251. Abstract accessed October 2009 at: http://www.ncbi.nlm.nih.gov/pubmed/17126693 Leong JR, Huang DT. Ventilator-associated pneumonia. Surg Clin North Am. 2006;86:1409- 1429. Abstract accessed October 2009 at: http://www.ncbi.nlm.nih.gov/pubmed/17116455 Markowicz P, Wolff M, Djedaini K, et al. Multicenter prospective study of ventilator associated pneumonia during acute respiratory distress syndrome. Incidence, prognosis, and risk factors. ARDS Study Group. Am J Respir Crit Care Med. 2000; 161:1942-1948. Accessed October 2009 at: http://ajrccm.atsjournals.org/cgi/content/full/161/6/1942 Rello J, Díaz E, Rodríquez A. Etiology of ventilator-associated pneumonia. Clin Chest Med. 2005;26:87-95. Abstract accessed October 2009 at: http://www.ncbi.nlm.nih.gov/pubmed/15802170 American Board of Pediatrics Content Specification(s): Know the pathogenesis of ventilator-associated pneumonia in a patient receiving mechanical ventilation Know how to diagnose ventilator-associated pneumonia in a patient receiving mechanical ventilation Know the strategies to decrease the incidence of ventilator-associated pneumonia in a patient receiving mechanical ventilation

Jan 2010 - Question 3 A 6-year-old boy is transferred to the PICU from an outlying hospital with acute respiratory distress due to aspiration pneumonia. The child is orotracheally intubated and mechanically ventilated with volume-control mode of ventilation. Ventilatory settings are: Tidal volume, 8 mL/kg Positive end-expiratory pressure (PEEP), 12 cm H2O Rate, 24 breaths/min Inspiratory time, 1.0 second Pressure support, 12 cm H2O above PEEP The peak inspiratory pressures vary from 24 to 26 cm H2O, and the mean airway pressure is currently 16 cm H2O. Spo2 is 92% on 0.55 Fio2. On physical examination, the child's heart rate is 176 beats/min, blood pressure is 75/43 mm Hg, and temperature is 37.5°C. Crackles are audible bilaterally, pulses are weak, and the capillary refill time is 4 to 5 seconds in the lower extremities. You evaluate the static pressure-volume (PV) curve on no end-expiratory pressure to gain a better understanding of the patient's respiratory physiology (Figure 1). Figure 1: Pressure-volume curve for the patient Of the following, the MOST appropriate next step is to A. decrease the PEEP B. increase the FiO2 C. place the patient in the prone position D. switch to high-frequency oscillatory ventilation with a mean airway pressure of 18 cm H2O E. switch to pressure-control mode of ventilation with a mean airway pressure of 17 cm H2O

A The child described in the vignette is demonstrating signs of decreased cardiac output (tachycardia, weak pulses, and prolonged capillary refill time) while receiving positivepressure ventilation with a mean airway pressure of 16 cm H2O. The PV loop reveals a lower inflection point (LIP) around 7 cm H2O and the PEEP is set at 12 cm H2O. Although the PEEP is set above the LIP and is on the compliant segment of the PV curve, the child is experiencing hemodynamic instability. Appropriate management includes the administration of intravenous fluid boluses and decreasing the mean airway pressure, which can be achieved by decreasing PEEP. Switching to high-frequency oscillatory ventilation or pressure-control ventilation at a higher mean airway pressure would exacerbate the hemodynamic instability. Increasing the FiO2 would not improve cardiac output and could be detrimental due to oxygen toxicity. Although the prone position has been shown to improve oxygenation in many cases of acute respiratory distress syndrome (ARDS), it should be used in association with adequate ventilator strategy and would be unlikely to reverse this child's hemodynamic instability. The primary indication for initiating mechanical ventilation is respiratory failure manifested as inadequate oxygenation, inadequate ventilation, or both. An intrapulmonary shunt greater than 15% requires greater intervention than simply increasing the FiO2. Such intervention involves recruitment and re-expansion of collapsed alveoli, which can be accomplished with the PEEP. PEEP restores functional residual capacity to a level greater than the closing volume, thus preventing alveolar collapse. Determining the optimum level of PEEP has been a subject of considerable investigation because application of either insufficient or excessive PEEP has deleterious effects. Low PEEP contributes to ventilator-induced lung injury (VILI) due to shear stress injury from repetitive opening and closing of alveolar units. Excessive PEEP causes both detrimental respiratory and hemodynamic effects. In heterogeneous lung disease, such as acute lung injury (ALI)/ARDS, PEEP may cause hyperinflation of healthier, more compliant lung segments, driving pulmonary blood flow away from these segments and toward diseased regions. This increases intrapulmonary shunt as well as alveolar dead space. Cardiac output also may be affected adversely in some patients by the institution of PEEP, especially among patients who have hypovolemia or decreased vascular tone. The increase in intrathoracic pressure may decrease right ventricular filling and, consequently, right ventricular stroke volume by reducing the pressure gradient for venous return. As a result, left ventricular filling also is impaired and the patient shows signs of decreased cardiac output. In cases of lung hyperinflation due to increased mean airway pressure, right ventricular compliance is decreased, causing a shift in the interventricular septum to the left that further impairs cardiac output. In this situation, two therapeutic approaches may be employed, preferably simultaneously, to circumvent the adverse hemodynamic effects of positive-pressure ventilation. The first involves decreasing right atrial pressure by reducing intrathoracic pressure (decreasing PEEP or tidal volume). The second strategy is to increase venous pressure by administering intravenous fluids. It should be noted, however, that PEEP may have a salutary effect on patients who have left ventricular dysfunction because it decreases afterload. The goal of application of PEEP in ALI/ARDS is to prevent significant derecruitment of injured lung segments without causing overdistention of healthier regions. One method of assessing recruitment and overdistention is the use of static PV curves. Different levels of PEEP yield different PV curves. The PV curve in ALI/ARDS traditionally has been calculated with zero PEEP (ZEEP) and results in a classic sigmoidal shape in which the inspiratory limb consists of three segments of different compliance (Figure 2). Figure 2: Analysis of the pressure-volume curve The first segment at a low volume, starting at ZEEP, is relatively flat, indicating low compliance. Many lung units are collapsed, consolidated, or fluid-filled and require a higher insufflation pressure to open. The presence of a LIP denotes the critical pressure above which compliance improves. In the second, more linear segment, many lung units are open and the slope is steeper, denoting improved compliance. The third segment occurs above the upper inflection point (UIP), the point above which alveolar overdistention occurs. The LIP can be measured by intersecting the lines from the first and second portions of the inspiratory limb on a PV curve. The UIP can be derived by intersecting the lines from the second and third portions of the curve (Figure 2). Traditional teaching has been to set PEEP slightly above the LIP to prevent the cyclic opening and closing of lung units and below the UIP to prevent the harmful effects of alveolar overdistention. More recent hypotheses have suggested that alveolar recruitment is continuous throughout the linear, compliant portion of the PV curve. It is also believed that the UIP does not represent overdistention of alveoli, but rather recruitment of the most dorsal lung regions, with lower compliance at the end of inspiration. Consistent with the continuous recruitment model, the LIP may not be a precise indicator for determining the optimum level of PEEP. When using the PV curve for ventilator settings, gas exchange parameters (improved oxygenation with the lowest Fio2) also should be considered. In addition, ventilator settings should be such that there is little effect on systemic hemodynamics, which could compromise delivery of oxygen to the tissues further. References: Barbas CS, De Matos GF, Okamoto V, Borges JB, Amato MB, de Carvalho CR. Lung recruitment maneuvers in acute respiratory distress syndrome. Respir Care Clin N Am. 2003;9:401-418. Abstract accessed October 2009 at: http://www.ncbi.nlm.nih.gov/pubmed/14984063 Curley MA, Hibberd PL, Fineman LD, et al. Effect of prone positioning on clinical outcomes in children with acute lung injury. JAMA. 2005;294:229-237. Accessed November 2009 at: http://jama.ama-assn.org/cgi/content/full/294/2/229 Grinnan DC, Truwit JD. Clinical review: respiratory mechanics in spontaneous and assisted ventilation. Crit Care. 2005;9:472-484. DOI: 10.1186/cc3516. Accessed October 2009 at: http://ccforum.com/content/9/5/472 MacIntyre NR. Is there a best way to set positive expiratory-end pressure for mechanical ventilator support in acute lung injury? Clin Chest Med. 2008;29:233-239. Abstract accessed October 2009 at: http://www.ncbi.nlm.nih.gov/pubmed/18440433 American Board of Pediatrics Content Specification(s): Understand the appropriate application of CPAP/PEEP Understand the deleterious effects of CPAP/PEEP Understand the effects of lung volume and breathing on right ventricular afterload Understand the effects of lung volume and breathing on left ventricular afterload

Feb 2010 - Question 7 A 6-month-old infant who has a history of tetralogy of Fallot (TOF) is admitted to the PICU after undergoing repair. At 2 months of age, she had a Blalock-Taussig shunt placed without complications. Her central lines were removed 2 days after that surgery, and she was discharged 3 days later. Her immediate postoperative laboratory tests today reveal a platelet count of 80x103/mcL (80x109/L), with no signs of excessive bleeding. The preoperative platelet count was 320x103?mcL (320x109/L). On postoperative day 1, she has a temperature of 38.4°C. You obtain blood and urine specimens for culturing and begin broad-spectrum antibiotics. You transfuse platelet concentrates on days 2 through 5 (Figure), but her platelet count continues to fall. Figure On postoperative day 3, you note swelling and poor perfusion of her left lower extremity. A Doppler study reveals thrombosis of her common femoral vein. Of the following, the MOST appropriate component of your initial treatment of this patient's condition is A. argatroban B. corticosteroids C. low-molecular weight heparin D. platelet transfusion E. splenectomy

A The infant described in the vignette has thrombocytopenia, with a progressively decreasing platelet count. In the trauma or postoperative patient, a decrease in platelet count may be due to excessive bleeding or hemodilution from the administration of crystalloids or blood products. Hypothermia is another important cause that necessitates rewarming the patient, not replacing platelets. Thrombocytopenia also occurs in 50% to 75% of patients who have fungemia or bacteremia. Septic shock and disseminated intravascular coagulation are responsible for a large number of thrombocytopenia cases. Thrombocytopenia is expected with the use of extracorporeal membrane oxygenation, with platelets altered and sequestered in the lung, liver, and spleen. Also in the differential diagnosis is drug-induced thrombocytopenia, most notably heparin-induced thrombocytopenia (HIT). It is safe to assume that the infant was exposed to heparin as flushes for an arterial line or even peripheral intravenous flushes during shunt placement. During cardiopulmonary bypass, she was re-exposed to heparin and developed thrombocytopenia immediately postoperatively, which worsened with the transfusion of platelets. She also had a thrombotic event on postoperative day 3. These findings suggest HIT, and treatment should be instituted immediately with argatroban, a direct thrombin inhibitor (DTI). Platelet infusions are contraindicated in cases of HIT due to the immunologic nature of the disease. HIT is an immune-mediated reaction caused by immunoglobulin G antibodies against heparin-platelet 4 (PF4) complexes. PF4 is a chemokine stored in platelet alpha granules that binds to the surface of endothelial cells and heparin. Heparin alone is not immunogenic, but when combined with PF4, it becomes highly immunogenic. A conformational change occurs with such binding and stimulates the production of specific antibodies. Activation of platelets leads to release of procoagulant substances (adenosine diphosphate, histamine, and serotonin). Activation of monocytes and endothelial cells favors the production of thrombin. There are two types of HIT. HIT type I (also called heparin-associated thrombocytopenia) is more frequent, occurring in 10% to 30% of patients receiving heparin. It is a nonimmune-mediated reaction and is not associated with hemorrhagic or thrombotic complications. Resolution of thrombocytopenia occurs despite continuation of heparin. Type II HIT occurs in 1% to 3% of patients who receive unfractionated heparin and fewer than 1% of patients receiving low-molecular weight heparin LMWH. Although thrombocytopenia typically is associated with bleeding complications, HIT type II paradoxically is associated with thrombotic complications. Venous thrombotic complications are four times more common than arterial thrombotic complications. The postoperative patient is at higher risk for HIT type II, especially after cardiac, vascular, or orthopedic surgery. There is a high risk of loss of limb or life with HIT type II. HIT type II typically occurs 5 to 10 days after exposure to heparin. Of patients who develop HIT, 25% have a rapid onset, developing manifestations within 24 hours of re-exposure. Antibodies against the heparin-PF4 complex also develop in patients who do not manifest thrombocytopenia and thrombotic complications. Only a subset of patients who have HIT antibodies develop HIT; most patients expressing HIT antibodies in the circulation do not have any clinical consequences. Therefore, the presence of HIT antibodies alone is not significant. Criteria for the diagnosis of HIT are: 1. Thrombocytopenia, defined as a platelet count of less than 150x103/mcL (150x109/L) or a 50% decrease from baseline 2. No other identifiable cause of thrombocytopenia 3. Current or recent use of heparin 4. Confirmation by a HIT assay 5. A new thrombotic or thromboembolic event Of note, HIT may present 40 days after exposure to heparin. The most widely used assays to diagnose HIT are the PF4-heparin enzyme-linked immunosorbent assay and the C-serotonin release assay. The serotonin release assay has a higher sensitivity (60% to 80%) but is more costly. Treatment of HIT type II involves immediate discontinuation of any possible sources of heparin. Because HIT type II is an immunologic reaction, even minute amounts of heparin, such as in heparin flushes used to maintain intravenous lines or heparin-coated catheters, may trigger complications. It is imperative to initiate anticoagulation because 30% to 50% of patients diagnosed with HIT who do not have thrombotic complications at the time of diagnosis and are not given anticoagulation therapy develop thromboembolic events. Delay in instituting therapy while awaiting laboratory confirmation is associated with a significantly higher risk of thrombotic complications compared with patients in whom a DTI is initiated promptly (6.1% versus 0.6%). Inhibition of thrombin generation is the focus of anticoagulation therapy in HIT. DTIs currently approved by the United States Food and Drug Administration include lepirudin, argatroban, and bivalirudin. The first two are primarily used for HIT. These drugs inhibit free and clot-bound thrombin. Lepirudin is excreted renally and argatroban is excreted hepatically. Argatroban has a rapid onset of action, and its binding to thrombin is quickly reversible upon cessation of administration. The half-life of argatroban is 40 minutes and that of lepirudin is 80 minutes. Both drugs are administered as continuous infusions. The activated prothrombin time is used as a therapeutic endpoint (1.5 to 3 times baseline). In cases of HIT, warfarin should not be initiated until the platelet count is higher than 150x103/mcL (150x109/L). Warfarin causes a decrease in protein S and protein C concentrations and may precipitate thrombotic events while the patient is thrombocytopenic. Warfarin and DTI therapy should overlap for 4 to 5 days. DTIs prolong the international normalized ratio (INR), even if warfarin effects still are subtherapeutic. Discontinuing DTIs prematurely may precipitate recurrence or progression of thrombosis. Two INRs within the target range obtained 24 hours apart in addition to normal platelet counts are necessary for discontinuation of the DTIs. Danaparoid, a heparinoid with antifactor Xa activity and no in vivo cross-reactivity with HIT antibodies, is used in the therapy of HIT in Europe and Canada. It has been unavailable for use in the United States since 2002. Of note, HIT is markedly less common in the pediatric than the adult population. Corticosteroids and intravenous immune globulin should be considered in patients who have presumed idiopathic thrombocytopenic purpura (ITP). Splenectomy is used to treat life-threatening complications of ITP or select cases associated with hypersplenism. Although infection could be a cause of thrombocytopenia for this infant, the lack of improvement after administration of broad-spectrum antibiotics, no other described signs of sepsis, and failure of the platelet count to increase with the platelet transfusions argue against this possibility. Use of low-molecular weight heparin is not recommended for the treatment of HIT because it also can induce HIT. Platelets are formed from cytoplasmic fragmentation of megakaryocytes in the bone marrow. The average lifespan of platelets is 9 to 10 days. Thrombocytopenia is defined as a platelet count of less than 150x103/mcL (150x109/L). Spontaneous bleeding occurs with counts less than 10x103/mcL (10x109/L), and studies have shown that vascular integrity is compromised with counts lower than 7x103/mcL (7x109/L) Although platelets are involved in every step of the hemostatic process, their major function is to maintain primary hemostasis following reflex vasoconstriction after vascular injury. An aggregate of platelets then forms at the site and leads to the third phase of hemostasis, with the formation of a more stable platelet-fibrin plug. Thrombocytopenia may be due to decreased production, increased consumption, peripheral destruction, or splenic pooling of platelets. In cases of increased platelet destruction, the mean platelet volume is elevated, signifying increased release of younger, bigger platelets from the bone marrow. Destruction of platelets may be caused by immune or nonimmune mechanisms (Table). Table: Thrombocytopenia Due to Platelet Destruction Immune Mechanisms Idiopathic thrombocytopenic purpura Autoimmune disorders Neonatal thrombocytopenia Drug-induced thrombocytopenia Sedatives Anticonvulsants Penicillin and derivatives Heparin Nonsteroidal anti-inflammatory drugs Procainamide Nonimmune Mechanisms Hypersplenism Kasabach-Merritt Syndrome TORCH (Toxoplasma, other viruses, rubella, cytomegalovirus, herpes simplex) infections Disseminated intravascular coagulation Hypothermia Hemolytic-uremic syndrome Thrombotic thrombocytopenic purpura Massive transfusion Decreased production of platelets in the bone marrow is a less common cause of thrombocytopenia in the pediatric population. Marrow infiltration with tumors or the marrow-depressant effects of antineoplastic agents may be responsible for decreased platelet production. Hypersplenism causes thrombocytopenia primarily by increased pooling of platelets in the spleen. A normal spleen may hold up to 30% of the total platelet mass; a massively enlarged spleen can hold up to 90% of circulating platelets. The thrombocytopenia seen with hypersplenism is usually in the range of 50 to 150x103/mcL (50 to 150x109/L). More severe thrombocytopenia suggests a different cause. The neutrophil count also may be reduced due to increase in the marginated granulocyte pool in the spleen. Bone marrow examination invariably yields normal results. The most important issue is to determine the cause of the splenomegaly. Thrombocytopenia also may be caused by infection. Almost 100% of patients who have septic shock or disseminated intravascular coagulation (DIC) develop thrombocytopenia. Hemophagocytosis may be one mechanism responsible for sepsis-related thrombocytopenia. Other presumed mechanisms in the absence of DIC include direct activation of platelets by endogenous mediators of inflammation or microbial products. Platelet-reactive autoantibodies have been implicated in thrombocytopenia seen in subacute bacterial endocarditis and infection with varicella zoster. Infection with human immunodeficiency virus (HIV) may lead to chronic platelet destruction with platelet autoantibodies. Antibodies against platelet-adsorbed microbial antigens are seen in malaria. Another mechanism of platelet destruction in malaria is hypersplenism. Hypersplenism associated with infection and thrombocytopenia may occur with disseminated Mycobacterium avium infection in patients who have HIV infection or chronic active hepatitis. Careful investigation for infection as a cause of thrombocytopenia in the hospitalized patient is warranted because resolution of infection also results in resolution of thrombocytopenia. Reviewing the past medical history and family history is imperative when evaluating a patient who has thrombocytopenia. Associated findings may suggest the diagnosis. For example, the presence of limb abnormalities in addition to pancytopenia suggests Fanconi anemia or thrombocytopenia absent radii. Hemorrhagic diarrhea and acute renal failure associated with thrombocytopenia point to hemolytic-uremic syndrome. An otherwise healthy child who presents with thrombocytopenia more likely has ITP. Petechiae, purpura, or more severe bleeding with injury suggest thrombocytopenia in the outpatient setting. Patients may present with epistaxis, menorrhagia, or gingival bleeding. Spontaneous bleeding into a joint suggests coagulopathy due to factor deficiency. The most common cause of thrombocytopenia in childhood is ITP, with the autoimmune disorder acute ITP seen most commonly in children between the ages of 2 and 6 years in the winter months. The presentation typically follows a viral illness in a previous healthy child. ITP is usually a self-limited disorder, with complete resolution occurring within 6 to 12 months in most pediatric patients. The most serious complication is intracranial hemorrhage, which occurs in fewer than 1% of patients. ITP is a diagnosis of exclusion. Therapy is controversial. Expectant observation is used frequently because of the self-resolving nature of the disease. Drug therapy with corticosteroids, intravenous immunoglobulin, or anti-D immunoglobulin may shorten but does not alter the clinical disease course. Platelet transfusions do not raise the count for a significant period of time but may be warranted in emergency situations. Splenectomy is considered in life-threatening situations. References: Kaplan RN, Bussel JB. Differential diagnosis and management of thrombocytopenia in childhood. Pediatr Clin North Am. 2004;51:110911140. Abstract accessed October 2009 at: http://www.ncbi.nlm.nih.gov/pubmed/15275991 Levy JH, Hursting MJ. Heparin-induced thrombocytopenia, a prothrombotic disease. Hematol Oncol Clin North Am. 2007;21:65-88. Abstract accessed October 2009 at: http://www.ncbi.nlm.nih.gov/pubmed/17258119 Menajovsky B. Heparin-induced thrombocytopenia: clinical manifestations and management strategies. Am J Med. 2005;118:21S-30S. Abstract accessed October 2009 at: http://www.ncbi.nlm.nih.gov/pubmed/16125511 Mullen MP, Wessel DL, Thomas KC, et a. The incidence and implications of anti-heparinplatelet factor 4 antibody formation in a pediatric cardiac surgical population. Anesth Analg. 2008;107:371-378. Accessed November 2009 at : http://www.anesthesiaanalgesia. org/cgi/content/full/107/2/371 Shantsila E, Lip GY, Chong BH. Heparin-induced thrombocytopenia. A contemporary clinical approach to diagnosis and management. Chest. 2009;135:1651-1664. Abstract accessed October 2009 at: http://www.ncbi.nlm.nih.gov/pubmed/19497901 American Board of Pediatrics Content Specification(s): Recognize hypersplenism as a cause of thrombocytopenia Recognize infection as an etiology of thrombocytopenia Know the causes of thrombocytopenia Know the complications of thrombocytopenia Distinguish among the causes of thrombocytopenia Understand how to diagnose and treat heparin‑induced thrombocytopenia

Feb 2010 - Question 2 You are treating a 15-month-old girl for sepsis. She has mild purpura fulminans and is supported with dopamine at 8 mcg/kg per minute and epinephrine at 0.07 mcg/kg per minute. She is receiving mechanical ventilation with a tidal volume of 6 mL/kg, a rate of 25, a positive endexpiratory pressure of 7 cm H2O, and an FiO2 of 0.6. Her most recent arterial blood gas shows a pH of 7.25, PCO2 of 50 mm Hg, PaO2 of 63 mm Hg, and base excess of -6 mEq/L. Other laboratory studies demonstrate: Sodium, 135 mEq/L (135 mmol/L) Potassium, 2.9 mEq/L (2.9 mmol/L) Chloride, 98 mEq /L (98 mmol/L) Bicarbonate, 18 mEq /L (18 mmol/L) Blood urea nitrogen, 28 mg/dL (10.0 mmol/L) Creatinine, 1.1 mg/dL (97.2 mcmol/L) Glucose, 123 mg/dL (6.8 mmol/L) White blood cell count, 17.3x103/mcL (17.3x109/L) Hemoglobin, 8.1 g/dL (81 g/L) Hematocrit, 24% (0.24) Platelet count, 97x103/mcL (97x109L) On physical examination, the girl's temperature is 37.8°C, heart rate is 153 beats/min, respiratory rate is 27 breaths/min, and blood pressure is 88/61 mm Hg. Her distal extremities are cool and capillary refill is delayed. Echocardiography suggests mildly decreased contractility globally but good intraventricular volumes. Of the following, the intervention MOST likely to increase microvascular blood flow to the tissues for this girl is to A. begin a milrinone infusion B. change epinephrine to norepinephrine C. cool the child to 36.0°C D. normalize the pH E. transfuse 20 mL/kg packed red blood cells

A The patient described in the vignette has evidence of satisfactory intracardiac volumes, but skin signs of impaired perfusion, depressed contractility by echocardiography, and metabolic acidosis, suggesting inadequate nutrient delivery at the tissue level. Milrinone is a type III phosphodiesterase inhibitor that can increase cyclic adenosine monophosphate concentrations, which can lead to improved myocardial contractility, improved diastolic relaxation, and vasodilation. The combination of actions results in increased cardiac output and oxygen delivery through improved contractility, stroke volume, and tissue blood flow. In addition, in some situations, phosphodiesterase inhibitors have been shown to reduce platelet and white blood cell interactions that may lead to more sluggish flow through capillaries. The clinician should assure good intravascular volume before administering milrinone because vessel dilation could lead to hypotension unless volume is addressed adequately. Many patients who have catecholamine-resistant shock demonstrate improved cardiac output with milrinone. Milrinone is considered appropriate therapy for the treatment of shock in the presence of low cardiac index, normal blood pressure, and high systemic vascular resistance, as described for the girl in the vignette. Many factors interact to determine microvascular blood flow to tissues. The clinician treating a patient who has any form of shock ultimately must consider how to deliver oxygen and other nutrients to the capillaries perfusing vital organs. Increasing oxygen delivery at the macrocirculation level, as measured by pulmonary artery catheters, can be beneficial, but augmenting nutrient delivery at the microvascular level is believed to be a better target for resuscitation. Abnormalities of microvascular flow are a hallmark of sepsis, and the clinician must consider all factors that increase or decrease microvascular flow (Table) through a given capillary and an entire capillary bed. In shock states, and particularly in sepsis, capillaries within tissues may have no or reduced flow due to microthromboses, arteriole-venule shunts, and leukocyte aggregation, even in the presence of normal vital signs. Table: Factors Related to Microvascular Flow Increase Flow Decrease Flow Increased cardiac output Increased viscosity Increased perfusion pressure Alkalosis Acidosis Alpha-receptor agonists Hypoxemia Angiotensin Local increased metabolism Vasopressin Bradykinin Endothelin Histamine Increased calcium Increased potassium Decreased local metabolism Increased magnesium Thromboxane Increased PCO2 Thrombosis Nitric oxide Decreased red blood cell deformability Prostacyclin Increased white blood cell-endothelial adhesion Increased cGMP and cAMP Beta2 receptor agonists cGMP=cyclic guanosine monophosphate, cAMP=cyclic adenosine monophosphate The first priority in resuscitation from shock is improving cardiac output and blood pressure. Cardiac output is dependent on four variables: preload, afterload, contractility, and heart rate. In most forms of pediatric shock, intravascular volume is depleted, preload is reduced, and aggressive volume administration improves outcome by increasing myocardial stretch and intraventricular volume that enhances stroke volume and, subsequently, blood pressure. Increasing blood pressure leads to augmented flow through the microcirculation. Normally, increased flow enhances oxygen and nutrient delivery, which reduces local release of hydrogen ion, adenosine, and other factors, leading to vasoconstriction. Increased flow also leads to a reflexive "myogenic" constriction and endothelial cell shear stress. Such stress prompts the release of nitric oxide, resulting in local arteriolar dilatation and a decrease in blood pressure and systemic vascular resistance. When microvascular blood flow, hypoxemia, or other micronutrient (glucose, vitamin B complex) deficiency do not meet the metabolic needs of the tissues, anaerobic metabolism produces hydrogen ions and releases adenosine from the breakdown of adenosine triphosphate, causing local vasodilation that enhances local blood flow. In addition, inadequate nutrient substrate does not provide adequate energy for smooth muscle contraction, prompting the blood vessels to dilate. Because norepinephrine has more potent alpha-agonist effects than epinephrine, it is a drug of choice for warm shock with hypotension and brisk capillary refill. It may increase blood pressure, which could increase flow, but the concomitant vasoconstriction likely would impair flow to many vascular beds. Once blood pressure is adequate, as in the vignette, the clinician's goal should be to improve overall clinical perfusion by using vasodilators and inodilators in conjunction with ongoing attention to the provision of adequate intravascular volume. Attempts to decrease metabolic requirements are appropriate when struggling to deliver adequate nutrients to cells. Early intubation to reduce the work of breathing has become part of clinical practice parameters for pediatric shock. Changing the body temperature by 1.0°C changes oxygen consumption, carbon dioxide production, and related metabolic parameters by approximately 10% to 12%. However, a patient usually is cooled with cooling blankets or garments that can lead to vasoconstriction of the skin, patient discomfort, and endogenous release of alpha-agonist catecholamines. Also, in areas where vascular reactivity is normal, blood flow is autoregulated to meet the metabolic needs of those tissues, so a reduction in metabolism by decreasing temperature could lead to decreased microvascular blood flow. A common goal of intensive care is to correct acid-base disturbances to a normal pH of 7.4. However, acidosis (increased hydrogen ion concentration or elevated PCO2) is a stimulus for vasodilation. Correction of the acidosis for this girl by administering exogenous alkali or increasing minute ventilation could lead to vasoconstriction and reduced microvascular blood flow. Optimizing microvascular blood flow should improve nutrient delivery and allow spontaneous improvement in metabolic acidosis. Although myocardial contractility may be optimized at normal pH, a pH of 7.25 generally is well tolerated, and increasing the pH is unlikely to improve flow related to improved cardiac output significantly. Similarly, catecholamine responsiveness is minimally depressed by modest acidosis. Global oxygen delivery is determined by oxygen content of the blood and cardiac output: Oxygen delivery = [(PaO2)(0.003) + (hemoglobin)(1.34)(% oxygen saturation)] X cardiac output Packed red blood cell transfusion can increase hemoglobin and increase oxygen delivery, but increasing hemoglobin increases blood viscosity, thereby reducing microvascular flow. Transfused red blood cells are relatively depleted in 2,3 diphosphoglycerate and, therefore, are less deformable, leading to more sluggish flow through capillaries and reduced ability to unload the oxygen at the tissue level. Paradoxically, some studies have shown decreased microvascular blood flow after transfusion, as measured by gastrointestinal tonometry and orthogonal polarization spectral imaging. This finding may explain, in part, the increased mortality seen among adult patients who receive transfusions to maintain hemoglobin greater than 10 g/dL (100 g/L) compared with patients transfused to maintain hemoglobin greater than 7 g/dL (70 g/L). References: Ando J, Yamamoto K. Vascular mechanobiology: endothelial cell responses to fluid shear stress. Circ J. 2009;73:1983-1992. DOI: 10.1253/circj.CJ-09-0583. Accessed October 2009 at: http://www.jstage.jst.go.jp/article/circj/73/11/73_1983/_article Brierley J, Carcillo JA, Choong K, et al. Clinical practice parameters for hemodynamic support of pediatric and neonatal septic shock: 2007 update from the American College of Critical Care Medicine. Crit Care Med. 2009;37:666-688. Abstract accessed October 2009 at: http://www.ncbi.nlm.nih.gov/pubmed/19325359 den Uil CA, Klijn E, Lagrand WK, et al. The microcirculation in health and critical disease. Prog Cardiovasc Dis. 2008;51:161-170. Abstract accessed October 2009 at: http://www.ncbi.nlm.nih.gov/pubmed/18774014 Guyton AC, Hall JE. Overview of the circulation; medical physics of pressure, flow, and resistance. In: Textbook of Medical Physiology, 11th ed. Philadelphia, Pa: Elsevier Saunders; 2006:162-170 Guyton AC, Hall JE. The microcirculation and the lymphatic system: capillary fluid exchanges, interstitial fluid, and lymph flow. In: Textbook of Medical Physiology, 11th ed. Philadelphia, Pa: Elsevier Saunders; 2006:171-194 Guyton AC, Hall JE. Vascular distensibility and functions of the arterial and venous systems. In: Textbook of Medical Physiology, 11th ed. Philadelphia, Pa: Elsevier Saunders; 2006:171-180 Hébert PC, Wells GF, Blajchman MA, et al. A multicenter, randomized, controlled clinical trial of transfusion requirements in critical care. Transfusion Requirements in Critical Care Investigators, Canadian Critical Care Trials Group. N Engl J Med. 1999;340:409-417. Accessed October 2009 at: http://content.nejm.org/cgi/content/full/340/6/409 Schlame M, Blanck TJJ. Cardiovascular System. In: Gabrielli A, Layon AJ, Yu M, eds. Civetta, Taylor & Kirby's Critical Care. 4th ed. Philadelphia, Pa: Lippincott Williams & Wilkins, a Wolters Kluwer company; 2009:682-697 Spronk PE, Zandstra DF, Ince C. Bench-to-bedside review: sepsis is a disease of the microcirculation. Crit Care. 2004;8:462-468. DOI: 10.1186/cc2894. Accessed October 2009 at: http://ccforum.com/content/8/6/462 Trzeciak S, Cinel I, Phillip Dellinger R, et al. Resuscitating the microcirculation in sepsis: the central role of nitric oxide, emerging concepts for novel therapies, and challenges for clinical trials. Acad Emerg Med. 2008;15:399-413. DOI: 10.1111/j.1553- 2712.2008.00109.x. Accessed October 2009 at: http://www3.interscience.wiley.com/cgibin/ fulltext/119413681/HTMLSTART American Board of Pediatrics Content Specification(s): Understand how pH alters myocardial function Understand the relationship between intravascular volume and pressure and organ blood flow Understand the concept of blood flow autoregulation Understand the contribution of autoregulation to cardiac output Know which humoral factors influence tissue blood flow Understand the effects of hypoxemia on systemic vascular resistance Understand how acidosis affects tissue perfusion Understand how alkalosis affects tissue perfusion

Feb 2010 - Question 5 You are treating a 16-year-old girl who has enterovirus-induced myocarditis. Her myocardial function is recovering slowly. Her ejection fraction is 0.22, she has a normal lactic acid value, and she is normotensive. Her cardiovascular support consists of milrinone 0.375 mcg/kg per minute, furosemide 1 mg/kg intravenously every 8 hours, and spironolactone 25 mg/kg orally every 12 hours. The consulting cardiologist recommends starting enalapril 2.5 mg orally every 12 hours and titrating the enalapril dose up by 2.5 mg every other day until the maximum tolerated dose is reached, not to exceed 20 mg daily. He recommends weaning milrinone to offset the vasodilation from enalapril. The patient's current electrolyte results are: Sodium, 135 mEq/dL (135 mmol/L) Potassium, 4.8 mEq/dL (4.8 mmol/L) Chloride, 99 mEq/dL (99 mmol/L) Total carbon dioxide, 28 mEq/dL (28 mmol/L) Urea nitrogen, 32 mg/dL (11.4 mmol/L) Creatinine, 1.5 mg/dL (132.6 mcmol/L) Your fellow asks how to assess the maximum tolerated enalapril dose. Of the following, the finding that MOST commonly signals the need to discontinue angiotensinconverting enzyme (ACE) inhibitors is A. increase in serum creatinine by 0.7 mg/dL (61.9 mcmol/L) B. increase in serum potassium by 0.9 mEq/L (0.9 mmol/L) C. onset of angioedema D. orthostatic hypotension for 1 hour after each dose E. persistence of a dry cough for 2 weeks

A Treatment of moderate-to-severe congestive heart failure (CHF) in pediatric patients is not supported by a large volume of literature; much of the current therapeutic knowledge comes from large adult trials. The standard of care includes early use of an ACE inhibitor and diuretics, as planned for the girl in the vignette. Optional additional therapy includes beta-adrenergic blockade and digoxin. ACE inhibitor and diuretic therapy should begin early in the stabilization because they have been proven to minimize mortality and maximize clinical function in adults. ACE inhibitors have variable risks and benefits in individual CHF patients. The risks can be monitored as the starting dose is titrated upward. The most common risks relate to the interaction of intravascular volume depletion (which may result from loop diuresis), poor cardiac output, and vasodilation. Intolerance requires discontinuation of therapy in 3% to 6% of cases. The most common abnormality requiring discontinuation of enalapril and other ACE inhibitors is a decrease in renal function. ACE inhibitors modulate the renin-angiotensin system that, in turn, preserves renal perfusion pressure. Enalapril competes with angiotensin I at the ACE molecule and reduces the conversion of angiotensin I to angiotensin II. Cardiovascular effects include systemic vasodilation and vasodilation of the efferent arteriole of the glomerulus. The renal perfusion pressure may decline below the level required to preserve renal autoregulation. In adults, the lower end of the physiologic range of renal perfusion pressure is approximately 80 mm Hg. A lack of autoregulation means the glomerular filtration rate may not be preserved. When the renal perfusion pressure is below the range that supports autoregulation, the glomerular filtration rate declines. The initial clinical effect of this decline is an increase in creatinine and urea. If the effect is clinically important and not alleviated, acute renal failure may develop. Management may include reduction in ACE inhibitor dose, fluid loading, vasopressor infusion, and renal replacement therapy. The humoral effects of ACE inhibitors include a decrease in the aldosterone concentration in plasma. Decreased plasma aldosterone may lead to transient hyperkalemia in about 4% of cases of CHF treated with enalapril. This effect may resolve with adjustment of the enalapril dose, the potassium intake, and the dose of potassium-sparing diuretics. Angioedema is a life-threatening swelling of the upper airway and possibly the gastrointestinal tract that may result from accumulation of bradykinin. Angioedema occurs in fewer than 1% of patients receiving ACE inhibitors. African Americans are at a fourfold greater risk compared with whites. Angioedema requires expert airway management and resolves with discontinuation of the ACE inhibitor. Dry cough is another effect of bradykinin accumulation that affects approximately 2% of patients who have CHF and receive enalapril, but it requires discontinuation of the drug in fewer than 1% of cases. Postdose hypotension, which can be mild or moderate, is expected with ACE inhibitor therapy. For patients who have CHF, blood pressure must be monitored for the duration of enalapril titration and several weeks thereafter. The reduction in blood pressure reflects the afterload reduction that is desired in CHF. However, orthostasis and syncope can occur in 5% to 7% of cases of CHF treated with enalapril. Patients whose intravascular volume is depleted from diuretic use appear to be at increased risk. Approximately 2% of CHF patients require discontinuation of enalapril because of this effect. Additional patients require fluid repletion or adjustment of the diuretic dose. References: Enalaprilat/Enalapril Maleate. In: McEvoy GK, ed. AHFS Drug Information 2009. Bethesda, Md: American Society of Health System Pharmacists; 2009:1990-2000. Fenton M, Burch M. Understanding chronic heart failure. Arch Dis Child. 2007;92:812-816. Abstract accessed October 2009 at: http://www.ncbi.nlm.nih.gov/pubmed/17715446 Jackson EK. Renin and angiotensin. In: Brunton LL, Lazo JS, Parker KL, eds. Goodman and Gilman's The Pharmacological Basis of Therapeutics. 11th ed. New York, NY: McGraw-Hill Companies, Inc; 2006:789-823 Lewis AB, Chabot M. The effect of treatment with angiotensin-converting enzyme inhibitors on survival in pediatric patients with dilated cardiomyopathy. Pediatr Cardiol. 1993;14:9-12. Abstract accessed October 2009 at: http://www.ncbi.nlm.nih.gov/pubmed/8456034 Schnellmann RG. Toxic responses of the kidney. In: Klaassen CD, ed. Casarett and Doull's Toxicology: The Basic Science of Poisons. 7th ed. New York, NY: McGraw-Hill Companies, Inc; 2008:chapter 14 American Board of Pediatrics Content Specification(s): Plan medical therapy for a patient with congestive heart failure Know the mechanism of action of commonly used antihypertensive drugs Know the adverse effects and toxicities associated with the use of various antihypertensive agents

Feb 2010 - Question 8 A 16-year-old boy who has severe intellectual disability, cerebral palsy, and chronic seizure disorder is admitted to the PICU because of worsening of his seizures. At the facility where he resides, he has been receiving appropriate dosages of phenytoin and phenobarbital via his gastrostomy tube (GT). Caretakers noted today that he exhibited more tachypnea this morning, and he desaturated during a coughing episode. They administered GT prednisone, albuterol nebulizers, and GT erythromycin for a clinical exacerbation of his wheezing and possible pneumonia. Upon arrival to the PICU, the boy is having several generalized tonic-clonic seizures that are increasing in frequency and intensity. Your review of his medical history reveals that his seizures are very difficult to treat. You decide to add another anticonvulsant that will work via a different mechanism from those he already is receiving. Of the following, the MOST appropriate anticonvulsant to add is A. carbamazepine B. ethosuximide C. fosphenytoin D. lorazepam E. valproic acid

A seizure is a paroxysmal disorder of the central nervous system (CNS) grey matter characterized by an abnormal neuronal discharge associated with a change in function of the patient. Seizures are caused by abnormal synchronous electrical discharge (depolarization) of a group of neurons in the CNS. Depolarization results from the influx of sodium into the neuron; repolarization results from the egress of potassium from the cell, which restores the resting negative electrical potential across the cell. This potential is regulated by a sodium-potassium pump that is driven by adenosine triphosphate. Figures 1 and 2 provide an excellent overview of the physiology of a normal neuron and the pathophysiology associated with seizures. Figure 1: Normal neuronal firing. Schematic of neuron with one excitatory (E) and one inhibitory (I) input. Right side shows membrane potential (in mV), beginning at resting potential (-70mV). Activation of E leads to graded excitatory postsynaptic potentials, the larger of which reaches threshold (about -40mV) for an action potential. The action potential is followed by an after hyperpolarization (AHP), the magnitude and duration of which determine when the next action potential can occur. Activation of I causes an inhibitory postsynaptic potential. Inset shows magnified portion of the neuronal membrane as a lipid bilayer with interposed voltage-gated Na+ and K+ channels; the direction of ion fluxes during excitation is shown. After firing, the membrane-bound Na-K pump and star shaped astroglial cells restore ionic balance. Reprinted with permission from Stafstrom CE. Back to basics: the pathophysiology of epileptic seizures: a primer for pediatricians. Pediatr Rev. 1998;19:342-351. Figure 2: Abnormal neuronal firing. Abnormal neuronal firing at the levels of A) the brain and B) a simplified neuronal network, consisting of two excitatory neurons (1 and 2) and an inhibitory interneuron (3). EEG (top set of traces) and intracellular recordings (bottom set of traces) are shown for the normal (left column), interictal (middle column), and ictal conditions (right column). Numbered traces refer to like numbered recording sites. Note time scale differences in different traces. A. (Brain) Three EEG electrodes record activity from superficial neocortical neurons. In the normal case, activity is low voltage and "desynchronized" (neurons are not firing together in synchrony). In the interictal condition, large spikes are seen focally at electrode 2 (and to a lesser extent at electrode 1, where they might be termed" sharp waves"), representing synchronized firing of a large population of hyperexcitable neurons (expanded in time below). The ictal state is characterized by a long run of spikes. B. (Neuronal Network) At the neuronal network level, the intracellular correlate of the interictal EEG spike is called the "paroxysmal depolarization shift" (PDS). The PDS is initiated by a non-NMDA-mediated fast excitatory postsynaptic potential (EPSP) but is maintained by a longer, larger NMDA-mediated EPSP. The post-PDS hyperpolarization (*) temporarily stabilizes the neuron. If this post-PDS hyperpolarization fails (right column, thick arrow), ictal discharge can occur. The lowermost traces, recordings from neuron 2, show activity similar to that recorded in neuron 1, with some delay (double-headed arrow). Activation of inhibitory neuron 3 by firing of neuron 1 prevents neuron 2 from generating an action potential (the inhibitory excitatory postsynaptic potential [IPSP] counters the depolarization caused by the EPSP). If it does not reach firing threshold, neuron 2 can then recruit additional neurons, leading to an entire network firing in synchrony (seizure). Reprinted with permission from Stafstrom CE. Back to basics: the pathophysiology of epileptic seizures: a primer for pediatricians. Pediatr Rev. 1998;19:342-351. Ineffective recruitment of inhibitory neurons coupled with excessive neuronal stimulation is key to the initiation and propagation of the electrical disturbance occurring in epileptic seizures. Figure 3 demonstrates normal synaptic transmission for both inhibitory and excitatory neurons. Figure 3: Normal synaptic transmission. Representative inhibitory and excitatory presynaptic terminals and postsynaptic neurons are shown. A. Inhibitory synapse. GABA binding to its postsynaptic GABAA receptors allows influx of Cl- ions, which hyperpolarizes the postsynaptic neuron (inhibitory postsynaptic potential; see text). GAD=glutamic acid decarboxylase. B. Excitatory synapse. Glutamate released from the terminal crosses the synaptic cleft and binds to one of several glutamate receptor subtypes (NMDA or non-NMDA; see text). Binding to non-NMDA receptors causes a "fast" excitatory postsynaptic potential; binding to NMDA receptors produces a longer, "slow" excitatory postsynaptic potential. If the postsynaptic neuron is depolarized sufficiently to reach firing threshold, an action potential will occur. Inset shows details of the NMDA receptor-ion pore complex. For the NMDA-ion pore to open, several events must occur: glutamate (circle) must bind to the receptor, glycine (triangle) must bind to its own receptor site on the NMDA receptor complex, and when the cell is sufficiently depolarized, Mg++ must leave the channel pore. Only then can Na+ and Ca++ flow into the neuron and produce a prolonged NMDA-mediated excitatory postsynaptic potential. Illustration by Marcia Smith and Alan Michaels. Reprinted with permission from Stafstrom CE. Back to basics: the pathophysiology of epileptic seizures: a primer for pediatricians. Pediatr Rev. 1998;19:342-351. Gamma-aminobutyric acid (GABA) is the major inhibitory neurotransmitter in the CNS. GABA receptors have a chloride ion channel complex and binding sites for barbiturates and benzodiazepines. Activation of these receptors allows for an inward movement of chloride, which serves to restore the negative resting potential to the neuron to prevent further firing. The use of GABA agonists (benzodiazepines, phenobarbital) and the resultant suppression of seizure activity support the role that GABA plays in the normal termination of a seizure. Glutamate is a major excitatory neurotransmitter of the CNS and has been implicated in the propagation of seizures by its effects on the N-methyl-D aspartate (NMDA), non-NMDA channels and via secondary messenger systems (metabotropic receptors). Glutamate agonists produce seizures and glutamate antagonists inhibit seizures. The continued and unabated stimulation of neurons by glutamate ultimately leads to neuronal cell failure and increased concentrations of intracellular calcium, which contribute to neuronal injury in patients suffering prolonged seizures. Calcium activates proteases and lipases that degrade intracellular elements, leading to mitochondrial dysfunction and cellular necrosis. The longer that a seizure continues, the harder it becomes to stop the seizure pharmacologically, possibly due to a shift from inadequate GABAergic inhibitory activity to excessive NMDA excitation. The boy described in the vignette is receiving phenobarbital and phenytoin. Phenobarbital binds to the GABA receptor and improves its effect by extending the duration of GABAmediated chloride channel openings, leading to decreased neuronal firing. Lorazepam and other benzodiazepines also exert their effect on the GABA receptor. Phenytoin blocks neuronal sodium channels, and carbamazepine works via a similar mechanism. Fosphenytoin is a prodrug of phenytoin that works via the same mechanism as phenytoin, with effects on sodium channels. Although ethosuximide works via a different mechanism by inhibiting T-type calcium current and blocking synchronized thalamic firing, it is only effective in treating absence seizures and has no activity in generalized tonic-clonic seizures, which the boy in the vignette is experiencing. Valproic acid works via multiple mechanisms: 1) increases brain GABA concentrations; 2) enhances GABA release by a presynaptic effect (it has no direct effects on GABA receptors); 3) increases GABA synthesis by activating glutamic acid decarboxylase; 4) inhibits GABA metabolism by inhibiting nerve terminal GABA transaminase; and 5) suppresses high-frequency, repetitive neuronal firing by blocking sodium channels at different sites from phenytoin and carbamazepine. Figure 4 demonstrates the actions of some of the more commonly used antiepileptic drugs and their effects on both the inhibitory and excitatory neurons. Figure 4: Actions of antiepileptic drugs on inhibitory (A) and excitatory (B) mechanisms. Drugs that enhance inhibition have been developed to act at both preand postsynaptic sites to enhance GABAergic inhibition. AEDs targeting excitation affect primarily postsynaptic mechanisms. Ketamine and Mg++ are not strictly AEDs, but are shown here to illustrate their actions at a specific site (the ion pore) on the NMDA receptor. Several of the newer AEDs (gabapentin, lamotrigine, felbamate, topiramate) probably have multiple mechanisms of action. Illustration by Marcia Smith and Alan Michaels. Reprinted with permission from Stafstrom CE. Back to basics: the pathophysiology of epileptic seizures: a primer for pediatricians. Pediatr Rev. 1998;19:342-351. Many new drugs are being studied. Some of the more common agents and their proposed mechanisms of action are summarized in the Table. Table: Common Antiepileptic Agents Agent Proposed Mechanism of Action Felbamate Inhibits glutamatergic neurotransmission (reduces NMDA action and blocks gylcine site on NMDA receptor) Gabapentin GABA analog, increases synaptic GABA, decreases intraneuronal calcium Lamotrigine Decreases glutamate release, inhibits calcium currents, blocks sodium channels Levetiracetam Unknown, but likely multiple mechanisms Oxcarbazepine Inhibits sodium channels, inhibits voltageactivated calcium currents Progabide GABA agonist at A and B sites Tiagabine GABA uptake inhibitor Toprimate Sodium channel blocker, decreases L-type calcium currents, potentiates GABA and enhances chloride flux, inhibits glutamatergic neurotransmission Vigabatrin GABA-transaminase inhibitor, inhibits GABA uptake Zonisamide Blocks sodium channels, blocks calcium channels, enhances GABA action As for the boy in the vignette, drug interactions must be considered in patients receiving multiple medications. Carbamazepine, phenytoin, fosphenytoin, and phenobarbital are metabolized in the liver via the cytochrome P450 enzyme system and excreted by the kidney. Carbamazepine also has an active epoxide metabolite. Medications that are cytochrome P450 enzyme inhibitors may increase concentrations of antiepileptic medications; drugs that are cytochrome P450 enzyme inducers may increase the metabolism of antiepileptics, leading to subtherapeutic concentrations. Common inhibitors of the P450 system include acetazolamide, cimetidine, clarithromycin, diltiazem, erythromycin, ketoconazole, protease inhibitors, and verapamil. The boy in the vignette is receiving erythromycin, which may decrease clearance of carbamazepine and phenytoin. Enzyme inducers, including rifampin, steroids, phenobarbital, phenytoin, primidone, and carbamazepine, may lead to decreased concentrations of other antiepileptics and medications. The interaction of these medications is complex, and serum concentrations of the antiepileptics must be followed closely. References: Macleod S and Appleton RE. The new antiepileptic drugs. Arch Dis Child Educ Pract Ed. 2007; 92:182-188 Sills GJ, Brodie MJ. Update on the mechanisms of action of antiepileptic drugs. Epileptic Disord. 2001;3:165-169. Accessed October 2009 at: http://www.john-libbeyeurotext. fr/en/revues/medecine/epd/e-docs/00/01/AD/DD/article.phtml Stafstrom CE. Back to basics: the pathophysiology of epileptic seizures: a primer for pediatricians. Pediatr Rev. 1998;19:342-351. Accessed October 2009 at: http://pedsinreview.aappublications.org/cgi/content/full/19/10/342 Taketomo CK, Hodding JH, Kraus DM. Pediatric Dosage Handbook. 14th ed. Hudson, Ohio: Lexicomp Inc; 2007 American Board of Pediatrics Content Specification(s): Understand the normal mechanisms of metabolism, absorption, distribution, and elimination of the common anticonvulsants and the relationship between their pharmacokinetics and their effects Recognize the effects of anticonvulsants on the metabolism of other drugs Understand the mechanism of action of specific anticonvulsants

Jan 2010 - Question 5 A 10-month-old girl underwent tracheal reconstruction shortly after birth and has been in the ICU all of her life. She has a tracheostomy and is ventilator-dependent. She has developed a temperature of 40.0oC, and the chest radiograph shows new infiltrates and pulmonary consolidation. You order cultures and begin empiric cefepime and vancomycin. Respiratory cultures subsequently grow multidrug-resistant Acinetobacter baumannii. Of the following, the MOST important reason to add intravenous colistin to the antibiotic regimen is because colistin A. can prevent bacterial DNA replication B. causes bacterial cell wall disruption C. confers bacteriostatic activity D. has a high volume of distribution E. is resistant to beta-lactamase action

B

April 2010 - Question 3 A 12-year-old, 40-kg girl who has idiopathic scoliosis, but is otherwise well, is admitted to the PICU following anteroposterior spinal fusion. She is sedated, intubated, and mechanically ventilated. The resident chose initial ventilator settings of a rate of 24, tidal volume of 8 mL/kg, peak inspiratory pressure of 34 cm H2O, positive end-expiratory pressure of 5 cm H2O, and Fio2 of 0.4. A respiratory therapist brings the following results of arterial blood gas analysis (ABG) to you: pH of 7.22, Paco2 of 49 mm Hg, Pao2 of 132 mm Hg, and base excess of -6 mEq/L. However, there is no name on the laboratory sheet, and you are not sure that this ABG applies to this patient. Your physical examination confirms adequate chest rise and appropriate function of the ventilator. You order the ABG to be repeated. Of the following, the patient in the PICU who is MOST likely to have these results is A. a 10-year-old boy who has acute respiratory distress syndrome, has been receiving ventilation for 2 weeks, and now is responding briskly to aggressive diuresis B. a 13-year-old boy who has muscular dystrophy; receives ventilation at home; has a history of recurrent urinary tract infections; was admitted today because of fever, tachycardia, and hypotension; and has been placed on a ventilator with his routine home ventilator settings C. a 30-month-old boy who was just admitted after ingesting multiple codeine tablets approximately 2 hours ago D. a 4-year-old girl who has septic shock, was admitted to the PICU 4 hours ago after resuscitation with 80 mL/kg of 0.9% sodium chloride, and currently is intubated and mechanically ventilated in synchronized intermittent mandatory ventilation with pressure control E. the girl described in the vignette

B Acid production in the body occurs mostly as carbon dioxide (CO2) production during oxidative metabolism of carbohydrates, fats, and amino acids. CO2 subsequently is hydrated by carbonic anhydrase to form carbonic acid that, in turn, dissociates into hydrogen ion and bicarbonate: H+ + HCO3 - <-> H2CO3 <-> CO2 + H2O. Acid production also occurs by generation of nonvolatile metabolic acids that are the byproducts of a variety of metabolic processes, such as incomplete catabolism of fat and carbohydrates or oxidation of sulfur-containing amino acids. An increase in H+ (or decrease in pH) shifts the previously noted equation to the right, and increased minute ventilation, which occurs within minutes, promotes elimination of the CO2 and return of pH toward normal. The respiratory response to altered pH or PaCO2 occurs nearly instantaneously as a result of central nervous system and peripheral chemoreceptors. The renal contribution to acid-base balance is complex. The kidneys must reabsorb filtered bicarbonate and excrete the daily metabolic acid load. Most filtered bicarbonate is reabsorbed in the proximal tubule. A smaller amount is reabsorbed by intercalated cells of the distal tubule and collecting ducts. Although filtered HCO3 - cannot cross from the tubular lumen into the renal tubular cell, it combines with secreted hydrogen ions to form H2CO3, which rapidly dissociates to form CO2 and H2O, as described previously. The CO2 readily diffuses back into the cell, combines with H2O, and regenerates H2CO3. Thus, bicarbonate "reabsorption" requires secretion of hydrogen ions into the tubule lumen. Hydrogen ions are actively excreted into the tubular lumen by an Na+-H+ antiporter in the proximal tubule and an apical H+-ATPase in alphaintercalated cells in the loop of Henle and collecting duct. Secreted hydrogen ions are bound in the lumen by ammonia and other titratable acids, preventing back diffusion of the protons and promoting acid secretion, or combine with filtered bicarbonate to form CO2 and H2O and regenerate H2CO3. Hydrogen ion secretion in the collecting tubule is primarily responsible for acidification of the urine and compensation for respiratory acidosis. In acidotic states, this process is markedly enhanced, but the compensation that results occurs over hours to days. Interpretation of blood gas analysis is a routine aspect of ICU management and contributes to both diagnosis and treatment. The first step is to determine whether the pH is acid, alkaline, or normal. The next logical step is to determine whether the pH and PaCO2 are both acid or both alkaline. If so, the PaCO2 contributes to the condition (eg, if the pH is acid and the PaCO2 is elevated [acid], respiratory acidosis is present). If the pH and the base excess (BE) are both acid or both alkaline, a primary metabolic disorder is present. If either the PaCO2 or the BE is normal in the setting of an abnormal pH, the disorder is uncompensated. If both PaCO2 and BE are abnormal in the same direction as the pH, this represents a combined metabolic and respiratory acidosis or alkalosis. These very simple rules-of-thumb do not adequately address the possibility of a mixed acid-base disturbance, which is evaluated better if the total CO2 and anion gap can be determined, but requires electrolyte and HCO3 - concentrations not presented in the vignette. The blood gas results presented in the vignette include a pH in the acidotic range. The PaCO2 is also acid (ie, elevated), indicating a primary respiratory acidosis. In addition, the BE is negative. Because both the PaCO2 and the BE are acid, not only is the respiratory acidosis not compensated, but a metabolic acidosis coexists. The patient most likely to have a combined respiratory and metabolic acidosis is the boy who has muscular dystrophy, chronic respiratory insufficiency, and acute sepsis. His chronic ventilator settings may not correct his respiratory insufficiency fully, leaving him with an elevated PaCO2. With the onset of septic shock, it appears that he has developed a superimposed metabolic acidosis. He is unable to compensate for either the metabolic or the respiratory acidosis. The boy who ingested codeine would be expected to have an uncompensated primary respiratory acidosis due to the central respiratory depression caused by codeine and other narcotic analgesics. Two hours after ingestion is too soon to expect significant metabolic compensation. Because the kidneys can only eliminate about 0.1 moles (100 mEq) of acid per day, metabolic compensation typically requires 1 to 3 days to become apparent. Assuming the child does not develop respiratory arrest with associated hypoxemia, the natural history is resolution of the respiratory acidosis after the drug is metabolized or possibly development of a mild compensatory metabolic alkalosis. An example of typical ABG results might be: pH of 7.34, PaCO2 of 49 mm Hg, Pao2 of 130 mm Hg, and BE of 1 mEq/L, with the pH and PaCO2 both acid and little, if any, change in the BE. The girl who has septic shock, has received volume resuscitation, and now is mechanically ventilated would be expected to have a metabolic acidosis. If she is breathing spontaneously, she should generate a compensatory respiratory alkalosis. She is likely to have a combined lactic and hyperchloremic acidosis as a consequence of her sepsis and large-volume sodium chloride resuscitation. A typical ABG result might be: pH of 7.22, PaCO2 of 31 mm Hg, Pao2 of 130 mm Hg, and BE of -12 mEq/L. The low pH defines the presence of an acidosis. The acid BE indicates a primary metabolic abnormality and reflects the severity of the metabolic derangement. The PaCO2 is alkaline (ie, the deviation is in the opposite direction from the pH), indicating a respiratory compensation. The boy who has acute respiratory distress syndrome and has been ventilated for 2 weeks most likely would have had normal acid-base balance or a compensated respiratory acidosis, if permissive hypercapnia had been necessary. Brisk diuresis frequently results in development of a metabolic alkalosis caused by chloride depletion, hypokalemia, and volume contraction. A typical ABG result might reveal: pH of 7.44, PaCO2 of 49 mm Hg, PaO2 of 132 mm Hg, and BE of 8 mEq/L. In this instance, the pH and BE are both alkalotic (positive BE), indicating a primary metabolic alkalosis, with the PaCO2 diverging from normal in the acidotic direction, suggesting that the that there is a compensatory respiratory acidosis. Knowledge of the patient's course and prior blood gas values can aid in determining how much of the alkalosis is due to the diuresis and how much can be considered compensatory for the earlier ventilation. It is unlikely that the ABG values belong to the girl in the vignette. The ventilator settings chosen by the resident should produce an acute, uncompensated, respiratory alkalosis. At a rate of 24 and a tidal volume of 8 mL/kg, her minute ventilation would be approximately 7.7 L. Assuming her tidal volume at rest is about 6 mL/kg and her respiratory rate is approximately 20 breaths/min, her expected minute ventilation is approximately 5 L and perhaps less, given her residual anesthetic-related sedation. Respiratory alkalosis occurs in some primary respiratory disorders (eg, asthma, pneumonia, pulmonary edema), hypoxemia, central nervous system injury, early sepsis, salicylate and other drug toxicity, pregnancy, liver failure, and transient episodes of anxiety. In the PICU, inadvertent mechanical hyperventilation may be the most common cause. Typical ABG values might be: pH of 7.53, PaCO2 of 28 mm Hg, PaO2 of 145 mm Hg, and BE of 1 mEq/L. The pH defines the presence of an alkalosis, and the similarly alkalotic PaCO2 indicates that the respiratory abnormality is primary. In this case, there is no metabolic compensation, which is expected because of the short time the patient has been on these ventilator settings. References: Brandis K. Acid-base physiology tutorial. Anaesthesia Education Website. Accessed October 2009 at:http://www.AnaesthesiaMCQ.com/AcidBaseBook/ABindex.php Carillo-Lopez H, Chavez A, Jarillo A, Olivar V. Acid-base disorders. In: Fuhrman BP and Zimmerman J, eds. Pediatric Critical Care. 3rd ed. Philadelphia, Pa: Mosby Elsevier; 2006:958-989 Grogono AW. Acid-Base Tutorial. 2008. Accessed October 2009 at: http://www.acidbase. com/index.php Martin L. Electrolytes and the anion gap. In: All You Really Need To Know To Interpret Arterial Blood Gases. Accessed October 2009 at: http://www.lakesidepress.com/pulmonary/ABG/bicarbgap.98.htm American Board of Pediatrics Content Specification(s): Distinguish disorders of acid base homeostasis, including primary, compensatory, and mixed disturbances Understand the role of ventilation in the acid buffering system Know the principal causes, pathophysiology, and treatment of respiratory alkalosis

April 2010 - Question 5 A 12-year-old girl presents with nausea, vomiting, oliguria, and facial rash that has a malar distribution. Laboratory analysis reveals anemia, thrombocytopenia, and renal failure. Complement values are low, and an antinuclear antibody panel is pending to confirm the diagnosis of suspected lupus erythematosus (SLE). After placing a femoral dialysis catheter to assist in acute management, the patient's nurse asks you what causes SLE. Of the following, a TRUE statement about the pathogenesis of SLE is that A. a family history of autoimmune disease is not associated with the risk of developing SLE B. complement deficiency is strongly associated with disease severity C. high cytokine concentrations bear little relation to disease activity D. interferon activates dendritic cells to differentiate into T cells E. plasma cell precursors are transiently elevated

B Homozygous deficiency of any of the early components of the complement pathway (C1q, C1r, C1s, C4, and C2) predisposes to the development of SLE. Such deficiencies represent the strongest SLE susceptibility factors identified in humans. Further, a hierarchy of association appears to exist between disease severity, prevalence, and the missing protein. The strongest prevalence (>80%) and the most severe disease is noted in patients who are deficient in one of the C1 complex or C4 molecules. Deficiency of the C2 complement is associated with less severe disease. Siblings of patients who have SLE display a 10- to 20-fold increased risk of developing the disease. Monozygous twins have a 24% concordance rate; heterozygous twins have a concordance rate of 2%. A familial history of autoimmune disease is a significant risk factor for SLE. The risk of developing SLE increases with the number of first-degree relatives who have autoimmune diseases from an odds ratio of 4.1 (one relative) to 11.3 (two or more relatives). Interferon appears to be an important intermediary in the pathogenesis of SLE. Interferon activates dendritic cells to differentiate into B cells. Dendritic cells are specialized bone marrow-derived leukocytes that are critical to the development of immunity. Dendritic cells in the periphery capture and process antigens, express lymphocyte costimulatory molecules, migrate to lymphoid organs, and secrete cytokines to initiate immune responses. They not only activate lymphocytes, but they tolerize T cells to antigens that are innate to the body (self-antigens), thereby minimizing autoimmune reactions. The dual role in regulating immune responses and immune tolerance suggests that when pathologic conditions exist, dendritic cells may play a role in the development of autoimmune diseases such as SLE. Dendritic cells may present ''self'' antigens as immunogens, fail to delete autoreactive T cells, produce excess inflammatory cytokines, and inhibit induction of regulatory T cells. Even though concentrations of dendritic cells are significantly decreased in the blood of children who have SLE, dendritic cells accumulate at sites of SLE involvement. The blood of children who have SLE contains unusual CD14+ "monocytes" that have properties of dendritic cells in that they have the ability to induce naïve T-lymphocyte proliferation. Upon exposure to the serum of such patients, healthy monocytes aggregate and differentiate into CD14+ dendritic cells in the presence of interferon. Interferon activates dendritic cells, and under the influence of interferon, dendritic cells may select rather than kill autoreactive T cells and contribute to the development of autoimmunity. Regardless of age at SLE presentation, patients have hypergammaglobulinemia and increased serum autoantibody titers. Adult and pediatric patients exhibit profound alterations in B cells. Plasma cell precursors (PCPs) in children display a phenotype similar to that of PCPs found in the blood during secondary immune responses. Normally, immune responses cause transient elevations of these cells, but children who have SLE show a persistent elevation of phenotypically altered PCPs. This finding suggests that mature B cells are driven persistently to differentiate into plasma cells that migrate through the blood from the secondary lymphoid organs in SLE. Such plasma cells are deposited in the bone marrow, where they may become long-lived, and end organs, where inflammation occurs. The concentrations of a number of cytokines (interleukin [IL]-6, -10, -12, and -18) are elevated in patients who have SLE, and their presence correlates with disease activity. The chronic elevation of interferon increases the differentiation of dendritic cells into B cells. Further, myeloid dendritic cells trigger B-cell growth and differentiation through stimulation by IL-12 and -6. Immune stimulation by viruses can induce the maturation of activated B cells into plasma cells under the influence of IL-6. Recent prospective studies suggest that clinical SLE is preceded by the progressive accumulation of autoantibodies many years (up to 9 years) before diagnosis. Antinuclear, anti-Ro, anti-La, and antiphospholipid antibodies appear first, followed by anti-dsDNA antibodies and then anti-Sm and antinuclear ribonucleoprotein antibodies. Clinically overt disease appears to develop closer to the time of breakdown of tolerance against DNA and ribonucleoproteins. First, immune complexes against DNA have been shown to coengage receptors on autoreactive B cells, leading to autoantibody secretion. Second, immune complexes containing anti-DNA and anti-RNP antibodies have been shown to activate dendritic cells via interferon that induce the differentiation of B cells into antibodysecreting plasma cells and activate monocytes that produce a self-amplifying pathogenic loop. T cells from patients who have SLE display increased and accelerated signaling responses to receptor engagement, suggesting that they are "primed" for and respond to activation. In SLE, T-cell hyperreactivity may be intrinsic (genetically determined) or result from abnormal in vivo exposure to cytokines or antigen-presenting cells (dendritic cells). References: Hardin JA. Dendritic cells: potential triggers of autoimmunity and targets for therapy. Ann Rheum Dis. 2005;64(suppl 4):iv86-iv90. DOI: 10.1136/ard.2005.044560. Accessed October 2009 at: http://www.ncbi.nlm.nih.gov/pmc/articles/PMC1766899 Lovell DJ, Ruth NM. Pediatric clinical research. Curr Opin Rheumatol. 2005;17:265-270. Abstract accessed October 2009 at: http://www.ncbi.nlm.nih.gov/pubmed/15838234 Stichweh D, Arce E, Pascuala V. Update on pediatric systemic lupus erythematosus. Curr Opin Rheumatol. 2004;16:577-587. Abstract accessed October 2009 at: http://www.ncbi.nlm.nih.gov/pubmed/15314498 American Board of Pediatrics Content Specification(s): Know the pathophysiology and pathogenesis of systemic lupus erythematosus

April 2010 - Question 7 You are treating a 6-year-old 25-kg boy who suffered blunt abdominal injury 4 days ago. He had pancreatic injury, a duodenal perforation, and lung injury that led to bronchopleural fistula and lung contusions, which has made his mechanical ventilation a challenge. Surgeons have reattached his small bowel, but they want to avoid enteral nutrition for at least 1 more week. For this reason, intravenous nutrition was started 2 days ago that includes: Dextrose, 17.5% Amino acids, 1 g/kg per day Intravenous fat emulsion, 1 g/kg per day Sodium, 6 mEq/100 mL Potassium, 4 mEq/100 mL Acetate, 1 mEq/100 mL Trace element solution Multivitamins His fluid rate is 60 mL/hr, and he appears clinically euvolemic. Laboratory studies document:Sodium, 133 mEq/L (133 mmol/L) Potassium, 4.4 mEq/L (4.4 mmol/L) Chloride, 92 mEq/L (92 mmol/L) Bicarbonate, 28 mEq/L (mmol/L) Blood urea nitrogen, 17 mg/dL (6.1 mmol/L) Creatinine, 0.8 mg/dL (70.7 mcmol/L) Blood glucose, 168 mg/dL (9.3 mmol/L) Triglycerides, 380 mg/dL (4.3 mmol/L) pH, 7.27 PaCO2, 64 mm Hg PaO2, 61 mm Hg Of the following, the MOST appropriate modification of the intravenous nutrition order is A. dextrose 12.5%, amino acids 0.5 g/kg per day, and lipids 1.5 g/kg per day B. dextrose 12.5%, amino acids 1.5 g/kg per day, and lipids 1 g/kg per day C. dextrose 17.5%, amino acids 1.5 g/kg per day, and discontinue lipids for 1 day D. dextrose 20%, amino acids 1.5 g/kg per day, and lipids 0.5 g/kg per day E. leave unchanged, but titrate an insulin infusion to normalize blood glucose

B The boy described in the vignette has hyperglycemia, hypertriglyceridemia, hypercarbia, and respiratory acidosis. Nutrition can have an impact on these conditions and the child's hospital course. Evidence suggests that significant hyperglycemia worsens both morbidity and mortality in critically ill patients. During the metabolic stress response that usually is seen with major trauma or sepsis, exogenous glucose does not shut down gluconeogenesis, so patients often cannot tolerate usual glucose infusion rates. Further, as a metabolic substrate, glucose has a respiratory quotient of 1.0, meaning that one mole of carbon dioxide is generated for every mole of oxygen consumed. Adequate nutrition using a substrate that generates less carbon dioxide is optimal when the patient has difficulty with ventilation. Excessive carbohydrate administration also leads to the production of fatty acids and triglycerides, and the respiratory quotient for creating fat from carbohydrate is very high. Among the potential harms of hypertriglyceridemia is worsening pancreatitis. The respiratory quotient of amino acids is 0.8 and that of fat is 0.7. These are among the reasons that fat is often a preferred substrate for patients experiencing severe stress. The nutrition goals for this boy are to supply appropriate caloric support, avoid hyperglycemia, provide ample amino acids to optimize protein synthesis, and provide adequate gas exchange without inducing ventilator-induced lung injury. Accordingly, intravenous nutrition should avoid carbohydrate administration in excess of caloric need, include ample amino acids to address the catabolic condition, and use lipids as an energy source that generates smaller amounts of carbon dioxide per calorie. Decreasing the glucose concentration to 12.5%, increasing the amino acids administration to 1.5 g/kg per day, and monitoring triglyceride values on a similar amount of administered fat (1 g/kg per day) is a prudent approach. Of note, caloric requirements for a patient on mechanical ventilation are reduced, and sedation and muscle relaxation further reduce required calorie intake. Finally, because intravenous feeding does not require specific dynamic action, about 10% fewer calories are required when feeding intravenously than enterally. Permissive underfeeding or supplying about 80% of calculated or measured (by indirect calorimetry) needs is recommended during the acute phase of critical illness. Underfeeding reduces the risks of hyperglycemia, insulin resistance, infection, and prolonged mechanical ventilation. Targeting "normal" calorie intake for most critically ill patients results in excess calories, which are metabolized to fat and carbon dioxide. This patient's blood urea nitrogen is not significantly elevated, so he is likely to tolerate an increase in the amino acids provided. Some experts suggest rapid advancement of amino acids in this setting, although the general recommendation is a gradual increase by 0.5 g/kg per day to a maximum of 2.5 to 3 g/kg per day to help minimize catabolism and muscle wasting. Protein administration of more than 4 g/kg per day has been associated with azotemia, metabolic acidosis, and neurodevelopmental abnormalities. Amino acid restriction should be considered only in the presence of significant uremia or hyperammonemia. Administration 0.5 g/kg per day of amino acids is generally the minimum needed to assure administration of essential amino acids and to decrease catabolism. Removing intravenous fat from the nutrition provided to the boy in the vignette may improve the hypertriglyceridemia but likely will not address his hyperglycemia and hypercarbia. Adjusting the nutrition by giving similar calories but less glucose should improve the hyperglycemia and hypercarbia and allow adjustment of ventilator settings to decrease ongoing lung injury. Over time, withholding intravenous fat can contribute to the development of essential fatty acid deficiency, especially because patients have increased fat oxidation during the metabolic stress response seen after major trauma. Triglyceride concentrations up to about 400 mg/dL (4.5 mmol/L) rarely cause acute problems. Ideally, triglycerides should be measured after a 4-hour interruption in lipid infusion to determine lipid tolerance. Increasing the glucose infusion rate likely will worsen the hyperglycemia and increase fatty acid production, hypertriglyceridemia, and carbon dioxide production, thereby worsening respiratory acidosis. The use of insulin to maintain euglycemia is common practice, but adding insulin to this patient's current intravenous nutrition likely would increase formation of fatty acids and worsen the hypertriglyceridemia. In addition, adding insulin to a generous glucose infusion is implicated in the development of hepatosteatosis. Because the production of fatty acids from carbohydrate produces a large amount of carbon dioxide, this patient's respiratory acidosis could worsen, leading to increases in ventilator support and related ventilator-induced lung injury. References: Baker SS, Baker RD, Macronutrients. In: Baker SS, Baker RD, Davis AM, eds. Pediatric Nutrition Support. Sudbury, Mass: Jones and Bartlett Publishers, Inc; 2007:299-311 Chwals WJ. Energy metabolism and appropriate energy repletion in children. In: Baker SS, Baker RD, Davis AM, eds. Pediatric Nutrition Support. Sudbury, Mass: Jones and Bartlett Publishers, Inc; 2007:65-79 Kleinman RE. Nutrition of children who are critically ill. In: Pediatric Nutrition Handbook. 5th ed. Elk Grove Village, Ill: American Academy of Pediatrics; 2004:643-652 McClave SA, Martindale RG, Vanek VW, et al. Guidelines for the provision and assessment of nutrition support therapy in the adult critically ill patient: Society of Critical Care Medicine (SCCM) and American Society for Partneral and Enteral Nutrition (A.S.P.E.N). JPEN J Parenter Enteral Nutr. 2009:33:277-316. Accessed October 2009 at: http://pen.sagepub.com/cgi/content/full/33/3/277 Mehta MN, Compher C, A.S.P.E.N. Board of Directors. A.S.P.E.N. clinical guidelines: nutrition support of the critically ill child, 2009. JPEN J Parenter Enteral Nutr. 2009;33:260-276. Accessed October 2009 at: http://pen.sagepub.com/cgi/content/full/33/3/260 Shulman RJ, Phillips S. Partneral nutrition indications, administration, and monitoring. In: Baker SS, Baker RD, Davis AM, eds. Pediatric Nutrition Support. Sudbury, Mass: Jones and Bartlett Publishers, Inc; 2007:273-286 American Board of Pediatrics Content Specification(s): Characterize protein metabolism of the stress response Understand aspects of stress metabolism Characterize carbohydrate metabolism of the stress response Understand the principles of nutritional support during the stress response Understand the principles of nutritional support for patients with multiple organ system failure

Feb 2010 - Question 4 Two days after undergoing replacement of a right ventricle-to-pulmonary artery conduit, a patient in the ICU develops bradycardia and hypotension. Her medications include continuous furosemide infusion, intravenous chlorothiazide, milrinone infusion, and sodium nitroprusside. Laboratory data show: Sodium 130 mEq/L (130 mmol/L) Potassium 2.7 mEq/L (2.7 mmol/L) Chloride 98 mEq/L (98 mmol/L) Carbon dioxide 19 mEq/L (19 mmol/L) Magnesium 1.1 mg/dL Calcium 7.2 mg/dL (1.8 mmol/L) Ionized calcium 4.84 mg/dL (1.21 mmol/L) Phosphorus 3.2 mg/dL (1.03 mmol/L) Blood urea nitrogen 10 mg/dL (3.57 mmol/L) Creatinine 0.8 mg/dL (70.7 mcmol/L) Her electrocardiographic tracing is shown in the Figure. Figure Of the following, in addition to an intravenous magnesium sulfate bolus, the MOST appropriate agent to administer intravenously as soon as possible is A. calcium chloride B. potassium chloride C. sodium bicarbonate D. sodium phosphate E. 3% sodium chloride

B The patient described in the vignette has developed torsades de pointes (TdP), a polymorphic, wide-complex ventricular tachycardia that is associated most often with hypomagnesemia. Her serum magnesium and potassium values are low and likely the cause of TdP. Many reports indicate that resolution of TdP in response to intravenous magnesium is sustained when hypokalemia is corrected at the same time. Therefore, this patient should receive immediate intravenous potassium chloride. Although she has a relatively low serum sodium value of 130 mEq/L (130 mmol/L), this is not the cause of TdP, and supplementation with 3% sodium chloride is not needed. Her total and ionized calcium are within normal limits, obviating the need for calcium chloride. The phosphorus value is normal, and although the carbon dioxide content is low at 19 mEq/L (19 mmol/L), it is not the cause of TdP. Magnesium is one of the most common cations in the body and is the second most predominant intracellular cation after potassium. Nearly half of total body magnesium is stored in bone, half exists in tissue, and only 0.3% is present in serum. Thus, low serum magnesium concentrations often underestimate total body depletion, and repletion with serial dosing frequently is recommended. Because magnesium is highly protein-bound, ionized hypomagnesemia may be more common than measurement of total magnesium would suggest. Magnesium is essential as a cofactor in several hundred enzymatic reactions and is crucial in activities such as gating of calcium channels, transmembrane (sodium and potassium pump) ion flux, cardiac excitability, and regulation of adenylate cyclase. Magnesium is excreted and controlled through the kidneys, particularly in the thick ascending limb of the loop of Henle. Magnesium and calcium have a strong relationship. The release of intracellular calcium is related to phospholipase C activation and hydrolysis of phosphatidylinositol 4,5- biphosphate into inositol 1,4,5-triphosphate (IP3). Binding of IP3 to IP3 receptors leads to opening of the calcium channel and release of intracellular calcium stored in the sarcoplasmic reticulum (SR). Magnesium decreases IP3 activation of calcium release from the SR. In addition, magnesium increases calcium ATPase, the enzyme responsible for moving calcium back into the SR. Therefore, hypomagnesemia leads to increased SR release of calcium and decreased calcium return to the SR, both of which increase intracellular calcium concentrations and smooth muscle vasoconstriction. In addition to its effects on calcium, magnesium has a significant role in transmembrane sodium and potassium movement. Na-K-ATPase is the enzyme needed to pump sodium and potassium across plasma membranes. The enzyme is dependent on ATP hydrolysis for energy, which, in turn, is dependent on magnesium binding. Studies have shown an increase in intracellular sodium and calcium in the presence of hypomagnesemia and a corresponding decrease in magnesium and potassium concentrations. In addition, hypomagnesemia increases renal wasting of potassium. These abnormalities have been shown to decrease sinus node rate, prolong AV conduction, and increase AV node refractoriness, forming the basis for life-threatening arrhythmias associated with hypomagnesemia. Causes of hypomagnesemia include reduced dietary intake; decreased gastrointestinal absorption in conditions such as diarrhea, fistulas, and laxative use; and increased gastrointestinal losses via nasogastric drainage. Increased renal losses can occur with tubular defects, diabetes mellitus, and diuretic use. Patients who undergo cardiopulmonary bypass (CPB) for open heart surgery are at increased risk for hypomagnesemia. CPB-induced hypomagnesemia is well documented and likely is due to hemodilution and intraoperative and postoperative cellular depletion. Causes include priming solutions low in magnesium and catecholamine-induced lipolysis and free fatty acid chelation of magnesium. Aggressive use of diuretics in the postoperative period following open heart surgery also may contribute to hypomagnesemia. Other clinical scenarios in which hypomagnesemia often occurs include treatment of fungal infections with amphotericin, aminoglycoside use, transplant patients receiving cyclosporine and related agents, and oncology patients previously treated with cisplatin. EDITORS' NOTE: The laboratory values in this question were amended on February 17, 2010. References: Fawcett WJ, Haxby EJ, Male DA. Magnesium: physiology and pharmacology. Br J Anaesth. 1999;83:302-320. Accessed October 2009 at: http://bja.oxfordjournals.org/cgi/reprint/83/2/302 Tong GM, Rude RK. Magnesium deficiency in critical illness. J Intensive Care Med. 2005;20:3- 17. Abstract accessed October 2009 at: http://www.ncbi.nlm.nih.gov/pubmed/15665255 American Board of Pediatrics Content Specification(s): Plan treatment for a patient with hypomagnesemia

August 2010 - Question 5 An obese, 16-year-old girl was admitted directly to the intensive care unit after symptoms of chest pain and the presence of blood-tinged sputum. She reports sexual activity and the use of birth control pills. Physical examination reveals the following: heart rate, 125/min; respiratory rate, 45/min; blood pressure, 130/87 mm Hg; and temperature, 38.3oC. Oxygen saturation as measured by pulse oximetry is 85% on 50% supplemental oxygen by face mask. She is in respiratory distress, with tachypnea and retractions. Chest radiograph shows right lower lobe atelectasis. Arterial blood gas on 50% oxygen by face mask reveals the following: pH, 7.41; PCO2, 42 mm Hg; and PO2, 52 mm Hg. The result of a pregnancy test is positive. Capnometry is performed and shows a reading of 25 mm Hg. Of the following, which is the MOST likely explanation for the difference between PaCO2 and end-tidal carbon dioxide? A. atelectasis B. increased alveolar dead space C. increased lung perfusion D. increased minute ventilation E. pregnancy

B The patient described in the vignette has risk factors for the development of pulmonary embolism (PE), including obesity and the use of birth control pills. Because of a positive pregnancy test result, procedures such as pulmonary angiography and computed tomography (CT) cannot be performed. Although the number of children with PE is nowhere near that of adults, the incidence of PE is increasing. Reasons for this include advances in the care of children, which has led to increasing numbers of chronic vascular catheter placements and other technological and invasive therapies. PE is part of a spectrum of venous thromboembolic disease. An embolus originating from anywhere in the venous system can cause pulmonary artery occlusion. Obstruction of flow through the pulmonary artery results in increased dead space ventilation, where the affected lung segments are ventilated but not perfused. This is reflected in a much lower end-tidal carbon dioxide measurement compared with arterial PCO2. A widened alveolar to arterial gradient is observed as well, as described for the patient in the vignette. Eventually, PE increases the right ventricular afterload, with resultant increase in right ventricular end-diastolic volume, leading to bowing of the interventricular septum to the left. This decreases left ventricular diastolic filling and eventually leads to life-threatening decreased cardiac output and hypotension. Capnography measures the concentration or partial pressure of carbon dioxide in respiratory gases (Figure). Figure: Normal features of a capnogram. A, Baseline represents the beginning of expiration and should start at zero. B, The transitional part of the curve represents mixing of dead space and alveolar gas. C, The alpha angle represents the change to alveolar gas. D, The alveolar part of the curve represents the plateau average alveolar gas concentration. E, The end-tidal carbon dioxide value. F, The beta angle represents the change to the inspiratory part of the cycle. G, The inspiration part of the curve shows a rapid decrease in carbon dioxide (CO2) concentration. The capnogram is presented as a graph of expired carbon dioxide against time and provides information about carbon dioxide production, pulmonary perfusion, alveolar ventilation, respiratory patterns in disease, and elimination of carbon dioxide. Pulmonary angiography, ventilation perfusion scintigraphy, and helical CT scan of the chest remain the definitive tests to use for the diagnosis of PE; however, it may not be possible to perform these tests emergently or in pregnancy. Less invasive and more rapid tests have been studied to use as screening measures or to make the diagnosis before pulmonary angiography or CT scans. This may be particularly important in children who are critically ill or unstable. Such tests include D-dimer and the use of capnography. In PE, the affected lung parenchyma maintains normal ventilation but has no perfusion. Because of continued ventilation without perfusion and increased alveolar dead space, the exhaled carbon dioxide level is low. Kurt et al correlated the alveolar dead space fraction (AVDSf) with more invasive tests for PE such as angiography. The AVDSf was calculated based on the following formula: AVDSf = (PaCO2 − PETCO2)/PaCO2, where PETCO2 is the end-tidal pressure of the carbon dioxide. Of the 58 patients with suspected PE who were included in the study, 40 (69%) had confirmed PE. Patients with PE had an AVDSf score of 0.174, whereas those who did not have PE had an AVDSf score of 0.136. In addition, patients with PE had a PETCO2 of 28 (19-40) mm Hg compared with 35 (30-40) mm Hg in those who did not (P = .003). Capnography can be a valuable tool in helping in the diagnosis of PE. Although the patient has evidence of atelectasis on chest radiograph, this would not increase the dead space fraction sufficiently to account for the clinical findings described. With PE, lung perfusion is decreased or absent in the affected segment, not increased. Although the patient is tachypneic and may have increased minute ventilation, this does not account for the disparity of the carbon dioxide in arterial blood gas and end-tidal carbon dioxide. Progesterone levels rise progressively throughout pregnancy. Progesterone is a respiratory stimulant; however, pregnancy would not be expected to result in respiratory distress with oxygen desaturation. References: Jensen D, Duffin J, Lam YM, et al. Physiological mechanisms of hyperventilation during human pregnancy. Respir Physiol Neurobiol. 2008;161:76-86. DOI: 10.1016/j.resp.2008.01.001. Abstract accessed May 2010 at: http://www.ncbi.nlm.nih.gov/pubmed/18289946. Kurt OK, Aplar S, Sipit T, et al. The diagnostic role of capnography in pulmonary embolism. Am J Emerg Med. In press. Machida H. Influence of progesterone on arterial blood and CSF acid-base balance in women. J Appl Physiol. 1981;51:1433-1436. Abstract accessed May 2010 at: http://www.ncbi.nlm.nih.gov/pubmed/6797997. Tapson VF. Medical progress: acute pulmonary embolism. N Engl J Med. 2008;358:1037- 1052. Abstract accessed April 2010 at: http://www.ncbi.nlm.nih.gov/pubmed/7656544. Thompson JE, Jaffe MB. Capnographic waveforms in the mechanically ventilated patient. Respir Care. 2005;50:100-109. Accessed April 2010 at: http://www.rcjournal.com/contents/01.05/01.05.0100.pdf. American Board of Pediatrics Content Specification(s): Know that capnography may contribute to the diagnosis of conditions such as pulmonary embolus

August 2010 - Question 4 A 16-year-old boy with newly diagnosed Burkitt lymphoma and a large mediastinal mass is admitted to the pediatric intensive care unit for monitoring of his respiratory status. He tolerated sedation and diagnostic procedures adequately yesterday and was started on antineoplastic therapy. During the night and morning, his urine output has decreased significantly. Clinically, he is awake and alert although nauseated. He is well saturated (98%) with 1 L/min of nasal cannula oxygen with a respiratory rate of 20/min. His heart rate is 102/min, and his blood pressure is 125/78 mm Hg. He has puffy eyelids, peripheral edema, and muscle cramps. A recent analysis of his renal function revealed a blood urea nitrogen level of 45 mg/dL (16 mmol/L) and a creatinine level of 2.1 mg/dL (185 micromoles/L). Of the following, the MOST likely metabolic derangement to be detected in sampling his blood is: A. hypercalcemia B. hyperuricemia C. hypokalemia D. hyponatremia E. hypophosphatemia

B The patient described in the vignette is experiencing tumor lysis syndrome. Tumor lysis syndrome is a condition characterized by a host of metabolic derangements secondary to the rapid release of intracellular ions, proteins, and nucleic acids from the dying malignant cells. These metabolic derangements may result in a number of serious and even lifethreatening conditions, including seizures, tetany, renal failure, and cardiac dysrhythmias. The metabolic derangements most commonly attributed to tumor lysis syndrome include hyperkalemia, hyperphosphatemia, hypocalcemia, and hyperuricemia. Tumor lysis syndrome is most commonly reported in rapidly growing tumors that are highly sensitive to antineoplastic therapy. Common tumors include non-Hodgkin lymphoma, Burkitt lymphoma, acute lymphoblastic leukemia, and acute myelogenous leukemia. As described above, the pathophysiology of tumor lysis syndrome results from the rapid release of intracellular ions, proteins, and nucleic acids from malignant cells being lysed by antineoplastic therapy. This lysis results in large quantities of intracellular nucleic acids being released into the bloodstream. These nucleic acids are quickly broken down into uric acid, resulting in hyperuricemia. The hyperuricemia contributes to renal dysfunction fostering the other metabolic derangements observed in the setting of tumor lysis. Hyperkalemia is another common, potentially life-threatening metabolic derangement of tumor lysis syndrome. It results from the release of high concentrations of intracellular potassium into the bloodstream with lysis of malignant cells. The hyperkalemia may be exacerbated by concomitant renal dysfunction associated with the malignant neoplasm. This renal dysfunction may be secondary to tumor obstruction of the kidney and/or its blood supply. In addition, renal dysfunction may result from the tumor lysis secondary to hyperuricemia and/or the formation of calcium phosphate crystals in the renal tubules. The hyperkalemia will be exacerbated by the administration of potassium in the intravenous fluids, and thus, exogenous potassium administration must be avoided in any patient at risk for tumor lysis. Hyperphosphatemia may also result from tumor lysis in much the same manner. High concentrations of phosphorus are released into the bloodstream when a large number of malignant cells are lysed rapidly. The condition is compounded by the finding that malignant cells may contain as much as 4 times the amount of phosphorus stored in healthy cells. This load of phosphorus overwhelms the ability of the kidney to excrete phosphorus, resulting in hyperphosphatemia. Much like with potassium, concomitant renal dysfunction associated with the tumor or the tumor lysis may further exacerbate the hyperphosphatemia. The elevated phosphorus level increases the risk of calcium phosphorus precipitation. Calcium phosphate crystals precipitating in the renal tubules may exacerbate renal dysfunction, contributing further to the hyperphosphatemia. This binding of phosphorus to calcium may result in significant hypocalcemia. Although frequently asymptomatic, the condition may result in muscle cramps as observed in the patient described in the vignette or more serious complications, such as tetany, hypotension, or cardiac dysrhythmias. Calcium phosphorus products (total calcium × phosphorus in milligrams per deciliter) exceeding 70 have been associated with the formation of metastatic calcium phosphate crystals, complicating the treatment of the hypocalcemia. Although hyponatremia may occur in the setting of tumor lysis syndrome, it is not one of the classic metabolic derangements associated with the condition. References: Cairo MS, Bishop M. Tumour lysis syndrome: new therapeutic strategies and classification. Br J Haematol. 2004;127:3-11. DOI: 10.1111/j.1365-2141.2004.05094.x. Abstract accessed April 2010 at: http://www.ncbi.nlm.nih.gov/sites/entrez/15384972. Coiffier B, Altman A, Pui CH, Younes A, Cairo MS. Guidelines for the management of pediatric and adult tumor lysis syndrome: an evidence-based review. J Clin Oncol. 2008;26:2767-2778. Abstract accessed April 2010 at: http://www.ncbi.nlm.nih.gov/sites/entrez/18509186. American Board of Pediatrics Content Specification(s): Recognize tumor lysis syndrome Plan the management of a patient with tumor lysis syndrome

April 2010 - Question 6 You are evaluating a 1-month-old girl who was intubated and placed on mechanical ventilation for 2 weeks because of respiratory syncytial virus bronchiolitis. She has improved, and you wean her off mechanical ventilation and extubate her. Within 30 minutes of extubation, she develops significant stridor, retractions, bilateral decreased air entry, tachypnea, and tachycardia. Of the following, the therapeutic option MOST likely to have prevented this problem is A. helium-oxygen mixture B. inhaled budesonide C. intravenous dexamethasone D. nebulized epinephrine E. subcutaneous epinephrine

C Dexamethasone is a glucocorticosteroid that has predominant anti-inflammatory and minimal mineralocorticoid activity. Its anti-inflammatory actions are related to the inhibition of leukocyte migration and lymphocytic immune responses. Glucocorticoids have been shown to inhibit the production of factors involved in inflammation, including vasoactive and chemoattractant factors, as well as the secretion of lipolytic and proteolytic enzymes and to decrease migration of leukocytes to areas of injury. In addition, glucocorticoids have significant inhibitory effects on factors affecting production of cytokines such as interferon-gamma, granulocyte/monocyte colony-stimulating factor, interleukins, and tumor necrosis factor-alpha. Two randomized, controlled trials in children have shown that the administration of dexamethasone prior to extubation significantly reduced postextubation stridor and the need for epinephrine. In addition, one of the studies showed that children who received dexamethasone prior to extubation had a significantly lower incidence of reintubation. The benefits of dexamethasone appear greater in infants than in older children and adults. Epinephrine provides alpha- and beta-agonist effects and acts on vascular smooth muscle, resulting in decreased blood flow to capillary beds that, in turn, shrinks mucosal edema. When administered in nebulized form, effects are limited primarily to the respiratory tract. However, systemic effects such as tachycardia, hypertension, and hyperglycemia have been reported. Subcutaneous epinephrine injection has not been used in the prevention of postextubation subglottic edema, but it is likely that the pain of injection and increased systemic adverse effects would make this modality undesirable. Although nebulized epinephrine is effective in reducing subglottic edema, its effects may be short-lived, and repeated dosing can lead to tachyphylaxis. Inhaled budesonide has been shown to reduce airway edema in children who have croup or laryngotracheobronchitis, but it has not been studied in the prevention of postextubation airway edema. Helium-oxygen mixture is not known to reduce airway edema and inflammation. Its role in the treatment of airway obstruction is related to the conversion of turbulent airflow to laminar airflow, which is related to the low fluid density of helium that allows a reduction in the Reynold's number: RN = v x D x d μ v = velocity of fluid D = fluid density d = diameter of tube μ = viscosity of fluid A Reynold's number of less than 2,000 results in laminar flow. Laminar airflow, as opposed to turbulent, decreases the airway resistance, thereby lowering the work of breathing. The use of helium-oxygen is not preventative of upper airway edema, but rather a symptomatic treatment of the problem. References: Anene O, Meert KL, Uy H, Simpson P, Sarnaik AP. Dexamethasone for the prevention of postextubation airway obstruction: a prospective, randomized, double-blind placebocontrolled trial. Crit Care Med. 1996;24:1666-1669. Abstract accessed October 1009 at: http://www.ncbi.nlm.nih.gov/pubmed/8874303 Davies MW, Davis PG. Nebulized racemic epinephrine for extubation of newborn infants. Cochrane Database Syst Rev. 2002;1:CD000506. DOI:10.1002/14651858.CD000506. Accessed October 2009 at: http://www.mrw.interscience.wiley.com/cochrane/clsysrev/articles/CD000506/frame.html Khemani RG, Randolph AG, Markovitz B. Corticosteroids for the prevention and treatment of post-extubation stridor in neonates, children and adults. Cochrane Database Syst Rev. 2009;3:CD001000. DOI: 10.1002/14651858.CD001000.pub3. Accessed November 2009 at: http://www.mrw.interscience.wiley.com/cochrane/clsysrev/articles/CD001000/frame.html Klassen TP, Feldman ME, Watters LK, Sutcliff T, Rowe PC. Nebulized budesonide for children with mild-to-moderate croup. N Engl J Med. 1994;331:285-289. Accessed October 2009 at: http://content.nejm.org/cgi/content/full/331/5/285 Tellez DW, Galvis AG, Storgion SA, Amer HN, Hoseyni M, Deakers TW. Dexamethasone in the prevention of postextubation stridor in children. J Pediatr. 1991;118:289-294. Abstract accessed October 2009 at: http://www.ncbi.nlm.nih.gov/pubmed/1993963 American Board of Pediatrics Content Specification(s): Know the pharmacologic therapy for the treatment or prevention of laryngeal and subglottic edema

August 2010 - Question 3 A 3-year-old girl in the pediatric intensive care unit underwent liver transplantation 3 days ago and is receiving tacrolimus, methylprednisolone, ganciclovir, and trimethoprimsulfamethoxazole. Intraoperatively, she received fresh frozen plasma, packed red blood cells, and platelets but has not required blood products postoperatively. She currently is receiving 5% dextrose in 0.9% sodium chloride with 20 mEq/L of potassium chloride at a maintenance rate and has tolerated small amounts of clear liquids by mouth. She was extubated on the first postoperative day, and plans are to send her to the routine patient unit after morning abdominal ultrasonography. Which of the following electrolyte disturbances is she MOST likely to share with an 11-month-old infant with short gut syndrome, a 14-year-old girl with severe anorexia nervosa, a 9-year-old boy with multiple organ dysfunction receiving furosemide and chlorothiazide, and a 3-year-old girl with trisomy 21 who underwent atrioventricular canal repair and has a junctional ectopic tachycardia? A. hypercalcemia B. hyperkalemia C. hypomagnesemia D. hyponatremia E. hypophosphatemia

C Electrolyte disorders are extremely common among intensive care unit (ICU) patients of all ages, ranging from neonates to elderly adults, and the presence of one disorder frequently increases the likelihood of another. Underlying disease processes, developing organ system dysfunction, and multiple therapeutic agents contribute to their development. Liver transplant recipients are vulnerable to a wide variety of complications, many of which are related to the immunosuppressive agents, including, among others, electrolyte disorders, opportunistic infection, neurologic complications, renal dysfunction, and development of diabetes mellitus and posttransplantation lymphoproliferative disease and cancer. The calcineurin inhibitor tacrolimus, although probably a more effective immunosuppressant than cyclosporine, has been associated with more frequent development of hypomagnesemia and related tremor, seizures, and cardiac dysrhythmias. Hypomagnesemia also occurs in other solid organ and bone marrow transplant recipients treated with tacrolimus. Fractional and total excretion of magnesium in the urine is increased. Patients with massive small bowel resection preventing adequate enteral absorption are at risk for hypomagnesemia if intravenous supplementation is not provided. In other patients with malabsorption, complexing of magnesium with unabsorbed fats may result in magnesium deficiency. Patients with protein calorie malnutrition and those with anorexia nervosa are commonly magnesium deficient. Ionized hypomagnesemia is also common among children undergoing complex congenital heart surgery and may contribute to postoperative dysrhythmias, including junctional ectopic tachycardia, torsades de pointes, and other ventricular dysrhythmias. Postoperative hypomagnesemia may also contribute to prolonged mechanical ventilation and longer ICU length of stay. Preoperative use of diuretics and digoxin appears to be associated with postoperative hypomagnesemia. Use of diuretics, especially combined use of loop and thiazide diuretics, causes greater loss of magnesium than of sodium and calcium. Under normal circumstances, sodium reabsorption in the thick ascending loop of Henle creates a gradient that drives passive magnesium and calcium reabsorption from the lumen into the bloodstream. When sodium reabsorption is blocked, urinary magnesium and calcium losses increase. Hypercalcemia is less common in the pediatric ICU than the other disorders discussed here, and its effects may be difficult to recognize. However, it is associated with renal, cardiovascular, and central nervous system effects. Causes include hyperparathyroidism, malignant neoplasms (especially metastatic disease to bone), vitamin D intoxication, and a variety of uncommon metabolic disorders. Thiazide diuretics, which decrease renal calcium excretion, may cause hypercalcemia, but this would be unlikely when combined with a loop diuretic as in the patient described in the vignette. The calciuresis induced by loop diuretics serves as the basis for its use with aggressive isotonic hydration for treatment of hypercalcemia. Dehydration and hyperproteinemia may also be associated with hypercalcemia, but neither is described for the patients in the vignette. Hyperkalemia also occurs frequently in patients taking tacrolimus as a result of impaired renal excretion. However, it would not be expected in the other patients described unless their ICU course was complicated by renal failure or metabolic acidosis. Hypokalemia would be much more likely in most of them, potentially caused or exacerbated by the hypomagnesemia. Hypomagnesemia impairs the sodium-potassium pump, which allows potassium to be lost from intracellular fluid and excreted in the urine. Potassium depletion occurs during starvation, and refeeding, with associated insulin secretion, leads to cellular uptake of potassium and potentially severe, life-threatening hypokalemia. Hyponatremia is a common disorder in the pediatric ICU, particularly in patients receiving hypotonic intravenous fluids, which is not the case in the liver transplant recipient. It is also common in those who have required diuretics, with hyponatremia resulting from decreased sodium reuptake in the renal tubules. Numerous underlying disease processes are also associated with hyponatremia. The boy with multiple organ dysfunction syndrome receiving furosemide and chlorothiazide might be expected to have hyponatremia, and the other patients would be vulnerable to hyponatremia if they are receiving hypotonic fluids, but it is unlikely in the girl in the vignette: the blood products given intraoperatively represent a significant sodium load, and her postoperative fluids have all been isotonic. Hypophosphatemia is not expected in patients after liver transplantation. Renal dysfunction related to calcineurin inhibitors is more likely to cause hyperphosphatemia. Hypophosphatemia is a hallmark of the refeeding syndrome and may be associated with respiratory failure, severe neurologic symptoms, cardiovascular collapse, and a host of other disorders affecting virtually every system. References: Filler G. Calcineurin inhibitors in pediatric renal transplant recipients. Paediatr Drugs. 2007;9:165-174. Abstract accessed April 2010 at: http://www.ncbi.nlm.nih.gov/pubmed/17523697. Fuentebella J, Kerner JA. Refeeding syndrome. Pediatr Clin North Am. 2009;56:1201-1210. DOI: 10.1016/j.pcl.2009.06.006. Abstract accessed April 2010 at: http://www.ncbi.nlm.nih.gov/pubmed/19931071. Munoz R, Laussen PC, Palacio G, Zienko L, Piercey G, Wessel DL. Whole blood ionized magnesium: age-related differences in normal values and clinical implications of ionized hypomagnesemia in patients undergoing surgery for congenital cardiac disease. J Thorac Cardiovasc Surg. 2000;119:891-898. DOI: 10.1016/S0022-5223(00)70083-3. Accessed April 2010 at: http://jtcs.ctsnetjournals.org/cgi/content/full/119/5/891. Wood EB, Lynch RE. Electrolyte management in pediatric critical illness. In: Furhman BP, Zimmerman J, eds. Pediatric Critical Care. 3rd ed. Philadelphia, PA: Mosby; 2006:939-957. American Board of Pediatrics Content Specification(s): Know the causes of hypomagnesemia Recognize the manifestations of hypomagnesemia

Dec 2010 - Question 1 An 18-month-old girl was initially admitted to the pediatric intensive care unit (PICU) a week ago for lip and tongue burns and swelling she sustained when she pulled a strand of glowing Christmas tree lights from a tree and put it into her mouth. She was brought by her parents to the emergency department, where she was noted to have particularly severe oral burns. She was admitted to the PICU, where she subsequently was intubated with a cuffed endotracheal tube for progressive lip and tongue edema. She was mechanically ventilated for 3 days at a ventilator rate of 10/min, a peak inspiratory pressure of 22 cm H2O, a positive end-expiratory pressure of 5 cm H2O, and a fraction of inspired oxygen of 0.30 and maintained saturations of approximately 95%. Her PaCO2 was not determined. Examination in the operating room on the day after admission revealed full-thickness burns in the right oral commissure and a 1-cm necrotic lesion on the right anterior portion of her tongue with surrounding edema. In the operating room, her original oral endotracheal tube was replaced with a nasal tube when an upper incisor was noted to be loose, perhaps as a consequence of laryngoscopy, as well as to avoid further lip injury from the tube. A nasogastric tube was also placed to permit enteral feedings in anticipation of oral discomfort. During the next 2 days airway edema improved slightly, and she was extubated during the morning of the fifth hospital day. That evening she was transferred to the routine patient unit. On day 7 of her hospital stay, her nurse found her asleep in a pool of bloody fluid, apparently coming from her mouth. Although she was arousable, her heart rate was 155/min, blood pressure was 80/50 mm Hg, and her extremities were cool with 3- to 4-second capillary refill. After receiving 20 mL/kg of normal saline, she is transferred back to the PICU for evaluation, stabilization, and management. Which of the following is the MOST likely explanation for her bleeding? A. electrical injury-induced disseminated intravascular coagulation with bleeding from the site of her injured incisor B. gastric stress ulceration C. hemorrhage from a previously thrombosed labial artery D. nasopharyngeal trauma from her nasotracheal or nasogastric tube E. pulmonary contusion with delayed hemorrhage

C Most electrical injuries in children are from low-voltage sources (<600 V) and commonly involve the mouth as a result of an infant or child biting an electrical cord or mouthing an electrical item. Fewer injuries result from contact with high-voltage sources, such as downed power lines, and most of these occur in older children. The severity of the burn depends on the duration of contact, the intensity of the current, the surface area of contact, and the degree of tissue resistance. Tissues with relatively low resistance include nerves, blood, mucous membranes, and muscle; higher resistance is found in bones, fat, and tendons. Skin has intermediate resistance and is quite variable: thick calloused skin has much greater resistance than the thin, high-water-content skin of an infant, and wet skin provides virtually no resistance. Injury may occur as a direct effect of the electrical current (eg, cardiac rhythm disturbances, severe muscle contractions, seizures), as a result of the transformation of electrical current to heat, or a combination of both. Oral tissues are readily injured because of the low resistance of moist, highly vascular structures. Current arcing through the lips and other oral structures causes the burns, which tend to be small and deep (full thickness) and involve the mucosa, submucosa, muscle, nerves, and blood vessels. Outpatient management is adequate for most patients with these injuries, but where severe injury occurs, airway protection may be necessary. As with other burns, significant edema occurs, often within hours, and is accompanied by eschar formation. Small arteries are frequently injured acutely, but bleeding may not occur until the eschar falls off days or even 2 to 3 weeks after the initial injury. At that time, significant hemorrhage may occur as noted in the patient described in the vignette. Although large arteries tend to dissipate heat because of the high blood flow rate within them, they are vulnerable to medial necrosis and subsequent aneurysm and rupture. On rare occasions children with electrical mouth burns experience injury to a carotid artery or one of its major branches. Severe electrical injury may cause multisystem injury. Skin burns vary from small and superficial (first degree) to much more extensive third-degree burns. Rhythm disturbances, including asystole, and myocardial necrosis of varying degree are common but are most common in patients in whom the current has traversed the thorax. Vascular injury as described above may result in tissue ischemia and extensive muscle necrosis. Respiratory arrest may result from injury to the central nervous system respiratory center or tetanic contraction of respiratory muscles. Pulmonary contusion may occur if the patient is thrown or knocked to the ground, but this is much more common in those experiencing high-voltage injury. Nervous system manifestations include spinal cord and cranial nerve injury, seizures, and peripheral nerve injuries. Depending on the severity and constellation of injuries, including cardiac arrest and the need for cardiopulmonary resuscitation or extensive surface burns, patients may develop multiorgan dysfunction, disseminated intravascular coagulation, acute respiratory distress syndrome, acute renal failure, gastric stress ulceration, and other complications of critical illness. The child described in the vignette does not appear to have had this degree of systemic illness. Ventilatory requirements during her initial pediatric intensive care unit stay did not suggest significant parenchymal disease. Although gastric ulceration is possible, gastric feeding further decreases the likelihood that this is the cause of her bleeding. Nasopharyngeal trauma related to nasotracheal tube placement and/or removal is possible but would be most likely to occur immediately or within a short time after the tube manipulation. Her nasogastric tube might also be a source of nasopharyngeal injury, but in most cases, superficial mucosal injury would be unlikely to cause bleeding sufficient to cause hemodynamic instability. References: Koumbourlis AC. Electrical injuries. Crit Care Med. 2002;30:S424-S430. Abstract accessed September 2010 at: http://www.ncbi.nlm.nih.gov/pubmed/12528784. Ewald MB, Baum CR. Environmental injuries. In: Fleisher GR, Ludwig S, Henretig FM, eds. Textbook of Pediatric Emergency Medicine. 5th ed. Baltimore, MD: Lippincott Williams & Wilkins; 2008:1026-1031. American Board of Pediatrics Content Specification(s): Recognize that an electrical injury may be associated with delayed vascular injury

Feb 2010 - Question 3 You are caring for a 2-week-old male infant who has refractory supraventricular tachycardia. He initially required synchronized cardioversion to establish a normal sinus rhythm and subsequently was started on digoxin therapy. However, he continues to experience frequent breakthrough episodes of supraventricular tachycardia. The resident caring for the child suggests administering verapamil to improve control of these tachydysrhythmias. You explain that verapamil should be avoided in this age group. Of the following, the BEST rationale for avoiding verapamil in the neonate is the A. decrease in the ratio of contractile elements to collagen, extracellular matrix, and vascular elements in the fetal myocardium B. difference in beta-receptor function in the neonatal myocardium and the higher circulating catecholamines C. immaturity of the sarcoplasmic reticulum in neonatal cardiomyocytes D. lower capacity to use fatty acids in the neonatal heart E. relative predominance of parasympathetic innervations in the neonatal heart compared with the mature myocardium

C Neonates have a limited ability to store calcium in the sarcoplasmic reticulum, and administration of calcium channel blockers can result in hemodynamic collapse. Several reports from the 1980s described an association between the administration of verapamil and hemodynamic collapse in this age group. The sarcoplasmic reticulum is much less developed in the neonatal myocardium. In fact, electron micrographs reveal that the sarcoplasmic reticulum not only is sparse in the immature cardiomyocytes, but it lacks transverse tubules. Therefore, neonatal cardiomyocytes are much more dependent on extracellular calcium influx and less reliant on intracellular sources of calcium, placing the neonate at greater risk with the use of a calcium channel blocker such as verapamil. Studies suggest that the neonatal heart differs from the adult heart in many aspects. Although there is a relative predominance of parasympathetic innervations in the neonatal heart compared with the mature myocardium, this does not explain the negative consequences of verapamil in this age group. Both parasympathetic and sympathetic innervations are present by 20 weeks of gestation, although parasympathetic innervation occurs first and continues to predominate throughout the neonatal age group. The lower capacity of the neonatal heart to use fatty acids may influence its ability to withstand hypoxemia or ischemia, but it is not likely to influence the impact of verapamil. Lactate and glucose are the primary energy substrates in the fetal heart; fatty acids are the primary energy substrate for the adult heart. The transition from carbohydrate to fatty acid metabolism is complex and involves maturation of mitochondrial processes and significant changes in circulating concentrations of fatty acids and lactate. The decrease in the ratio of contractile elements to collagen, extracellular matrix, and vascular elements in the fetal myocardium makes the fetal cardiomyocyte much less effective at generating force. Electron micrographs reveal that the diameter of the fetal cardiomyocyte is smaller than that of the adult and that the proportion of noncontractile mass to the number of myofibrils is significantly higher than in the adult. Approximately 60% of the adult cardiac muscle consists of contractile mass compared with only 30% of the fetal muscle. As a result, the adult myocardium generates significantly more force per unit area despite the ability of the individual myofilaments of the fetal and adult heart to generate similar force. Although this may contribute to the negative consequences of verapamil in the neonate, it provides a less complete explanation than the differences in the utilization of calcium. Beta-adrenergic receptors in the neonatal heart are pharmacologically identical to those in the adult. Beta-adrenergic receptors and the adenyl cyclase system are well developed by late fetal life, with receptor density peaking at birth. However, beta-adrenergic receptor responsiveness decreases with time due to downregulation and decreased agonist binding of beta-1 receptors, uncoupling of beta-2 receptors, and abnormal G proteinmediated signal transduction. These changes appear to be the result of increased thyroid and adrenocorticosteroid hormone concentrations at birth. The differences in beta receptor function at birth, along with higher circulating catecholamines, decrease responsiveness to exogenously delivered catecholamines. These findings should not be the primary influence on the impact of verapamil in the neonate. References: Friedman WF. The intrinsic physiologic properties of the developing heart. Prog Cardiovasc Dis. 1972;15:87-111. DOI: 10.1016/0033-0620(72)90006-0 Garson A Jr. Medicolegal problems in the management of cardiac arrhythmias in children. Pediatrics. 1987;79:84-88. Abstract accessed October 2009 at: http://pediatrics.aappublications.org/cgi/content/abstract/79/1/84 Kirk CR, Gibbs JL, Thomas R, Radley-Smith R, Qureshi SA. Cardiovascular collapse after verapamil in supraventricular tachycardia. Arch Dis Child. 1987;62:1265-1266. DOI: doi:10.1136/adc.62.12.1265. Abstract accessed October 2009 at: http://adc.bmj.com/cgi/content/abstract/62/12/1265 Kocis KC, Graciano AL, Meliones JN. Developmental cardiac physiology. In: Wheeler DS, Wong HR, Shanley TP, eds. Pediatric Critical Care Medicine: Basic Science and Clinical Evidence. London, England: Springer-Verlag; 2007:591-601 Lopaschuk GD, Collins-Nakai RL, Itoi T. Developmental changes in energy substrate use by the heart. Cardiovasc Res. 1992;26:1172-1180. DOI: 10.1093/cvr/26.12.1172. Abstract accessed October 2009 at: http://cardiovascres.oxfordjournals.org/cgi/content/abstract/26/12/1172 White M, Roden R, Minobe W, et al. Age-related changes in beta-adrenergic neuroeffector systems in the human heart. Circulation. 1994;90:1225-1238. Abstract accessed October 2009 at: http://circ.ahajournals.org/cgi/content/abstract/90/3/1225 American Board of Pediatrics Content Specification(s): Know the general contributions of the developmental embryology of the heart to congenital heart disease Know the changes in myocardial function that occur with postnatal development

April 2010 - Question 4 A 17-year-old boy is brought to the emergency department from a party by friends following the acute onset of altered mental status and agitation. The friends admit that they were drinking alcohol at the party and that some partygoers were smoking illicit drugs. Physical examination reveals diaphoresis, a Glasgow Coma Scale Score of 10 (E2, V3, M5), heart rate of 180 beats/min, blood pressure of 150/92 mm Hg, respiratory rate of 20 breaths/min, and temperature of 40.0°C. Abnormal laboratory values include a slightly elevated creatine kinase of 260 U/L and a serum bicarbonate of 16 mEq/L (16 mmol/L). Urine toxicology is negative for amphetamines, narcotics, and benzodiazepines. Of the following, the MOST likely explanation for this boy's findings is A. blockade of alpha-1 receptors B. blockade of beta-1 receptors C. blockade of catecholamine reuptake D. increased monoamine oxidase enzyme activity E. stimulation of alpha-2 receptors

C The boy described in the vignette exhibits the classic signs and symptoms of sympathomimetic toxicity via hyperactivity of the adrenergic sympathetic system: tachycardia, hypertension, fever, and sweating. He also has altered mental status and elevated creatine kinase and low serum bicarbonate values. These findings suggest that he has ingested cocaine. Sympathomimetic toxicity may result from direct release of norepinephrine (NE) stores from storage vesicles with drugs such as amphetamines and tyramine or from inhibition of the reuptake of NE that already has been released, as seen in cocaine intoxication Cocaine stimulates alpha-1-, beta-1-, and beta-2-adrenergic receptors through increased concentrations of NE and, to a lesser extent, epinephrine. Cocaine produces preferential alpha-receptor activity on the cardiac and peripheral vasculature and additional cardiac effects through betaadrenergic agonism. The euphoric effects derive from inhibition of neuronal serotonin reuptake in the central nervous system (CNS). Cocaine also increases the concentration of the excitatory amino acids glutamate and aspartate in the brain. Glutamate is the primary excitatory neurotransmitter of the CNS. CNS toxicity is characterized by altered mental status, dizziness, headaches, parasthesias, tremors, seizures, strokes, transient ischemic attacks, and coma. NE is an endogenous catecholamine that is synthesized in the body from L-tyrosine and is produced in the brain, chromaffin cells of the adrenal medulla, and neurons of the sympathetic nervous system. It is the predominant catecholamine in peripheral tissue and sympathetic nerves. NE is released from its storage in vesicles at the nerve cell endings after nerve stimulation. The release occurs by the process of exocytosis, is aided by the contraction of the neurofibrils of the cells, and is an energy-dependent process requiring adenosine triphosphate. NE has generalized effects because of the varied locations of its receptor sites. It also travels via the circulation as a neurohumor following spillover from the sympathetic system and via synthesis and secretion from the chromaffin cells in the adrenal medulla. Once released, NE acts on effector sites by binding to and stimulating adrenergic receptors, which prompts intracellular release of adenosine monophosphate. The effects of catecholamines are mediated by cell-surface receptors. Adrenergic neurons belong to the large family of G protein-coupled receptors, which are subdivided into alpha and beta subunits. Beta-adrenergic stimulation in the heart causes increased inotropy, dromotropy, and to a lesser extent, chronotropy. Alpha-1-adrenergic stimulation, especially in the peripheral vasculature, causes vasoconstriction, mediated by increased cyclic adenosine monophosphate and intracellular calcium influx. Sympathetic nerve fibers branch and supply all arteries throughout the body with NE-containing adrenergic nerve endings. These nerve endings reach all blood vessels except true capillaries and cause intense vasoconstriction, especially in the small arteries and arterioles, which are richly innervated. Thus, NE may cause an increase in blood pressure, and constriction of veins (not as richly innervated) may increase blood return to the heart. In addition to the postsynaptic receptors described previously, presynaptic alpha-2 receptors control the rate of NE release and when stimulated, cause a decrease in NE release. Once NE is released from adrenergic nerve endings or the adrenal medulla, it has a short plasma half-life and is inactivated quickly. Inactivation occurs primarily through reuptake of storage granules back into the neuron (50% to 80%) but also through conversion to metabolites and excretion. The remaining NE diffuses away from the nerve endings into surrounding body fluids and into the blood, which accounts for removal of most of the remaining NE, with small amounts degraded by tissue enzymes. Metabolism of NE may occur via one of two enzyme pathways. Mitochondrial monoamine oxidase (MAO) is a flavoprotein located on outer mitochondrial membranes found in the nerve endings. The MAO enzymes exist in two forms, with MAO-A preferentially deaminating NE and serotonin and MAO-B acting on dopamine. Increased MAO enzyme activity would be unlikely to produce findings in the vignette because NE concentrations would decrease. Catechol-Omethyltransferase, which is located in the cytoplasm of most tissues, is responsible for inactivation of NE that is not effectively removed from the synaptic cleft. The resulting metabolites are excreted in the urine. Cocaine is well absorbed following contact with the oral, nasal, gastrointestinal, rectal, and vaginal mucosa or via the pulmonary alveoli following inhalation. The bioavailability is 90% after inhalation and 80% after nasal insufflation. The three major metabolites are benzoylecgonine, ecgonine methyl ester, and norocaine. In the presence of ethanol, cocaine is transesterified to a cocaethylene metabolite, which has similar pharmacologic properties and a longer half-life than cocaine. This metabolite is also vasoconstrictive, cardiotoxic, dysrhythmogenic, and neurotoxic. The pharmacokinetics of cocaine are summarized in the Table. Table: Pharmacokinetics of Cocaine Route of Administration Onset of Action Peak of Action Duration of Action Intravenous <60 seconds 3 to 5 minutes 30 to 60 minutes Nasal 1 to 5 minutes 20 to 30 minutes 60 to 120 minutes Smoking <60 seconds 3 to 5 minutes 30 to 60 minutes Gastrointestinal 30 to 60 minutes 60 to 90 minutes Unknown Treatment of a suspected overdose begins with the airway, breathing, and circulation. The cardiovascular symptoms may be alleviated by sedation with a benzodiazepine because the cardiovascular stimulation is centrally mediated by the sympathetic nervous system. Sedation also can help calm the agitated patient. Beta-adrenergic antagonists (beta blockers) should NOT be used because they may create unopposed alpha- adrenergic stimulation. Patients exhibiting hyperthermia should be cooled rapidly (<30 min). More assistance or a consultation with a medical toxicologist can be obtained at the United States Poison Control Network (1-800-222-1222). References: Cantwell GP, Weisman RS. Poisoning. In: Nichols DG, ed. Rogers' Textbook of Pediatric Intensive Care. 4th ed. Philadelphia, Pa: Lippincott Williams & Wilkins, a Wolters Kluwer business; 2008:441-465 Guyton A, Hall JE. The autonomic nervous system and the adrenal medulla. In: Textbook of Medical Physiology. 11th ed. Philadelphia, Pa: Elsevier Saunders; 2006:748-760 Hoffman RS. Cocaine. In: Flomenbaum NE, Goldfrank LR, Hoffman RS, Howland MA, Lewin NA, Nelson LS, eds. Goldfrank's Toxicologic Emergencies. 8th ed. New York, NY: McGraw-Hill Medical Publishing Division; 2006:1133-1146 Ziegler JW, Englander R, Fugate JH. Pharmacologic support of the circulation. In: Todres ID, Fugate JH, eds. Critical Care of Infants and Children. Boston, Mass: Little, Brown and Company; 1996:313-335 American Board of Pediatrics Content Specification(s): Know the mechanisms of norepinephrine inactivation, including neuronal uptake, and how this process is altered Know the agents that alter norepinephrine reuptake (eg, cocaine, imipramine, amphetamines) Know the signs of acute cocaine ingestion Understand the pathogenesis and toxic effects of cocaine Know the metabolism, excretion and pharmacokinetics of cocaine Figure: How cocaine increases sympathetic tone by increasing neurotransmitters in the synapse. Reprinted with permission from Marx JA, Hockberg RS, Walls RM, eds. Rosen's Emergency Medicine, Vol 2, p. 1995. Reprinted with permission. Copyright (c) 2010 by Mosby, Inc., an affiliate of Elsevier, Inc. Cocaine

Dec 2010 - Question 3 A 7-year-old boy is admitted to the hospital from the emergency department. He has a history of asthma and has been diagnosed as having an acute exacerbation. His chest radiograph reveals only hyperinflation. He has had no fever. His heart rate is 150/min, respiratory rate is 40/min, and blood pressure is 110/60 mm Hg. On auscultation, he has inspiratory and expiratory wheezes. He has substantial accessory muscle use and nasal flaring. His pulse oximetry reads 94% on 0.30 fraction of inspired oxygen. His peak expiratory flow is 80 L/min. You give him a loading dose of methylprednisolone (2 mg/kg) and then 0.5 mg/kg every 6 hours. You prescribe continuous albuterol at 25 mg/h (1 mg/kg/h) via inhalation and nebulized ipratropium. He is required to take nothing by mouth. Several hours later you are called back to the bedside by the nurse, who expresses concern that the patient's fraction of inspired oxygen has increased from 0.30 via face mask to 0.50, with arterial saturations decreasing into the high 80s. He has expiratory wheezes, but his inspiratory wheezes appear improved; however, he is experiencing some mild chest pain. A repeat chest radiograph is unchanged. You ask the child to repeat a peak flow maneuver, which is now 120 L/min. Of the following, the MOST appropriate next step is: A. begin chest physiotherapy to help clear secretions B. give the patient a dose of ibuprofen to ease his pain C. increase the fraction of inspired oxygen to improve his saturations D. increase the inhaled albuterol dose to 50 mg/h to improve airflow E. increase the steroid dose to diminish airways inflammation

C The pathophysiology of asthma, the most common cause of medical emergencies in children, includes airway inflammation, reversible or partially reversible bronchoconstriction, and increased reactivity to a variety of stimuli. The inflammation results in mucosal edema and increased mucous secretions. Patients experience increases in work of breathing due to the increase in airway resistance, which is evidenced by accessory muscle use. When severe, episodes may result in respiratory failure, respiratory arrest, or altered mental status. The hypoxemia resulting from asthma is due to mismatch in the ventilation-perfusion ratio (VA/Q) with a bimodal distribution of pulmonary blood flow with normal units and substantial areas of low VA/Q. VA/Q mismatch occurs across the spectrum of asthmatic patients from those who have severe exacerbations to those in clinical remission. However, asthmatic patients generally do not experience intrapulmonary shunting (VA/Q = 0) probably because collateral ventilation provides some ventilation to severely affected bronchioles. Arterial saturations may be preserved by increases in cardiac output, which helps to raise the mixed venous oxygen saturation. In addition, airway plugging and obstruction may lead to regional alveolar hyperinflation associated with reduced perfusion, which increases pulmonary dead space. Patients often maintain normocarbia by increasing their respiratory rate and minute ventilation. However, an important issue for those caring for asthmatic patients is that almost no correlation exists between measurements of airways obstruction and VA/Q inequalities. These data suggest that spirometric changes predominantly reflect smooth muscle constriction in the larger and medium airways, whereas VA/Q mismatch occurs in the distal smaller airways. Thus, in the patient described in the vignette, his low peak flow on presentation indicates obvious airways obstruction. However, when he is reexamined his airflow has improved, which is evidenced by the increase in his peak flow measurement. Albuterol, which is a β2-agonist, produces smooth muscle relaxation and improved airflow. Although the intent in using β-agonists is to produce bronchodilation, albuterol influences the vascular smooth muscle and may decrease hypoxic pulmonary vasoconstriction (HPV). Because the patient shows improving airflow, it is not necessary to increase the albuterol dose. Often asthmatic patients have increased mucous production with migrating atelectasis. This may be found on a chest radiograph often as platelike atelectasis. This child has no atelectasis apparent on chest radiograph, and chest physiotherapy is unlikely to be of significant assistance in treating his hypoxemia. Because airway inflammation is a primary abnormality of asthma, corticosteroid therapy must be a first-line therapy in the treatment of asthma. Again, improvement in airflow indicators (peak flow and auscultation) suggests that this child's inflammation has improved, so an increase in the corticosteroid therapy is not indicated and will not induce an improvement in the boy's arterial oxygenation. Nonsteroidal anti-inflammatory drugs are contraindicated in asthma. Other medications that are contraindicated in asthma include aspirin, β-blockers (eg, propranolol), and angiotensin-converting enzyme inhibitors (eg, captopril). Furthermore, patients treated with high doses of β-agonist drugs may have evidence of cardiac arrhythmias and may have enzymatic evidence of myocardial ischemia. Consequently, in a child who experiences chest pain, a 12-lead electrocardiogram may be in order. Decreasing arterial saturations should prompt an increase in the inspired oxygen concentrations; high levels of alveolar oxygen are also potent pulmonary vasodilators. Such high inspired oxygen may decrease HPV in some portions of the lung. Furthermore, the high inspired oxygen may place areas of low VA/Q at risk for atelectasis. However, with improving air flow, these abnormalities will become less significant. References: Manser R, Reid D, Abramson MJ. Corticosteroids for acute severe asthma in hospitalized patients. Cochrane Database Syst Rev. 2001;(1):CD001740. Abstract accessed September 2010 at: http://www.ncbi.nlm.nih.gov/pubmed/11279726. Rotta AT. Asthma in pediatric critical care patients. In: Fuhrman BP, Zimmerman J, eds. Pediatric Critical Care. 3rd ed. Philadelphia, PA: Mosby Elsevier; 2006:588-607. Vaughn DJ, Brogan TV. Ventilation-perfusion mismatch. In: Fuhrman B, Zimmerman JJ. Textbook of Pediatric Critical Care. 3rd ed. Philadelphia, PA: Mosby Elsevier; 2006:536-542. American Board of Pediatrics Content Specification(s): Understand the pathogenesis of the mechanical and gas exchange abnormalities associated with asthma Understand the pathophysiology of disorders characterized by bronchial hyperreactivity Know the treatment of disorders characterized by bronchial hyperreactivity Know which drugs are contraindicated (relative and absolute) in a patient with asthma

Dec 2010 - Question 7 A 12-year-old boy was a passenger in a motor vehicle that was hit by another car. He sustained an isolated head injury. The Glasgow Coma Scale score in the emergency department was 6, but computed tomography of the head revealed no evidence of intracranial hemorrhage. In anticipation of later brain swelling and elevated intracranial pressure (ICP), an intraventricular pressure monitor was placed with its tip in the left lateral ventricle. A right radial artery catheter was also placed. The head of the bed was elevated, placing the tragus of the ear (used as a marker for the foramen of Monro) 20 cm above the level of the heart. The neurosurgeon wants to accurately measure ICP, arterial blood pressure (BP), and cerebral perfusion pressure (CPP). He places the ICP transducer on a pole and zeroes it to atmosphere at the level of the tragus of the ear. You have placed the arterial transducer on the bed and zeroed it at the level of the heart. The neurosurgeon instructs you to calculate the CPP as mean BP minus mean ICP. You know that the density of mercury is 13.6 times that of water. Which of the following adjustments will BEST meet the neurosurgeon's goals? A. divide the calculated CPP by 1.36 B. position the radial artery beside the ear C. subtract 20/1.36 from the calculated CPP D. zero both transducers at the level of the heart E. zero both transducers at the level of the tragus

C To accurately assess the pressure driving blood flow through the brain, one must know both the pressure driving flow into the brain and the pressure opposing cerebral perfusion. To that end, one must know the pressure in the arteries at the same vertical height as the measure of back pressure, that is, one must know carotid artery pressure not radial artery pressure. In the supine patient, when an arterial pressure transducer is zeroed at the level of the heart, transducer measurements approximate the pressures in both the aorta and carotid arteries. When the head of the bed is elevated, a true difference develops between carotid and aortic pressures. This is so because aortic pressure cannot generate forward flow against a greater back pressure. Were the sum of carotid artery pressure plus gravitational pressure exerted by the column of blood between the aorta and carotid artery greater than aortic pressure, blood would flow backward from the head to the aortic arch. So true carotid artery pressure cannot exceed aortic pressure minus fluid column back pressure. It is not convenient to directly measure carotid pressure. Instead, in the intensive care unit, we measure true systemic arterial pressure (zeroed to atmosphere at the level of the heart) and subtract fluid column back pressure. The correction for vertical height (in millimeters of mercury) is the measured elevation of the head (in this case 20 cm) corrected for the density of mercury: 20 cm H2O/1.36 cm H2O/mm Hg The back pressure to cerebral blood flow is the intracranial pressure (ICP). To accurately measure the ICP, the ICP transducer must be zeroed at the level of the head. One can then estimate the cerebral perfusion pressure (CPP) from the blood pressure (BP) zeroed at the heart and the ICP zeroed at the head: CPP = (mean BPm m Hg - mean ICPm m Hg) - elevationcm /1.36 It has been shown that elevation of the head of the bed for patients with brain trauma reduces both CPP and ICP. If CPP is not corrected for elevation of the head, elevation of the head appears to increase the CPP. It remains unclear whether elevation of the head actually is or is not beneficial in this clinical setting. The alternative responses posed in the question would not allow all 3 measures to be determined accurately. Dividing the difference between transduced pressure readings by 1.36 does not provide a correction for difference in height and yields a number with meaningless units (mm Hg2/cm H2O). Positioning the hand beside the ear does not correct for the difference in height because that maneuver does not change the zero level of the arterial transducer. Zeroing both transducers at the level of the heart would yield an erroneous value for ICP, and zeroing both at the level of the head would yield an erroneous value for BP. References: Feldman Z, Kanter MJ, Robertson CS, et al. Effect of head elevation on intracranial pressure, cerebral perfusion pressure, and cerebral blood flow in head-injured patients. J Neurosurg. 1992;76:207-211. DOI: 10.3171/jns.1992.76.2.0207. Abstract accessed September 2010 at: http://www.ncbi.nlm.nih.gov/pubmed/1730949. Ng I, Lim J, Wong HB. Effects of head posture on cerebral hemodynamics: its influences on intracranial pressure, cerebral perfusion pressure, and cerebral oxygenation. Neurosurgery. 2004;54:593-598. Abstract accessed September 2010 at: http://www.ncbi.nlm.nih.gov/pubmed/15028132. Rosner MJ, Coley IB. Cerebral perfusion pressure, intracranial pressure and head elevation. J Neurosurg. 1986;65:636-641. DOI: 10.3171/jns.1986.65.5.0636. Abstract accessed September 2010 at: http://www.ncbi.nlm.nih.gov/pubmed/3772451. American Board of Pediatrics Content Specification(s): Understand invasive techniques for measurement of vascular pressure

Jan 2010 - Question 2 An 11-month-old boy who has short gut syndrome, including chronic liver failure, developed acute respiratory distress syndrome, precipitated by an episode of catheter-related sepsis, that now has resolved. He has been sedated, paralyzed, and mechanically ventilated for the past 10 days. He is slowly weaning from ventilatory support. He has a cuffed endotracheal tube in place and currently is receiving an Fio2 of 0.6, rate of 20, peak inspiratory pressure of 28 cm H2O, and positive end-expiratory pressure of 12 cm H2O. His most recent arterial blood gas reveals a pH of 7.38, Paco2 of 55 mm Hg, Pao2 of 65 mm Hg, and base excess of 6 mEq/L. He weighs 7 kg (<5th percentile). He has had multiple episodes of hyperammonemia, but this is not currently a problem. He is dependent on parenteral nutrition, and his current nutrition includes dextrose 25% with amino acids (10% solution, 2 g/kg per day) running at 20 mL/hour and a 20% lipid emulsion running at 2.5 mL/hour 24 hours per day. Results of his morning laboratory studies include the following: Glucose 210 mg/dL (11.7 mmol/L) Sodium 138 mEq/L (138 mmol/L) Potassium 3.8 mEq/L (3.8 mmol/L) Chloride 100 mEq/L (100 mmol/L) Bicarbonate 22 mEq/L (22 mmol/L) Blood urea nitrogen 10 mg/dL (3.6 mmol/L) Creatinine 0.4 mg/dL (35.4 mcmol/L) Calcium 8.8 mg/dL (2.2 mmol/L) Albumin 2.8 mg/dL Triglycerides 125 mg/dL (1.4 mmol/L) Indirect calorimetry obtained yesterday indicates that the boy's resting energy expenditure is 50 kcal/kg per day, and his respiratory quotient is 1.12. Of the following, the MOST appropriate modification of his parenteral nutrition is to A. continue the current regimen and initiate an insulin infusion to maintain serum glucose between 80 and 150 mg/dL (4.5 and 8.3 mmol/L) B. decrease the dextrose concentration to 20% and lipid emulsion to 1.5 mL/hour C. decrease the dextrose concentration to 15% and maintain amino acids and lipid emulsion infusions at the current rate D. decrease the dextrose infusion rate to 12 mL/hour and maintain current amino acid delivery (2 g/kg per day) and lipid infusion at 2.5 mL/hour E. increase the amino acid infusion to 4 g/kg per day and initiate an insulin infusion to maintain serum glucose between 80 and 150 mg/dL (4.5 and 8.3 mmol/L)

C infant described in the vignette is poorly nourished, which is common in ICU patients, especially those who have underlying chronic diseases. His weight is well below the 5th percentile and serum albumin is low, although the latter likely is markedly influenced by his chronic liver disease. Many critically ill infants and children have increased caloric requirements, but others demonstrate decreased energy consumption. When available, indirect calorimetry currently is the best tool to determine the needs of individual ICU patients. This patients measured energy expenditure is low at 50 kcal/kg per day. Growth often is suspended in critical illness, and motor activity is diminished by bedrest and even more in patients who are sedated and paralyzed, particularly in those who do not have active inflammatory processes, such as sepsis, burns, or trauma. The appropriate distribution of calories between carbohydrate and fat is not well established in pediatrics, but providing 30% to 45% as fat commonly is recommended. Some authors recommend providing a considerably greater percentage as fat, especially for patients who have severe lung disease, to avoid the burden of greater carbon dioxide production. The boy currently is receiving 528 nonprotein kcal/day (75 kcal/kg per day), of which 408 kcal (77%) are from dextrose and 120 kcal (23%) are from fat (Table). Table: Caloic Calclaion Deail Carbohydrate and Protein: 3.4 kcal/g Fat: 9 kcal/g; 20% intravenous lipid infusion: 2 kcal/mL Dextrose 25% at 20 mL/hr = 3.4 kcal/g x 25% 100 g/100 mL x 20 mL/hr x 24 hr/day = 408 kcal/day 20% intravenous lipid infusion at 2.5 mL/hr = 2 kcal/mL x 2.5 mL/hr x 24 hr/day = 120 kcal/day This is significantly more than required, and the respiratory quotient greater than 1.0, obtained from indirect calorimetry, provides evidence that he is converting carbohydrate to fat. The current dextrose infusion provides 12 mg/kg per minute, which is greater than required by many, if not most, patients. It is likely that the high carbohydrate infusion rate is contributing to the hyperglycemia. Decreasing the dextrose concentration to 15% and maintaining the current amino acid

April 2010 - Question 8 A 17-year-old boy who has known human immunodeficiency virus (HIV) infection presents to the emergency department with a 1-week history of headache, low-grade fever, and general malaise. He exhibits waxing and waning mental status and is transferred to the PICU. His CD4 count is 60 cells/mL on admission. Physical examination reveals a thin, ill-appearing male. He has nonfocal neurologic examination findings and no papilledema. Noncontrast head computed tomography scan shows mild hydrocephalus and no focal lesions. Lumbar puncture reveals an opening pressure of 25 cm H2O, white blood cell count of 18/mm3, glucose of 30 mg/dL, and protein of 100 mg/dL. India ink preparation is positive, and you send appropriate specimens for culture and for antigen studies Of the following, the opportunistic pathogen responsible for this child's illness would be MOST likely also to cause disease in a patient who has A. Chediak-Higashi syndrome B. complement deficiency C. splenectomy after traumatic injury D. steroid-dependent systemic lupus erythematosus E. X-linked agammaglobulinemia

D The boy described in the vignette has HIV disease, a low CD4 count, and clinical and laboratory findings typical for Cryptococcus neoformans meningoencephalitis. Of the other immunosuppressive conditions listed, a patient who has systemic lupus erythematosus and is treated chronically with corticosteroids would be at the highest risk for C neoformans central nervous system (CNS) infection. Host defenses normally are very effective in excluding fungi from the CNS, but certain conditions lead to their failure. HIV affects CD4+ T cells, B cells, mononuclear phagocytes, and polymorphonuclear (PMN) leukocytes quantitatively and functionally. Glucocorticoids also exert a broad range of effects on circulating lymphocytes. They produce lymphopenia within 24 hours, and circulating leukocytes increase because band forms are released from the bone marrow. PMN chemotaxis and migration are diminished. Glucocorticoids block tissue macrophage responses to migration inhibitory factor and inhibit phagocytosis and presentation of antigen. Steroids also inhibit interleukin-1 release from macrophages and interleukin-2 synthesis and release from T cells. The body's defense mechanisms consist of surface barriers (skin, enzymes, mucus) as well as innate (natural) and acquired (adaptive) immune responses. Innate immunity is activated immediately upon exposure to an infecting agent, rapidly controlling replication and allowing the requisite 3 to 5 days for the adaptive/acquired component to clone sufficient T and B cells to respond more specifically. Major components of the innate immune system are physical barriers, phagocytic cells, eosinophils, natural killer (NK) cells, endothelial cells, and various noncellular mechanisms and molecules. Adaptive/acquired immunity is stimulated specifically by exposure to pathogens and includes antibodies (produced by B lymphocytes), lymphocytes, antigen-presenting cells, and complement. A key feature of the acquired immune response is to "remember" pathogen exposure, which provides a mechanism to mount a robust and specific response against pathogens after repeat exposure. The complement cascade is important for producing inflammation and leukocytosis and in recruiting leukocytes to sites of infection by producing chemoattractants. Patients who have inherited complement deficiencies are predisposed to frequent and recurrent infections with Streptococcus pneumoniae, Haemophilus influenzae, and especially Neisseria meningitidis and Neisseria gonorrhoeae but not fungal infections. The risk of meningococcal infection is increased several thousandfold in patients who have inherited complement deficiencies and most often develops in those deficient in C3 and in late components (C5 through C8). Paradoxically, the disease is usually milder and mortality is reduced in patients who have complement deficiency, suggesting that the host response may be responsible for disease severity in patients who have normal immune systems and attenuated in those who have complement deficiency. Although knockout mice with complement deficiencies have been demonstrated to have higher mortality from fungal infections, including cryptococcal infections, and although some investigators have questioned a potential role for deficiency of mannose binding lectin in human cryptococcosis, patients who have inherited complement deficiencies are not known to have higher rates of cryptococcal meningitis. T lymphocytes comprise the major portion of cellular acquired immunity and consist of helper cells (CD4), cytolytic cells, NK cells, and regulatory T cells (CD8). When stimulated by antigens, T cells secrete a variety of cytokines that cause activation, differentiation, and proliferation of other cells of the acquired immune system (B cells, T cells, and macrophages). Cytotoxic T cells lyse cells bearing membrane-bound foreign antigens and serve as a primary mechanism for elimination of viruses and other intracellular pathogens. Regulatory (CD8) cells (aka suppressor cells) limit the adaptive and innate immune response to allow the antigen-infected host to return to homeostasis after pathogen eradication. A decline in the number of CD4 cells, with a predominance of CD8 cells, is responsible for the increased susceptibility to infection in patients who have acquired immune deficiency syndrome (AIDS). Despite the cytotoxicity of CD8 cells, immunity is reduced without adequate numbers of CD4 cells. In addition to AIDS and immune suppression with corticosteroids, other underlying medical conditions that suppress normal immune response, such as penetrating head injury, diabetes, pregnancy, malignancies, near-drowning, and iron chelation therapy, favor invasion by fungi. C neoformans is an encapsulated yeast that is the most common cause of fungal meningitis. Rates of cryptococcal meningitis in patients who are not infected with HIV approaches 1 per 100,000 population, a rate similar to that of meningococcal meningitis. Most C neoformans infections occur through inhalation of small yeast or basidiospore forms from the environment. Once inhaled, these organisms or spores reach the lung parenchyma through the airways. The yeast is deposited in the alveoli, and the primary response is via lung macrophages. The yeast is found in bird excreta, soil, and animals. After ingestion by the host, the fungi develop large polysaccharide capsules that strongly resist phagocytosis and release polysaccharide antigen that affects host immunity. A primary lung-lymph node complex forms, which usually limits spread of the organism. Cryptococci can remain dormant in the lung or lymph nodes for long periods following exposure and can be disseminated during the primary infection or reactivation years later, when host immune responses become weakened. If distant infection occurs, the most commonly infected site is the CNS. The predilection of the encapsulated yeast for the subarachnoid space suggests the existence of a receptor on CNS cells for a ligand on the yeast, although this has not yet been proven. Host resistance to cryptococcosis depends primarily on intact cell-mediated immunity. It appears that CD4 response is important to eradication and, thus, most cases of cryptococcal meningitis occur in patients who have diseases that affect this response, such as AIDS, reticuloendothelial malignancies, sarcoidosis, organ transplantation, collagen-vascular diseases, and idiopathic CD4 lymphocytopenia, as well as in patients receiving corticosteroids. The chance of cryptococcal meningitis increases dramatically as the CD4 cell count decreases below 100 cells/mL. Anticryptococcal activity of NK cells also is impaired in patients who have HIV infection. Humoral immunity also plays a role in host defenses against cryptococcosis, with immunoglobulin G and C3b binding to and within the capsule to enhance phagocytosis. However, the rarity of cryptococcal meningitis in patients who have either congenital or acquired deficiencies in antibody or complement production argues that humoral immunity is less important than cellular immunity. The CNS findings seen in patients who have AIDS or are receiving high-dose steroids are similar. There are low cerebrospinal fluid (CSF) cell counts, large burden of organisms, high antigen titers, and high mortality. Many patients who have HIV and cryptococcal meningitis present with a CSF cell count of fewer than 20 leukocytes/mm3 and some with almost no inflammatory cells. This lack of inflammatory response indicates both defective immunity and poor prognosis. Diagnosis is confirmed by demonstrating cryptococcal antigen in the CSF, which is nearly 100% sensitive and specific. Other diagnostic tests include India ink stain (60% to 80% sensitive), fungal culture (95% sensitive), and serum cryptococcal antigen (95% sensitive). Approximately 25% to 30% of patients who have AIDS and cryptococcal meningoencephalitis have normal CSF profiles. Almost 70% of patients who have cryptococcal meningitis have elevated opening pressures greater than 20 cm H2O on the initial lumbar puncture. Neuroimaging should be considered in any patient exhibiting signs and symptoms of elevated intracranial pressure. Computed tomography scan may detect hydrocephalus. Mass lesions are rare, and if present, other diagnoses should be entertained, including toxoplasmosis, lymphoma, tuberculosis, and syphilis. Among the predictors of death are abnormal mental status, CSF antigen titer greater than 1:1,024, and CSF white blood cell count less than 20/mm3. The combination of amphotericin B and flucytosine is used as induction therapy for 2 weeks, followed by fluconazole for a minimum of 10 weeks. Lumbar puncture should be performed at 2 weeks and shows sterile CSF in approximately 70% of patients. Those who have positive results at 2 weeks require a longer treatment course. Patients who have AIDS require lifelong maintenance with an anticryptococcal drug. The incidence of X-linked agammaglobulinemia is 1 in 150,000 and is characterized by extremely low concentrations of all immunoglobulin subclasses and an inability to make specific antibodies against either infections or immunizations. Precursor B cells are present in the bone marrow, but circulating B cells are absent, as are plasma cells from the lymph nodes. Patients present with recurrent pyogenic infections, and physical examination demonstrates a paucity of tonsillar tissue. Children have severe invasive infections, including pneumonia, meningitis, or sepsis caused by Pseudomonas, H influenzae type b, or S pneumoniae. They also are at risk for recurrent enteroviral infections, especially meningoencephalitis. Many patients develop chronic lung disease. Chediak-Higashi syndrome is a rare autosomal disease that involves abnormal packaging of the PMN lysosomes. These cells show decreased degranulation, poor chemotactic response, and delayed microbial killing. Recurrent infections of the respiratory tract and skin are common and Staphylococcus aureus frequently is isolated. Patients also have partial albinism, photophobia, and nystagmus. The frequency of sepsis is 60 times greater in patients who have had a splenectomy. The spleen has an active role in phagocytosis; is a major source of T lymphocytes; and produces immunoglobulin M antibodies, complement, opsonins, and tuftsin. S pneumoniae is the most common cause of sepsis in those who have undergone splenectomy. Despite prompt diagnosis and treatment, pneumococcal sepsis is associated with a very high mortality rate in splenectomized patients. Eighty percent of postsplenectomy infections are caused by bacteria that have capsular polysaccharides: S pneumoniae, H influenzae, and N meningitidis. Patients should receive multivalent pneumococcal polysaccharide vaccine, vaccination against H influenzae type b, and N meningitidis, and antibiotic prophylaxis with penicillin V administered twice daily. References: Hughes WT. Cryptococcosis. In: Feigin RD, Cherry JD, Demmler GJ, Kaplan SL, eds. Textbook of Pediatric Infectious Diseases. 5th ed. Philadelphia, Pa: Saunders; 2004:2602- 2606 Perfect JR. Fungal meningitis. In: Scheld WM, Whitely RJ, Marra CM, eds. Infections of the Central Nervous System. 3rd ed. Philadelphia, Pa: Lippincott Williams & Wilkins; 2004:691- 712 Singhi PD, Singhi SC, Newton CRJC, Simon J. Central nervous system infections. In: Nichols DG, ed. Roger's Textbook of Pediatric Intensive Care. 4th ed. Philadelphia, Pa: Lippincott Williams & Wilkins, a Wolters Kluwer business; 2008:1353-1399 American Board of Pediatrics Content Specification(s): Recognize the signs and treatment of cryptococcal central nervous system infections Recognize the clinical signs and symptoms and laboratory findings characteristic of fungal central nervous system infections Know the factors associated with an increased risk of fungal central nervous system infection

August 2010 - Question 8 A 4-year-old child with attention-deficit/hyperactivity disorder was in the care of his grandparents and was found extremely lethargic by his grandmother, who called emergency medical services. The child has no significant medical history and is taking no medications. The grandfather wears a "patch of some medication" to control his blood pressure, which he discarded in the wastebasket earlier in the day. The child was brought to the emergency department, where you are consulted on the case. Vital signs include the following: heart rate, 60/min; blood pressure, 76/40 mm Hg; and respiratory rate, 10/min. Pulse oximetry reveals 97% saturation on 8-L nasal canulae. No bruises or rashes are noted, and capillary refill is approximately 2 to 3 seconds. The child awakens briefly while an intravenous line is placed. The patient is given 20 mL/kg of 0.9% sodium chloride. Administration of which of the following agents may improve this child's condition? A. atropine B. dopamine C. glucagon D. naloxone E. pyridostigmine

D The patient described in the vignette demonstrates signs of probable unintentional drug overdose, including bradycardia, hypotension, bradypnea, and altered mental status. These findings suggest that an opioid or sedative-hypnotic may be the cause. This, coupled with the fact that the grandfather is receiving a transdermal patch for blood pressure, supports clonidine as a likely cause of the findings. Although there is no specific antidote for clonidine poisoning, of the choices listed, naloxone appears to be the pharmacologic agent that could possibly improve both the mental and respiratory depression seen in this patient. Several studies have demonstrated increased arousal, respiratory effort, heart rate, and blood pressure after administration of naloxone. Intravenous dosing has ranged from 0.05 to 0.1 mg/kg to a maximum of 10 mg. This treatment should not delay intubation and mechanical ventilation in patients who do not immediately respond. Clonidine was initially trialed as a topical nasal decongestant because of its α2-adrenergic agonistic effects; however, hypotension was a common adverse effect, which led to clonidine being useful in treating hypertension. Clonidine exerts its blood pressure-lowering effect by reducing the sympathetic output in the central nervous system by stimulating presynaptic α2- adrenergic receptors in the brain. This central α2-receptor agonist enhances the activity of inhibitory neurons in the vasoregulatory regions of the central nervous system (CNS) (nucleus tractus solaris in the medulla), resulting in a decrease in norepinephrine release. Although overdoses of clonidine are relatively uncommon, this agent may cause significant morbidity and rare mortality in pediatric patients in whom "one pill can kill." The fact that the patch was discarded in a wastebasket gave the child access to an object that probably fascinated him and allowed the child to apply, taste, or ingest the patch. Clonidine is indicated in the treatment of hypertension, anesthetic premedication, spinal anesthesia, opioid detoxification, alcohol withdrawal, attention-deficit/hyperactivity disorder, refractory conduct disorder, and Tourette syndrome; aids in smoking cessation; and decreases postmenopausal hot flashes. Clonidine is well absorbed from the gastrointestinal tract (approximately 75%) with an onset of action within 30 to 60 minutes. The peak concentration occurs at 2 to 3 hours and may last as long as 8 hours. Clonidine has 20% to 40% protein binding and an apparent volume of distribution of 3.2 to 5.6 L/kg. Most clonidine is excreted unchanged via the kidneys. It is available in both oral and patch forms, and the patch allows for slow delivery of the medication during a prolonged period (1 week). Each patch contains more drug than what is delivered, so, for example, a patch that delivers 0.1 mg/d of clonidine contains 2.5 mg total, and the higherdose product of 0.3 mg/d contains 7.5 mg of clonidine. In 1 week of use, approximately 35% to 70% of the drug may remain in the patch. Also, if one punctures the outer membrane or backing of the drug reservoir, it will allow for a more significant amount of drug to be released. It is important that patients be warned of proper disposal techniques when receiving these transdermal medications. The signs and symptoms of all centrally acting antihypertensive medications are similar. The CNS and cardiovascular system are affected most commonly. Signs and symptoms include bradycardia, hypotension, and occasionally hypothermia. CNS effects can range from mild lethargy as seen in the patient described in the vignette to coma. Obtunded patients may hypoventilate and become acidotic and hypoxic. Other CNS effects include hypotonia, hyporeflexia, irritability, and miotic pupils. Hypothermia is thought to occur secondary to α-adrenergic effects within the thermoregulatory center. Bradycardia may occur in up to 50% of patients. This patient appears to be hemodynamically stable; thus, there is no need for atropine or dopamine. Hypotension is the major cardiovascular finding seen in central antihypertensive toxicity. The mainstay of therapy is volume resuscitation (up to 60-100 mL/kg), and in the rare case of fluid unresponsiveness dopamine could be considered. Paradoxical hypertension has been encountered in massive overdose presumably from stimulation of peripheral α1-receptors. Management of patients with clonidine ingestion always involves the ABC's of resuscitation. Once the patient is stabilized, a serum glucose measurement should be determined in all patients with altered mental status. One must remove any transdermal patches if they are present. If it is an intentional overdose or suicide attempt in an older patient, other drug levels should be obtained, such as serum aspirin and acetaminophen levels. There are no specific hematologic or electrolyte abnormalities associated with clonidine overdose. A 12-lead electrocardiogram should be performed, and patients receive continuous cardiac and respiratory monitoring. Arterial blood gas evaluation and pulse oximetry may help to assess the adequacy of ventilation. Clonidine binds well to activated charcoal, which may serve as the primary route of decontamination for patients presenting within 1 to 2 hours of ingestion. If patients present more than 2 hours after ingestion, the likelihood of activated charcoal preventing absorption at this point is low. Gastric emptying by lavage or induced emesis is not recommended in patients with altered mental status because the risk of aspiration is great. In patients who have swallowed a clonidine patch, activated charcoal should be administered followed by whole bowel irrigation with 500 mL/h to 1 L/h of polyethylene glycol. There is no role for administration of glucagon in clonidine ingestions (it is used for b-blocker overdoses). Pyridostigmine is used for symptomatic treatment of myasthenia gravis, for reversal of effects of nondepolarizing neuromuscular blockers, and as a pretreatment for nerve gas exposures by the military; thus, there is no role for its use in clonidine poisoning. With appropriate treatment, almost all clonidine poisoned patients will have a good outcome. References: Spiller HA, Klein-Schwartz W, Colvin JM, et al. Toxic clonidine ingestion in children. J Pediatr. 2005;146:263-266. DOI: 10.1016/j.jpeds.2005.09.027. Abstract accessed April 2010 at: http://www.ncbi.nlm.nih.gov/pubmed/15689921. Taketomo CK, Hodding JH, Kraus DM. Pediatric Dosage Handbook. 16th ed. Hudson, OH: Lexicomp Inc; 2009:307-311. American Board of Pediatrics Content Specification(s): Recognize the clinical and laboratory manifestations of clonidine intoxication Know the pathophysiology and toxic effects of clonidine intoxication Know the normal metabolism, excretion, and pharmacokinetics of clonidine Plan the diagnostic assessment and evaluation of suspected clonidine intoxication Plan appropriate therapy for a child with clonidine intoxication

Jan 2010 - Question 4 An otherwise healthy 13-year-old boy presents with a 2-day history of fever (maximum temperature of 39.0°C), cough, and myalgia and a 1-day history of tachypnea and vomiting. In the emergency department, his rapid screening test for influenza yields negative results, but your hospital has been seeing large numbers of patients with novel H1N1 influenza recently. He is admitted to the PICU due to respiratory distress and hypoxemia. During the first 6 hours of admission, his oxygen saturations decrease, and he progresses to respiratory failure. Of the following, the MOST appropriate personal protective precautions to take during endotracheal intubation of this boy are A. hand hygiene and gloves B. hand hygiene, gloves, and isolation gown C. hand hygiene, gloves, and N95 respirator D. hand hygiene, gloves, N95 respirator, and isolation gown E. hand hygiene, gloves, surgical mask, and isolation gown

D The presentation of the patient described in the vignette is suggestive of influenza, for which droplet precautions are recommended to prevent spread of infection. Aerosolized small droplets created during procedures such as endotracheal intubation, suctioning, or bronchoscopy present a significant risk of spread to those in the vicinity of the procedure. Therefore, the Red Book suggests the use of an N95 (or higher) respirator for personnel involved in aerosol-generating procedures (bronchoscopy, intubation, nebulizer treatments) when patients are suspected of having influenza, severe acute respiratory syndrome (SARS), or hemorrhagic fever. Standard precautions are intended to prevent the spread of infection following contact with any secretion, excretion, body fluid, or nonintact skin. The precautions are based on the premise that the aforementioned body fluids may contain transmissible infectious agents. Standard precautions apply to all patients, regardless of suspected or confirmed infection status, in any setting in which health care is delivered. The most important precaution is hand hygiene; gloves, gown, mask, and eye protection may be warranted, depending on the anticipated exposure. Accordingly, the most appropriate precautions for the boy in the vignette involve hand hygiene, gloves, gown, and an N95 respirator. Equipment and items in the patients environment are likely to be contaminated with infectious body fluids and must be handled in a manner to prevent transmission (eg, wear gloves for direct contact, contain heavily soiled equipment, properly clean and disinfect or sterilize reusable equipment before use on another patient). Required precautions are determined by the nature of the interaction between the health-care worker and the patient and the extent of anticipated blood, body fluid, or pathogen exposure. Beyond standard precautions, a series of safety measures are recommended to prevent the spread of infection based on transmission patterns that have been identified for specific microorganisms (Table 1). Table 1: Clinical Sndromes or Conditions Warranting Precautions in Addition to Standard Precautions to Prevent Transmission of Epidemiologicall Important Pathogens Pending Confirmation of Diagnosisa Clinical Sndrome or Conditionb Potential Pathogensc Empiric Precautionsd Diarrhea Acute diarrhea with a likely infectious cause Enteric pathogense Contact Diarrhea in patient with a history of recent antimicrobial use Clostridium difficile Contact; use soap and water for handwashing Meningitis Neisseria meningitidis Droplet Enteroviruses Contact Rash or exanthems, generalized, cause unknown Petechial or ecchymotic with fever N meningitidis, Haemophilus influenae Droplet Hemorrhagic fever viruses Add Contact plus face/eye protection Vesicular Varicella-zoster virus Airborne and Contact Maculopapular with coryza and fever Measles virus Airborne Respirator tract infections Pulmonary cavitary disease Mcobacterium tuberculosis Airborne Paroxysmal or severe persistent cough during periods of pertussis activity in the community Bordetella pertussis Droplet Viral infections, particularly bronchiolitis and croup, in infants and young children Respiratory viral pathogens Contact plus Droplet until adenovirus, pneumonia, rhinovirus, and influenza virus excludedf Risk of multidrug-resistant microorganismsf History of infection or colonization with multidrug-resistant organisms Resistant bacteria Contact Skin, wound, or urinary tract infection in a patient with a recent stay in a hospital or chronic care facility Resistant bacteria Contact until resistant organism is excluded by cultures Skin or wound infection Abscess or draining wound that cannot be covered Staphlococcus aureus, group A Streptococcus Contact a Infection-control professionals are encouraged to modify or adapt this table according to local conditions. To ensure that appropriate empiric precautions are implemented, hospitals must have systems in place to evaluate patients routinely according to these criteria as part of their preadmission and admission care. bPatients with the syndromes or conditions listed may have atypical signs or symptoms (eg, pertussis in neonates, absence of paroxysmal or severe cough in adults). The clinicians index of suspicion should be guided by the prevalence of specific conditions in the community and clinical judgment. cThe organisms listed in this column are not intended to represent the complete or even most likely diagnoses but, rather, possible causative agents that require additional precautions beyond Standard Precautions until a causative agent can be excluded. dDuration of isolation varies by agent and the antimicrobial treatment administered. eThese pathogens include Shiga toxin-producing Escherichia coli including E coli O157:H7, Shigella organisms, Salmonella organisms, Camplobacter organisms, hepatitis A virus, enteric viruses including rotavirus, Crptosporidium organisms, and Giardia organisms. Use masks when cleaning vomitus or stool during norovirus outbreak. fResistant bacteria judged by the infection control program on the basis of current state, regional, or national recommendations to be of special clinical or epidemiologic significance. Reprinted with permission from Red Book Online, 2009. The three categories of transmission-based precautions include: contact precautions, droplet precautions, and airborne precautions. Transmission-based precautions are used in addition to standard precautions when methods of transmission are not completely interrupted using standard precautions alone (Table 2). Some diseases have demonstrated multiple routes of transmission (eg, SARS) that require multiple levels of precaution. Table 2: Transmission-Based Precautions for Hospitalied Patientsa Categor of Precautions Single-Patient Room Respirator Tract/Mucous Membrane Protection Gowns Gloves Airborne Yes, with negative air-pressure ventilation, 6-12 air exchanges per hour, ± HEPA filtration Respirators: N95 or higher level Nob Nob Droplet Yesc Surgical masksd Nob Nob Contact Yesc No Yes Yes HEPA indicates high-efficiency particulate air. aThese recommendations are in addition to those for Standard Precautions for all patients. bGowns and gloves may be required as a component of Standard Precautions (eg, for blood collection or during procedures likely to cause blood splashes or if there are skin lesions containing transmissible infectious agents). cPreferred. Cohorting of children infected with the same pathogen is acceptable if a single-patient room is not available, a distance of more than 3 feet between patients can be maintained, and precautions are observed between all contacts with different patients in the room. dMasks should be donned on entry into the room. Reprinted with permission from Red Book Online, 2009. Contact precautions are intended to prevent transmission of infectious agents by direct or indirect contact with the patient or the patient's environment. Contact precautions also apply when the presence of excessive wound drainage, fecal incontinence, or other discharges from the body suggest an increased potential for extensive environmental contamination or when patients are known to be colonized or infected with resistant organisms (eg, methicillin-resistant Staphlococcus aureus, multiple drug-resistant gramnegative bacilli). A single patient room is preferred for patients who require contact precautions. If a single patient room is not an option, cohorting or maintaining spatial separation of more than 3 feet between beds is advised to prevent sharing of items in the environment. Health-care personnel should wear gowns and gloves for all interactions that may involve contact with the patient or the patient's environment. Personnel should don personal protective equipment before entering a room and discard the equipment before exiting the patient room to contain pathogens. Droplet precautions are intended to prevent transmission of pathogens propelled short distances (usually <3 feet) by coughing, sneezing, or talking or spread by some procedures. Infectious agents that require droplet precautions include Bordetella pertussis, influenza virus, adenovirus, rhinovirus, Neisseria meningitidis, and group A Streptococcus (for the first 24 hours of antimicrobial therapy). The droplets are relatively large particles that do not remain suspended in air. Under normal patient care circumstances, a surgical mask in addition to gown, gloves, and hand hygiene is adequate for the care of a patient with influenza or other agents requiring droplet precautions. However, personnel performing special procedures are at greater risk due to proximity and the significant possibility that the procedure will stimulate cough that aerosolizes a greater number of particles and, therefore, should wear at least an N95 mask. Because the pathogens do not remain infectious over long distances, special air handling and ventilation are not required to prevent droplet transmission. As with contact precautions, a single patient room is preferred for patients who require droplet precautions. Cohorting and spatial separation are acceptable alternatives if a single patient room is not available. If droplet precautions are required, spatial separation should be augmented by a curtain between patients. Airborne precautions prevent transmission of small particles that can be suspended and transported by air currents over long distances. Microorganisms transmitted by airborne spread include rubeola virus (measles), varicella virus (chickenpox), and Mcobacterium tuberculosis. Particles can result from the evaporation of water in small droplets containing microorganisms or dust particles containing infectious organisms or spores. The suspended particles can be inhaled by people both in close proximity and at some distance because of the airborne broadcast. A single patient isolation room is required that has monitored negative pressure relative to the surrounding area, six to twelve air exchanges per hour (depending on the age of the construction), and air exhausted directly to the outside or recirculated through high-efficiency particulate air filtration before return. Susceptible health-care personnel should not enter the rooms of patients who have measles or varicella infections. If a susceptible health-care worker must enter the room, he or she must wear an N95 or higher mask. Those who have proven immunity to measles or varicella are not required to wear masks when in the room of a patient who has these diseases. Most children who have tuberculosis are not contagious and do not require isolation. When isolation is required, it can be discontinued once therapy is initiated. Isolation is only required for children who have cavitary pulmonary tuberculosis, positive sputum acidfast bacilli smears, laryngeal involvement, extensive pulmonary infections, or congenital tuberculosis undergoing procedures that involve the oropharyngeal airway. The major concern regarding infection control of children who have tuberculosis is limiting visitation of adult contacts, who may be the source case and have active infection. References: American Academy of Pediatrics. Isolation precautions. In: Pickering LK, Baker CJ, Kimberlin DW, Long SS, eds. Red Book: 2009 Report of the Committee on Infectious Diseases. 28th ed. Elk Grove Village, Ill: American Academy of Pediatrics; 2009:149-158 Siegel JD, Rhinehart E, Jackson M, Chiarello L; the Healthcare Infection Control Practices Advisory Committee. 2007 Guideline for Isolation Precautions: Preventing Transmission of Infectious Agents in Healthcare Settings. Atlanta, Ga: Centers for Disease Control and Prevention; 2007. Accessed October 2009 at: http://www.cdc.gov/ncidod/dhqp/pdf/guidelines/Isolation2007.pdf American Board of Pediatrics Content Specification(s): Understand the rationale for universal precaution isolation techniques Know the key procedures that characterize isolation protocols for varicella infections Know the key procedures that characterize isolation protocols for meningitis infections Know the key procedures that characterize isolation protocols for tuberculosis Know the key procedures that characterize isolation protocols for wound and skin infections Know the key procedures that characterize isolation protocols for respiratory infections

Dec 2010 - Question 6 A 3-year-old boy is transferred to the intensive care unit (ICU) from the inpatient department, where he was admitted 4 days ago for fever and respiratory distress. He was well until 1 week before admission, when he developed runny nose, cough, and low-grade fever. He was seen by the pediatrician, and amoxicillin was prescribed. His symptoms persisted and worsened. He was seen in the emergency department and admitted because of increasing difficulty in breathing. A chest radiograph showed left lower lobe consolidation, and ceftriaxone and vancomycin were prescribed. Blood and respiratory cultures yielded Streptococcus pneumoniae. On the day of transfer, he developed worsening respiratory distress with a respiratory rate of 80/min, grunting, and an oxygen saturation as measured by pulse oximetry of 92% on a fraction of inspired oxygen of 0.65 by face mask. Arterial blood gas measurement revealed the following: pH, 7.36; PCO2, 45 mm Hg; and PO2, 67 mm Hg. He was intubated and transferred to the ICU. His chest radiograph is shown in the Figure. A. autolysin B. hyaluronidase C. neuraminidases D. pneumolysin E. polysaccharide capsule

E Despite pneumococcal vaccination and antimicrobial therapy, pneumococcal disease continues to cause significant morbidity and mortality in children and adults. Pneumococcal cells are surrounded by a trilaminar lipopolysaccharide cytoplasmic membrane. The polysaccharide capsule covers the cell wall. The cell wall is composed of peptidoglycans, lipoteichoic acid, teichoic acid, and surface proteins. Streptococcus pneumoniae primarily causes disease in humans. Virulence of the organism is conferred by several factors. Although pneumococcal organisms have a large number of virulence factors, the capsule remains the most important determinant of virulence because of its ability to prevent phagocytosis of the organism by polymorphonuclear leukocytes and macrophages. The protection conferred by the capsule allows proliferation of the organism. Autolysins are enzymes with the ability to disrupt and to disintegrate the bacterial cell itself and are naturally produced by all peptidoglycan-containing bacteria, including S pneumonia. Their purpose appears to be hydrolysis of the peptidoglycan matrix to allow further bacterial colony growth and replication. Hyaluronidases are enzymes that facilitate pneumococcal invasion of the bloodstream. In a murine model designed to simulate human meningitis, inoculation of the nasal mucosa with pneumococcus with hyaluronidase resulted in pneumococcal meningitis in 50% of the study animals. Neuraminidases are enzymes that can cleave sialic acid residues from cell surfaces, exposing receptors for pneumococcal adhesins. Although pneumolysin is not the major factor in the virulence of pneumococcus, it has some key functions that allow the organism to bind to membrane cholesterol and cause membrane damage. In addition to cytotoxicity to epithelial cells, pneumolysin is also known to disrupt alveolar epithelial cells and slow ciliary beating. Pneumolysin also attracts neutrophils, activates the complement system, and promotes nitric oxide production. Pneumococcus has numerous other factors that contribute to virulence. These factors include surface-located choline-binding proteins, phosphorylcholine esterase, cell wall stem peptides, iron transporters, IgA protease, phosphoglucomutase, NADH oxidase, pyruvate oxidase, and peptidoglycan N-acetylglucosamine deacetylase A. References: Dagan R, Greenberg D, Jacobs MR. Pneumococcal infections. In: Feigin RD, Cherry JD, Demmler GJ, Kaplan SL, eds. Textbook of Pediatric Infectious Diseases. Vol 1. 5th ed. Philadelphia, PA: Saunders (Elsevier Science); 2004:1204-1258. Mitchell AM, Mitchell TJ. Streptococcus pneumoniae: virulence factors and variation. Clin Microbiol Infect. 2010;16:411-418. DOI: 10.1111/j.1469-0691.2010.03183.x. Abstract accessed September 2010 at: http://www.ncbi.nlm.nih.gov/pubmed/20132250. American Board of Pediatrics Content Specification(s): Understand the pathophysiology and pathogenesis of infectious pneumonitis

Feb 2010 - Question 1 You hypothesize that a new drug augments cardiac contractility and plan an experiment to test the hypothesis in an intact piglet model. Preliminary studies show that the hemodynamic response to the drug is complex, including changes in preload, afterload, and heart rate. To sort out these effects, you design an experiment in which piglets are instrumented to place crystals on the myocardium that allow continuous estimation of left ventricular volume. You place catheters in the aorta and the left ventricle for pressure measurement and in the inferior vena cava for drug infusion. After recovery from surgery, awake animals can be monitored continuously, and left ventricular pressure-volume curves can be constructed on a beat-by-beat basis. You obtain Curve A (Fig. 1) at baseline. You infuse phenylephrine continuously to increase afterload without altering contractility. Once vital signs are stabile, you obtain Curve B (Fig. 2). A comparison of the curves is shown in Figure 3. After discontinuing phenylephrine and allowing vital signs to stabilize, you infuse the experimental drug. Following stabilization, you obtain Curve C, and you obtain Curve D after adding phenylephrine to the experimental treatment (Fig. 4). A. left ventricular (LV) pressure at C2 to that at A2 B. LV pressure at C3 to that at A3 C. LV volume at C4 to that at A4 D. LV volume change from C3 to C4 to that from A3 to A4 E. slope of the line from C4 to D4 to that from A4 to B4

E The experiment described in the vignette uses the response to afterload (phenylephrineinduced vasoconstriction) as a measure of a drug's effect on contractility. In the intact animal, changes in afterload generally also cause changes in preload, and changes in contractility may cause changes in preload and afterload. However, it is possible to study contractility in intact animals, independent of simultaneous changes in these other factors, using the ventricular pressure-volume curve. As shown in Curve A (Fig. 1), ventricular filling raises pressure in the ventricle. Pressure at point A2 corresponds to end diastolic pressure, and the heart contracts from that pressure to the pressure above the aortic valve (A3) before any blood can be ejected. This phase is termed isovolemic contraction. The heart then ejects blood into the aorta, and its volume diminishes to that at A4 before aortic valve closure. The end systolic pressure-volume point (A4) defines the volume below which the heart cannot empty at the pressure existent in the aorta at the end of ejection. This pressurevolume point is the determinant of stroke volume. By analogy, consider the last chin-up athletes perform when exercising. As they strain to shorten their biceps, they reach a point of no further contraction. The length tension point essentially describes the contractility of the biceps. The end-systolic pressure-volume point is not fixed. If the heart is afterloaded, it cannot eject as fully. Hence, point B4 is to the right of A4 (Fig. 3). Using multiple doses of phenylephrine to achieve different degrees of afterload allows the investigator to generate a family of pressure-volume curves, all of which have an end systolic pressure-volume point on a line through A4 and B4. The slope of that line is believed to be a preload and afterload insensitive measure of contractility. The line connecting C4 and D4 (Fig. 4), obtained during test drug infusion, has a steeper slope than the line connecting A4 and B4 (Fig. 3), largely because point D4 reflects a lower endsystolic ventricular volume than point B4. This more complete ejection indicates that the experimental drug augments cardiac contractility. The line passing through C4 and D4 essentially sets the volume to which the heart can empty at any given level of afterload. Comparison of the pressure at C2 with that at A2 contrasts the end-diastolic pressure on and off the test drug at basal afterload; it is not a measure of contractility. Comparison of pressure at C3 to that at A3 contrasts the afterload on and off the test drug. In the intact animal, this is the aortic pressure generated by the cardiac output as it crosses resistance vessels in diastole. If the test drug increases cardiac output, it may raise afterload, but the change in afterload might reflect a change in heart rate or in systemic vascular resistance and is not itself indicative of a change in contractility. The change in LV volume from C3 to C4 is the stroke volume. The difference in stroke volume between Curve C and Curve A might reflect differences in preload or afterload and is not itself a measure of contractility. The LV volume at C4 and A4 represents end-systolic volumes. A decline in LV volume at the end of ejection suggests more complete emptying but is influenced by preload and afterload, as discussed previously. References: Graham TP Jr. Disorders of the circulation: myocardial dysfunction. In: Fuhrman BP, Zimmerman JJ, eds. Pediatric Critical Care. 2nd ed. St. Louis, Mo: Mosby Inc; 1998:261-271 Graham TP Jr. Ventricular performance in congenital heart disease. Circulation. 1991;84:2259-2274. Accessed November 2009 at: http://circ.ahajournals.org/cgi/reprint/84/6/2259 American Board of Pediatrics Content Specification(s): Interpret pressure‑volume relationships Understand how to quantitate contractility (slope of pressure‑volume curves)

August 2010 - Question 2 You have developed an interest in recombinant bactericidal/permeability-increasing protein as a treatment modality for meningococcemia. Data from a previous randomized controlled trial of the therapy failed to demonstrate an effect on mortality because of a lower than anticipated mortality rate in the placebo arm. However, more patients treated with recombinant bactericidal/permeability-increasing protein were found to have a functional outcome similar to that of their premeningococcemia baseline. Consequently, you wish to design a similar randomized, placebo-controlled trial but using pediatric overall performance category (POPC) scores as a marker of functional outcome as your primary outcome (Figure). Figure In comparing the POPC scores between controls and treated patients, the statistical analysis 7/18/12 August 2010.prepicu.courses.aap.org/subscriber/coursenav/august?page=2 2/2 that you should apply would be: A. Cochran-Mantel-Haenszel test B. Fisher exact test C. independent 2-sample t test D. Mann-Whitney U test E. paired t test

The data obtained from the pediatric overall performance category (POPC) scores must be considered ordinal and not cardinal. Cardinal data derive from a scale where the distance between 2 points is meaningful (eg, blood pressure measurements, height, and so on). In contrast, ordinal data refer to those data that can be ordered, but there is no meaning to the mathematical difference between 2 points. As such, a nonparametric analysis is indicated. Nonparametric statistical analyses should also be used when the underlying distribution of the data cannot be assumed to be normal and the central limit theorem is inapplicable. The Mann-Whitney U test, also known as the Wilcoxon rank sum test, is used to compare 2 independent samples for data that are only ordinal and not cardinal and/or when the normal distribution of the data cannot be assumed. It is the analogue of the independent, 2-sample t test used for normally distributed cardinal data. This statistical tool groups all the POPC scores of the 2 groups and then ranks them from lowest to highest. Subsequently, the rank values from the control group are added, and the null hypothesis is rejected if the sum is too small or too large. Because the data described in the vignette are ordinal, a parametric statistical analysis is not appropriate. Parametric analyses are used for normally distributed cardinal data. The independent, 2-sample (unpaired) t test is a parametric analysis that compares the means of 2 independent groups. Consequently, applying this test to the proposed study would be inappropriate because the data are ordinal. The paired t test is used in a similar manner but when the 2 groups are dependent. For example, a trial designed to assess the response of a physiologic parameter (eg, heart rate) before and after administration of a medication on the same patient would lend itself well to a paired t test. A Fisher exact test is used to assess the null hypothesis of no association between 2 nominal (categorical) variables: a primary factor (eg, treatment group) and a response variable. Nominal variables refer to those variables that are strictly categorical with no numerical value. The Fisher exact test may be applied to ordinal data; however, its application would entail some loss of analytical power. The POPC score represents an ordinal and not a nominal variable, and thus, this statistical tool would not make full use of the data in the vignette. The Fisher exact test assesses the null hypothesis of no association between categorical variables reported in a standard r x c (with r indicating the number of rows and c the number of columns) table by calculating the difference between the observed and expected values. In general, the Fisher exact test is recommended over the χ2 test when the margins of the r x c table are uneven or there is an expected count of less than 5 in any of the cells, a situation in which the χ2 estimate is likely to be inaccurate. As described above, χ2 and Fisher exact testing allow for testing the relationship between a factor and a response variable when both are categorical variables. However, there are frequently confounding variables that may influence that response. The Cochran-Mantel- Haenszel test provides a mechanism for controlling for such confounding variables. The test compares groups for a categorical variable but adjusts for control variables. The data are presented as a series of r x c contingency tables, often 2 x 2, stratified on the basis of the potentially confounding variable. Traditionally, the rows correspond to the treatment category and the columns to the response variables stratified into multiple tables on the basis of the confounding variables. References: Armitage P, Colton T. Encyclopedia of Statistics. Vol 2. Chichester, England: John Wiley & Sons Inc; 2005:1519-1522. Armitage P, Colton T. Encyclopedia of Statistics. Vol 3. Chichester, England: John Wiley & Sons Inc; 2005:2378-2390. Fiser DH. Assessing the outcome of pediatric intensive care. J Pediatr. 1992;121:68-74. Abstract accessed April 2010 at: http://www.ncbi.nlm.nih.gov/pubmed/1625096. Levin M, Quint PA, Goldstein B, et al; rBPI21 Meningococcal Sepsis Study Group. Recombinant bactericidal/permeability-increasing protein (rBPI21) as adjunctive treatment for children with severe meningococcal sepsis: a randomised trial. Lancet. 2000;356:961- 967. Abstract accessed April 2010 at: http://www.ncbi.nlm.nih.gov/pubmed/11041396. Rosner B. Fundamentals of Biostatistics. 4th ed. Belmont, CA: Duxbury Press/Wadsworth Publishing Company; 1995:551-579. Simple Interactive Statistical Analysis. Accessed March 18, 2010 at: www.quantitativeskills.com/sisa/statistics/fishrhlp.htm. Statistics.com. Accessed March 18, 2010 at: www.statistics.com/resources/glossary/g/gcmhtest.php American Board of Pediatrics Content Specification(s): Understand the appropriate use of parametric and nonparametric statistical tests


Related study sets

SMSH-PROJECTS-HALF LIFE CALCULATIONS-10/12/23

View Set

NUR 236 PrepU Chapter 36: Pain Management in Children

View Set

MicroEcon TESTS (1,2,3) - Final 22

View Set

Java Programming - Chapter 7: Arrays

View Set

Biology - Quiz 2: Properties of Compounds

View Set

Final Exam: World History Study Guide Part 1

View Set